Population health exam 3 (chapters: 45, 9, 35, 46, 47, 38, 39, 48)

Lakukan tugas rumah & ujian kamu dengan baik sekarang menggunakan Quizwiz!

The patient is having at least 75% of nutritional needs met by enteral feeding, so the health care provider has ordered the parenteral nutrition (PN) to be discontinued. However, the nurse notices that the PN infusion has fallen behind. What should the nurse do? a. Increase the rate to get the volume caught up before discontinuing. b. Stop the infusion as ordered. c. Taper infusion gradually. d. Hang 5% dextrose.

ANS: C Sudden discontinuation of PN can cause hypoglycemia. PN must be tapered off. Usually, 10% dextrose is infused when PN solution is suddenly discontinued. Too rapid administration of hypertonic dextrose (PN) can result in an osmotic diuresis and dehydration. If an infusion falls behind schedule, the nurse should not increase the rate in an attempt to catch up.

A nurse is discussing the changing demographics of the US population. What is expected to be the fastest growing racial ethnic group by 2060? a. Hispanic b. Asian c. Multiracial d. Non-Hispanic Blacks

ANS: C The changing demographics of the US population create challenges for the health care system and health care providers. By the year 2060, the percentage of racial and ethnic minority groups in the United States is expected to climb to 32% of the population. The fastest-growing racial ethnic group in the United States is people whose ancestry is from two or more races, and this group is projected to grow by 200%. The next fastest growing is the Asian population, which is projected to double, followed by the Hispanic population.

The nurse caring for an immobile patient wants to decrease the risk of the formation of pressure ulcers. Which action will the nurse take first? a. Offer favorite fluids. b. Turn the patient every 2 hours. c. Determine the patient's risk factors. d. Encourage increased quantities of carbohydrates and fats.

ANS: C The first step in prevention is to assess the patient's risk factors for pressure ulcer development. When a patient is immobile, the major risk to the skin is the formation of pressure ulcers. Nursing interventions focus on prevention. Offering favorite fluids, turning, and increasing carbohydrates and fats are not the first steps. Determining risk factors is first, so interventions can be implemented to reduce or eliminate those risk factors.

A nurse is following the no-lift policy when working to prevent personal injury from twisting. Which type of personal back injury is the nurse most likely trying to prevent? a. Thoracic b. Cervical c. Lumbar d. Sacral

ANS: C The most common back injury for nurses is strain on the lumbar muscle group, which includes the muscles around the lumbar vertebrae that are commonly injured by twisting during the l

The nurse is caring for a group of patients. Which patient will the nurse see first? a. A patient with chronic obstructive pulmonary disease doing stretching exercises b. A patient with diabetes mellitus carrying hard candy while doing exercises c. A patient with a heart attack doing isometric exercises d. A patient with hypertension doing Tai Chi exercises

ANS: C The nurse must see the myocardial infarction patient first to stop this type of exercise. It is important to understand the energy expenditure (increased respiratory rate and increased work on the heart) associated with isometric exercises because the exercises are sometimes contraindicated in certain patients' illnesses (e.g., myocardial infarction or chronic obstructive pulmonary disease). All the rest are appropriate. Stretching exercises are beneficial for patients with chronic obstructive pulmonary disease. Also instruct patients to perform low- to moderate-intensity exercises, carry a concentrated form of carbohydrates (sugar packets or hard candy), and wear a medical alert bracelet. The effect of a Tai Chi exercise program has demonstrated a significant reduction in systolic and diastolic blood pressures.

The nurse is caring for a patient who cannot bear weight but needs to be transferred from the bed to a chair. The nurse decides to use a transportable hydraulic lift. What action indicates the nurse is aware of appropriate hydraulic life use? a. Places a horseshoe-shaped base on the opposite side from the chair. b. Removes straps before lowering the patient to the chair. c. Hooks longer straps to the bottom of the sling. d. Attaches short straps to the bottom of the sling.

ANS: C The nurse should attach the hooks on the strap to the holes in the sling. Short straps hook to top holes of the sling; longer straps hook to the bottom of the sling. The horseshoe-shaped base goes under the side of the bed on the side with the chair. Position the patient and lower slowly into the chair in accordance with manufacturer guidelines to safely guide the patient into the back of the chair as the seat descends; then remove the straps and the mechanical/hydraulic lift.

Which behavior indicates the nurse is using a team approach when caring for a patient who is experiencing alterations in mobility? a. Delegates assessment of lung sounds to nursing assistive personnel. b. Becomes solely responsible for modifying activities of daily living. c. Consults physical therapy for strengthening exercises in the extremities. d. Involves respiratory therapy for altered breathing from severe anxiety level

ANS: C The nurse should collaborate with other health care team members such as physical or occupational therapists when considering mobility needs. For example, physical therapists are a resource for planning ROM or strengthening exercises. Nurses often delegate some interventions to nursing assistive personnel, but assessment of lung sounds is the nurse's responsibility. Nursing assistive personnel may turn and position patients, apply elastic stockings, help patients use the incentive spirometer, etc. Occupational therapists are a resource for planning activities of daily living that patients need to modify or relearn. A mental health advanced practice nurse or psychologist should be used for severe anxiety.

The nurse is caring for a group of patients. Which patient will the nurse see first? a. A patient with a Stage IV pressure ulcer b. A patient with a Braden Scale score of 18 c. A patient with appendicitis using a heating pad d. A patient with an incision that is approximated

ANS: C The nurse should see the patient with an appendicitis first. Warm applications are contraindicated when the patient has an acute, localized inflammation such as appendicitis because the heat could cause the appendix to rupture. Although a Stage IV pressure ulcer is deep, it is not as critical as the appendicitis patient. The total Braden score ranges from 6 to 23; a lower total score indicates a higher risk for pressure ulcer development. A score of 18 can be assessed later. A healing incision is approximated (closed); this is a normal finding and does not need to be seen first.

The nurse is caring for patients with ostomies. In which ostomy location will the nurse expect very liquid stool to be present? a. Sigmoid b. Transverse c. Ascending d.Descending

ANS: C The path of digestion goes from the ascending, across the transverse, to the descending and finally passing into the sigmoid; therefore, the least formed stool (very liquid) would be in the ascending.

The nurse is admitting a patient to the hospital. The patient claims to be a very spiritual person but does not practice any specific religion. How will the nurse interpret this finding? a. This indicates a strong religious affiliation. b. This statement is contradictory. c. This statement is reasonable. d. This indicates a lack of hope.

ANS: C The patient's statement is reasonable and is not contradictory. Many people tend to use the terms spirituality and religion interchangeably. Although closely associated, these terms are not synonymous. Religious practices encompass spirituality, but spirituality does not need to include religious practice. When a person has the attitude of something to live for and look forward to, hope is present.

The nurse is planning care for a group of patients. Which task will the nurse assign to the nursing assistive personnel (AP)? a. Performing the first postoperative pouch change b. Maintaining a nasogastric tube c. Administering an enema d. Digitally removing stool

ANS: C The skill of administering an enema can be delegated to an AP. The skill of inserting and maintaining a nasogastric (NG) tube cannot be delegated to an AP. The nurse should do the first postoperative pouch change. Digitally removing stool cannot be delegated to nursing assistive personnel.

A nurse is preparing to administer an enteral feeding. In which order will the nurse implement the steps, starting with the first one? 1. Elevate head of bed to at least 30 degrees. 2. Check for gastric residual volume. 3. Flush tubing with 30 mL of water. 4. Verify tube placement. 5. Initiate feeding. a. 4, 2, 1, 5, 3 b. 2, 4, 1, 3, 5 c. 1, 4, 2, 3, 5 d. 2, 1, 4, 5, 3

ANS: C The steps for an enteral feeding are as follows: place patient in high-Fowler's position or elevate head of bed to at least 30 (preferably 45) degrees, verify tube placement, check for gastric residual volume, flush tubing with 30 mL of water, and initiate feeding.

A nurse is teaching a patient how to meditate. Which behavior presented by the patient indicates effective learning? a. Lying on the floor b. Breathing quickly c. Focusing on an particular image d. Engaging for 10 minutes every day

ANS: C The steps of meditation include sitting in a comfortable position with the back straight; breathe slowly; and focus on a sound, prayer, or image. Meditation should occur for 10 to 20 minutes twice a day.

The nurse is caring for a Asian patient using the teach-back technique. Which action by the nurse indicates successful implementation of this technique? a. Asks, ―Does this make sense?‖ b. Asks, ―Do you think you can do this at home?‖ c. Asks, ―What will you tell your spouse about changing the dressing?‖ d. Asks, ―Would you tell me if you don't understand something, so we can go over it?‖

ANS: C The teach-back technique asks open-ended questions, like what will you tell your spouse about changing the dressing, to verify a patient's understanding. When using the teach-back technique, do not ask a patient, ―Do you understand?‖ or ―Do you have any questions?‖ ―Does this make sense?‖, and ―Do you think you can do this at home?‖ are closed-ended questions. ―Would you tell me if you don't understand something, so we can go over it?‖ is not verifying a patient's understanding about the teaching.

Which action will the nurse take to reduce the risk of excoriation to the mucosal lining of the patient's nose from a nasogastric tube? a. Instill Xylocaine into the nares once a shift. b. Tape tube securely with light pressure on nare. c. Lubricate the nares with water-soluble lubricant. d. Apply a small ice bag to the nose for 5 minutes every 4 hours.

ANS: C The tube constantly irritates the nasal mucosa, increasing the risk of excoriation. Frequent lubrication with a water-soluble lubricant decreases the likelihood of excoriation and is less toxic than oil-based if aspirated. Xylocaine is used to treat sore throat, not nasal mucosal excoriation. While the tape should be secure, pressure will increase excoriation. Ice is not applied to the nose.

Upon palpation, the nurse notices that the bladder is firm and distended; the patient expresses an urge to urinate. Which question is most appropriate? a. ―Does your urinary problem interfere with any activities?‖ b. ―Do you lose urine when you cough or sneeze?‖ c. ―When was the last time you voided?‖ d. ―Are you experiencing a fever or chills?

ANS: C To obtain an accurate assessment, the nurse should first determine the source of the discomfort. Urinary retention causes the bladder to be firm and distended; time of last void is most appropriate. Further assessment to determine the pathology of the condition can be performed later. Questions concerning fever and chills, interference with any activities, and losing urine during coughing or sneezing focus on specific pathological conditions.

The wound care nurse is monitoring a patient with a Stage III pressure ulcer whose wound presents with healthy tissue. How should the nurse document this ulcer in the patient's medical record? a. Stage I pressure ulcer b. Healing Stage II pressure ulcer c. Healing Stage III pressure ulcer d. Stage III pressure ulcer

ANS: C When a pressure ulcer has been staged and is beginning to heal, the ulcer keeps the same stage and is labeled with the words ―healing stage‖ or healing Stage III pressure ulcer. Once an ulcer has been staged, the stage endures even as the ulcer heals. This ulcer was labeled a Stage III, and it cannot return to a previous stage such as Stage I or II. This ulcer is healing, so it is no longer labeled a Stage III.

The nurse is evaluating care of a patient for crutches. Which finding indicates a successful outcome? a. The top of the crutch is three to four finger widths from the armpit. b. The elbows are slightly flexed at 30 to 35 degrees when the patient is standing. c. The tip of the crutch is 4 to 6 inches anterior to the front of the patient's shoes. d. The position of the handgrips allows the axilla to support the patient's body weight.

ANS: C When crutches are fitted, the tip of the crutch is 4 to 6 inches anterior to the front of the patient's shoes, and the length of the crutch is two to three finger widths from the axilla. Position the handgrips so the axillae are not supporting the patient's body weight. Pressure on the axillae increases risk to underlying nerves, which sometimes results in partial paralysis of the arm. Determine correct position of the handgrips with the patient upright, supporting weight by the handgrips with the elbows slightly flexed at 20 to 25 degrees

A nurse is caring for a patient with a postsurgical wound. When planning care, which goal will be the priority? a. Reduce dependent nitrogen balance. b. Maintain negative nitrogen balance. c. Promote positive nitrogen balance. d. Facilitate neutral nitrogen balance.

ANS: C When intake of nitrogen is greater than output, the body is in positive nitrogen balance. Positive nitrogen balance is required for growth, normal pregnancy, maintenance of lean muscle mass and vital organs, and wound healing. Negative nitrogen balance occurs when the body loses more nitrogen than the body gains. Neutral nitrogen balance occurs when gain equals loss and is not optimal for tissue healing. There is no such term as dependent nitrogen balance.

The nurse is completing a skin risk assessment using the Braden Scale. The patient has slight sensory impairment, has skin that is rarely moist, walks occasionally, and has slightly limited mobility, along with excellent intake of meals and no apparent problem with friction and shear. Which score will the nurse document for this patient? a. 15 b. 17 c. 20 d. 23

ANS: C With use of the Braden Scale, the total score is a 20. The patient receives 3 for slight sensory perception impairment, 4 for skin being rarely moist, 3 for walks occasionally, 3 for slightly limited mobility, 4 for intake of meals, and 4 for no problem with friction and shear.

The patient reports being tired and weak and lacks energy. Upon assessment, the nurse finds that patient has gained weight, and blood pressure and pulse are elevated after climbing stairs. Which nursing diagnosis will the nurse add to the care plan? a. Fatigue b. Ineffective coping c. Activity intolerance d. Decreased cardiac output

ANS: C You consider nursing diagnoses of Activity intolerance or Fatigue in a patient who reports being tired and weak. Further review of assessed defining characteristics (e.g., abnormal heart rate and verbal report of weakness and the assessment findings occurring during the activity of climbing the stairs) leads to the definitive diagnosis (Activity intolerance). There is no data to support ineffective coping or decreased cardiac output.

Describe urine color by age: -1-3 days old: -4 days old: -5 days to 2 years old:

-1-3 days old -1 day old: reddish orange -2 days old: pale orange/pinkish -3 days old- yellow/orange -4 days old - lemon-yellow -5 days old- 2 years - pale yellow

A nurse is using Campinha-Bacote's model of cultural competency to improve cultural care. Which actions describe the components the nurse is using? a. In-depth self-examination of one's own background b. Ability to assess factors that influence treatment and care c. Sufficient comparative understanding of diverse groups d. Motivation and commitment to continue learning about cultures e. Cross-cultural interaction that develops communication skills 1. Cultural skills 2. Cultural desires 3. Cultural awareness 4. Cultural knowledge 5. Cultural encounters

1. ANS: B -OBJ:Explain the concepts of cultural awareness, cultural knowledge, cultural skill, cultural encounter, and cultural desire in the cultural competence model. 2. ANS: D -OBJ:Explain the concepts of cultural awareness, cultural knowledge, cultural skill, cultural encounter, and cultural desire in the cultural competence model. 3. ANS: A -OBJ:Explain the concepts of cultural awareness, cultural knowledge, cultural skill, cultural encounter, and cultural desire in the cultural competence model. 4. ANS: C -OBJ:Explain the concepts of cultural awareness, cultural knowledge, cultural skill, cultural encounter, and cultural desire in the cultural competence model. 5. ANS: E:- OBJ:Explain the concepts of cultural awareness, cultural knowledge, cultural skill, cultural encounter, and cultural desire in the cultural competence model.

Upon assessment a nurse discovers postural abnormalities on several patients. Match the abnormalities to the findings the nurse observed. a. Lateral-S- or C-shaped spinal column with vertebral rotation b. Exaggeration of anterior convex curve of lumbar spine c. Increased convexity in curvature of thoracic spine 1. Lordosis 2. Kyphosis 3. Scoliosis

1. ANS: B DIF: OBJ:Discuss physiological and pathological influences on mobility. 2. ANS: C OBJ:Discuss physiological and pathological influences on mobility. Physiological Adaptation 3. ANS: A OBJ:Discuss physiological and pathological influences on mobility.

A nurse is providing spiritual care to a group of patients. Match the group to their belief. a. Avoid caffeine. b. Says prayers five times a day. c. Shellfish is not eaten. d. Often follow a vegetarian diet. e. Generally fast on religious Holy days. 1. Hinduism 2. Buddhism 3. Islam 4. Judaism 5. Mormonism

1. ANS: D OBJ:Discuss the influence of spirituality on patients' health practices. 2. ANS: E OBJ:Discuss the influence of spirituality on patients' health practices. 3. ANS: B OBJ:Discuss the influence of spirituality on patients' health practices. 4. ANS: C OBJ:Discuss the influence of spirituality on patients' health practices. 5. ANS: A OBJ:Discuss the influence of spirituality on patients' health practices.

The nurse is caring for patients who need wound dressings. Match the type of dressing the nurse applies to its description. a. Absorbs drainage through the use of exudate absorbers in the dressing b. Very soothing to the patient and do not adhere to the wound bed c. Barrier to external fluids/bacteria but allows wound to ―breathe‖ d. Manufactured from seaweed and comes in sheet and rope form e. Oldest and most common absorbent dressing 1. Gauze 2. Transparent 3. Hydrocolloid 4. Hydrogel 5. Calcium alginate

1. ANS: E OBJ:Develop a nursing care plan for a patient with impaired skin integrity. 2. ANS: C OBJ:Develop a nursing care plan for a patient with impaired skin integrity. 3. ANS: A OBJ:Develop a nursing care plan for a patient with impaired skin integrity. 4. ANS: B OBJ:Develop a nursing care plan for a patient with impaired skin integrity. 5. ANS: D OBJ:Develop a nursing care plan for a patient with impaired skin integrity.

A patient with a fecal impaction has an order to remove stool digitally. In which order will the nurse perform the steps, starting with the first one? 1. Obtain baseline vital signs. 2. Apply clean gloves and lubricate. 3. Insert index finger into the rectum. 4. Identify patient using two identifiers. 5. Place patient on left side in Sims' position. 6. Massage around the feces and work down to remove. a. 4, 1, 5, 2, 3, 6 b. 1, 4, 2, 5, 3, 6 c. 4, 1, 2, 5, 3, 6 d. 1, 4, 5, 2, 3, 6

ANS: A The steps for removing a fecal impaction are as follows: identify patient using two identifiers, obtain baseline vital signs, place on left side in Sims' position, apply clean gloves and lubricate, insert index finger into the rectum, and gently loosen the fecal mass by massaging around it and work the feces downward toward the end of the rectum.

8. Which explanation provided by the nurse is the most accurate meaning for "providing culturally congruent care"? 1. It fits the patient's valued life patterns and set of meanings. 2. It is the same set of values as those of the health care team member providing daily care. 3. It holds one's own way of life as superior to those of others. 4. It redirects the patient to a more socially expected set of values.

ANS: 1

A nurse is caring for a patient who has just had a near-death experience (NDE) following a cardiac arrest. Which intervention by the nurse best promotes the spiritual well-being of the patient after the NDE? 1. Allowing the patient to discuss the experience 2. Referring the patient to pastoral care 3. Having the patient talk to another patient who had an NDE 4. Offering to pray for the patient

ANS: 1

A nurse is helping a patient perform active assisted ROM in the right elbow. Which statement describes the correct technique? 1. Support elbow by holding distal part of extremity. 2. Grasp joint with fingers to provide support. 3. Have patient move joint independently. 4. Move the joint past the point of resistance. 5. Perform the exercise three times during the session, and gradually build up to more.

ANS: 1

A patient has been on bed rest for over 5 days. Which of these findings during the nurse's assessment may indicate a complication of immobility? 1. Decreased peristalsis 2. Decreased heart rate 3. Increased blood pressure 4. Increased urinary output

ANS: 1

During a nursing assessment a patient displayed several behaviors. Which behavior suggests the patient may have a health literacy problem? 1. Patient has difficulty completing a registration form at a medical office 2. Patient asks for written information about a health topic 3. Patient speaks Spanish as primary language 4. Patient states unfamiliarity with a newly ordered medicine

ANS: 1

During the administration of a warm tap-water enema, a patient starts to have cramping abdominal pain that he rates a 6 out of 10. What nursing action should the nurse take first? 1. Stop the instillation. 2. Ask the patient to take deep breaths to decrease the pain. 3. Tell the patient to bear down as he would when having a bowel movement. 4. Continue the instillation; then administer a pain medication.

ANS: 1

The nurse is caring for an older adult in a long-term care setting. The nurse reviews the medical record to find that the patient has progressive loss of total bone mass. The patient's history and tendency to take smaller steps with feet kept closer together will most likely result in which of the following? 1. Increase the patient's risk for falls and injuries 2. Result in less stress on the patient's joints 3. Decrease the amount of work required for patient movement 4. Allow for mobility in spite of the aging effects on the patient's joints

ANS: 1

The nurse is changing the PN tubing. What action should the nurse take to prevent an air embolus? 1. Have the patient turn on the left side and perform a Valsalva maneuver. 2. Have the patient cough vigorously when tubing is disconnected. 3. Have the patient take a deep breath and hold it. 4. Place patient in supine position with head of bed elevated 90 degrees.

ANS: 1

What is the removal of devitalized tissue from a wound called? 1. Debridement 2. Pressure distribution 3. Negative-pressure wound therapy 4. Sanitization

ANS: 1

Which action can a nurse delegate to AP? 1. Performing glucose monitoring every 6 hours on a stable patient 2. Teaching the patient about the need for enteral feeding 3. Administering enteral feeding bolus after tube placement has been verified 4. Evaluating the patient's tolerance of the enteral feeding

ANS: 1

Which of the following is the proper sequence for a four-point crutch gait? Set of three gait diagrams are drawn next to tripod position of crutch. 1. Move right crutch forward followed by left leg. Then, left crutch followed by right leg. 2. Move both crutch forward followed by right leg forward. 3. Move left crutch and right leg forward. Then, right crutch and left leg forward.

ANS: 1

A nurse is seeing all of these patients in the community health clinic. The nurse identifies which patient(s) as having a health disparity? (Select all that apply.) 1. A patient who has a homosexual sexual preference 2. A patient unable to access primary care services 3. A patient living with chronic schizophrenia 4. A family who relies on public transportation 5. A patient who has a history of hypertension

ANS: 1, 2, 3

Which statements made by a nursing student about the teach-back technique show understanding of the technique? (Select all that apply.) 1. "After teaching a patient how to use an inhaler, I need to use the teach-back technique to test my patient's ability to use the inhaler correctly." 2. "The teach-back technique is an ongoing process of asking patients for feedback." 3. "Using teach-back will help me identify explanations and communication strategies that my patients will most commonly understand." 4. "Using pictures, drawings, and models can enhance the effectiveness of the teach-back technique." 5. "When doing my patient teaching, I will use plain language to make the material easier to understand for the patient."

ANS: 1, 2, 3, 4, 5

Health care agencies must provide which of the following based on federal civil rights laws? (Select all that apply.) 1. Provide language assistance services at all points of contact free of charge. 2. Provide auxiliary aids and services, such as interpreters, note takers, and computer-aided transcription services. 3. Use patients' family members to interpret difficult topics. 4. Ensure that interpreters are competent in medical terminology. 5. Provide language assistance to all patients who speak limited English or are deaf.

ANS: 1, 2, 4, 5;

A 35-year-old woman has Medicaid coverage for herself and two young children. She missed an appointment at the local health clinic to get an annual mammogram because she has no transportation. She gets the annual screening because her mother had breast cancer. Which of the following are social determinants of this woman's health? (Select all that apply.) 1. Medicaid insurance 2. Annual screening 3. Mother's history of breast cancer 4. Lack of transportation 5. Woman's age

ANS: 1, 4, 5;

Before transferring a patient from the bed to a stretcher, which assessment data does the nurse need to gather? (Select all that apply.) 1. Patient's weight 2. Patient's activity tolerance 3. Patient's level of mobility 4. Recent laboratory values 5. Nutritional intake 6. Safe mobility algorithm

ANS: 1,2,3

Which skills does the nurse teach a patient with a new colostomy before discharge from the health care agency? (Select all that apply.) 1. How to change the pouch 2. How to empty the pouch 3. How to open and close the pouch 4. How to irrigate the colostomy 5. How to determine whether the ostomy is healing appropriately

ANS: 1,2,3,5

A patient is admitted to a rehabilitation facility for cardiac rehabilitation following open heart surgery. The patient is 72 years old, 4 days postoperative, and reportedly was walking with a one-person assist in the hospital before transfer. The patient has a history of hypertension. His wife accompanies him at the time of transfer. Which of the following assessment data would you collect for this patient? (Select all that apply.) 1. Condition of surgical wound 2. Patient's expectations of rehabilitation 3. Previous hospitalization experience 4. Vital signs 5. Ability to sit on side of bed unassisted 6. Gait and balance 7. History of recent weight changes 8. Social support from wife

ANS: 1,2,4,5,6,8

A nurse is teaching a patient about the warning signs of possible colorectal cancer according to the American Cancer Society guidelines. Which statements reflect that the patient understands the teaching? (Select all that apply.) 1. "I need to let my doctor know if my bowel habits start to change." 2. "Blood in the stool is one warning sign I need to look for." 3. "Muscle aches are common in people with colorectal cancer." 4. "It is not normal to see food particles in the stool." 5. "Some people with colorectal cancer have unexplained abdominal or back pain."

ANS: 1,2,5

The effects of immobility on the cardiac system include which of the following? (Select all that apply.) 1. Thrombus formation 2. Increased cardiac workload 3. Weak peripheral pulses 4. Irregular heartbeat 5. Orthostatic hypotension

ANS: 1,2,5

A patient is receiving total parenteral nutrition (TPN). What are the primary interventions the nurse should follow to prevent a central line infection? (Select all that apply.) 1. Change the dressing using sterile technique. 2. Change TPN containers every 48 hours. 3. Change the TPN tubing every 24 hours. 4. Monitor glucose levels to watch and assess for glucose intolerance. 5. Elevate head of the bed 45 degrees to prevent aspiration.

ANS: 1,3

The nurse is caring for a 50-year-old woman visiting the outpatient medicine clinic. The patient has had type 1 diabetes mellitus since age 13. She has numerous complications from her disease, including reduced vision, heart disease, and severe numbness and tingling of the extremities. Knowing that spirituality helps patients cope with chronic illness, which of the following principles should the nurse apply in practice? (Select all that apply.) 1. Pay attention to the patient's spiritual identity throughout the course of her illness. 2. Select interventions that you know scientifically support spiritual well-being. 3. Listen to the patient's story each visit to the clinic, and offer a compassionate presence. 4. When the patient questions the reason for her longtime suffering, try to provide answers. 5. Consult with a spiritual care adviser, and have the adviser recommend useful interventions.

ANS: 1,3

A 51-year-old adult comes to a medical clinic for an annual physical exam. The patient is found to be slightly overweight and reports being inactive, walking only 2 to 3 times a week with his wife after work. He has good muscle strength and coordination of lower extremities. Which of the following recommendations from the Physical Activity Guidelines for Americans should the nurse suggest? (Select all that apply.) 1. Move more and sit less throughout the day. 2. Participate in at least 90 minutes a week of moderate-intensity aerobic physical activity. 3. Perform muscle-strengthening activities using light weights on 2 or more days a week. 4. Walk at a vigorous pace with wife at least 150 minutes over five days a week. 5. Focus on balance training.

ANS: 1,3,4

Medical adhesives, such as tape securing a wound dressing, cause MARSI. Which of the following interventions reduce the risk for MARSI? (select all that apply) 1. Gently loosen the ends of the tape and gently pull the outer end parallel with the skin surface toward the wound. 2. Change dressing only when saturated. 3. Apply adhesive remover. 4. Use Montgomery ties to secure the dressing. 5. Immobilize area of wound.

ANS: 1,3,4

What should the nurse teach a young woman with a history of UTIs about UTI prevention? (Select all that apply.) 1. Maintain regular bowel elimination. 2. Limit water intake to 1 to 2 glasses a day. 3. Wear cotton underwear. 4. Cleanse the perineum from front to back. 5. Practice pelvic muscle exercises (Kegel) daily.

ANS: 1,3,4

Which of the following are measures to reduce tissue damage from shear? (Select all that apply.) 1. Use a transfer device (e.g., transfer board). 2. Have head of bed elevated when transferring patient. 3. Have head of bed flat when repositioning patient. 4. Raise head of bed 60 degrees when patient is positioned supine. 5. Raise head of bed 30 degrees when patient is positioned supine.

ANS: 1,3,5

A patient has been hospitalized with a serious flulike infection and is on bed rest. He is receiving multiple medications through two different IV infusions and is on high-flow oxygen therapy by oxygen mask. Currently the patient's head of bed is elevated to semi-Fowler position. The patient initiates little movement and responds only to being shaken. Vitals signs are temperature, 38.6°C (101.6°F); heart rate, 88 beats/min; blood pressure 140/84 mm Hg; and respirations, 20. Which of the following assessment findings suggest that the patient has a risk for an immobility complication? (Select all that apply.) 1. High-flow oxygen therapy by mask 2. Positioned semi-Fowler 3. Temperature 38.6°C (101.6°F) 4. Receiving multiple medications 5. Initiates little movement 6. Reduced conscious response 7. Bed rest

ANS: 1,3,5,6,7

A middle-aged adult patient has limited mobility following a total knee arthroplasty. During assessment, the nurse notes that the patient is having difficulty breathing while lying supine. Which assessment data support a pulmonary issue related to immobility? (Select all that apply.) 1. Oxygen saturation of 89% 2. Irregular radial pulse 3. Diminished breath sounds in bilateral bases of lungs 4. Blood pressure 132/84 mm Hg 5. Pain reported at 3 on scale of 0 to 10 following medication 6. Respiratory rate of 26

ANS: 1,3,6

After surgery the patient with a closed abdominal wound reports a sudden "pop" after coughing. When the nurse examines the surgical wound site, the sutures are open, and small bowel sections are observed at the bottom of the now-opened wound. Which are the priority nursing interventions? (Select all that apply.) 1. Notify the health care provider. 2. Allow the area to be exposed to air until all drainage has stopped. 3. Place several cold packs over the area, protecting the skin around the wound. 4. Cover the area with sterile, saline-soaked towels immediately. 5. Cover the area with sterile gauze and apply an abdominal binder.

ANS: 1,4

A patient is scheduled to have an intravenous pyelogram (IVP) tomorrow morning. Which nursing measures should be implemented before the test? (Select all that apply.) 1. Ask the patient about any allergies and reactions. 2. Instruct the patient that a full bladder is required for the test. 3. Instruct the patient to save all urine in a special container. 4. Ensure that informed consent has been obtained. 5. Instruct the patient that facial flushing can occur when the contrast medium is given.

ANS: 1,4,5

Family members have asked for a meeting with the nursing staff of an assisted-living residential center to discuss the feasibility of their mother using a walker. The family is worried that her health is declining; they wonder whether she can use the walker safely. Which of the following instructions should the nurse give the family after assessing that it is safe for the woman to use a walker? (Select all that apply.) 1. A walker is useful for patients who have impaired balance. 2. The patient uses a walker by pushing the device forward. 3. Leaning over the walker improves the patient's balance. 4. Walkers should not be used on stairs. 5. If the patient has difficulty advancing the walker, a walker with wheels is an option.

ANS: 1,4,5

Which skin-care measures are used to manage a patient who is experiencing fecal and/or urinary incontinence? (Select all that apply.) 1. Frequent position changes 2. Keeping the buttocks exposed to air at all times 3. Using a large absorbent diaper, changing when saturated 4. Using an incontinence cleaner 5. Applying a moisture barrier ointment

ANS: 1,4,5

Which instructions does the nurse include when educating a person with chronic constipation? (Select all that apply.) 1. Increase fiber and fluids in the diet. 2. Use a low-volume enema daily. 3. Avoid gluten in the diet. 4. Take laxatives twice a day. 5. Exercise for 30 minutes every day. 6. Schedule time to use the toilet at the same time every day. 7. Take probiotics 5 times a week.

ANS: 1,5,6

The nurse is caring for a patient who is very depressed and decides to complete a spiritual assessment using the FICA tool. Using the FICA assessment tool, match the criteria on the left with the appropriate assessment question on the right. ___1.F—Faith ___2.I—Importance of spirituality ___3.C—Community ___4.A—Interventions to address spiritual needs a. Tell me if you have a higher power or authority that helps you act on your beliefs. b. Describe which activities give you comfort spiritually. c. To whom do you go for support in times of difficulty? d. Your illness has kept you from attending church. Is that a problem for you?

ANS: 1a, 2d, 3c, 4b

2. Match the pressure injury stages with the correct definition. ___ 1.Stage 1 ___ 2.Stage 2 ___ 3.Stage 3 ___ 4.Stage 4 ___ 5.Unstageable pressure injury a. Partial-thickness loss of skin with exposed dermis. The wound bed is viable, pink or red, and moist, and may also present as an intact or ruptured serum-filled blister. Adipose tissue (fat) and deeper tissues are not visible. Granulation tissue, slough, and eschar are not present. These injuries commonly result from adverse microclimate and shear in the skin over the pelvis and shear in the heel. This stage should not be used to describe moisture-associated skin damage (MASD), including incontinence-associated dermatitis (IAD), intertriginous dermatitis (ITD), medical adhesive-related skin injury (MARSI), or traumatic wounds (skin tears, burns, abrasions). b. Intact skin with a localized area of nonblanchable erythema, which may appear differently in darkly pigmented skin. Presence of blanchable erythema or changes in sensation, temperature, or firmness may precede visual changes. Color changes do not include purple or maroon discoloration; these may indicate deep-tissue pressure injury. c. Full-thickness skin and tissue loss with exposed or directly palpable fascia, muscle, tendon, ligament, cartilage, or bone in the ulcer. Slough and/or eschar may be visible. Epibole (rolled edges), undermining, and/or tunneling often occurs. Depth varies by anatomical location. If slough or eschar obscures the extent of tissue loss, this is an Unstageable pressure injury. d. Full-thickness loss of skin, in which adipose tissue (fat) is visible in the ulcer, and granulation tissue and epibole (rolled wound edges) are often present. Slough and/or eschar may be visible. The depth of tissue damage varies by anatomical location; areas of significant adiposity can develop deep wounds. Undermining and tunneling may occur. Fascia, muscle, tendon, ligament, cartilage, and bone are not exposed. If slough or eschar obscures the extent of tissue loss, this is an unstageable pressure injury. e. Full-thickness skin and tissue loss in which the extent of tissue damage within the injury cannot be confirmed because it is obscured by slough or eschar. If slough or eschar is removed, a Stage 3 or Stage 4 pressure injury will be revealed. Stable eschar (i.e., dry, adherent, intact, without erythema or fluctuance) on the heel or ischemic limb should not be softened or removed.

ANS: 1b, 2a, 3d, 4c, 5e

. Match the cultural concepts on the left with the correct definitions on the right. ____ 1. Etic worldview ____ 2. Worldview ____ 3. Cultural desire ____ 4. Intersectionality ____ 5. Emic worldview a. Factor that shapes how people perceive others and how they relate to reality b. Insider's perspective in an intercultural encounter c. A policy model that describes factors and power structures that shape and influence life d. An outsider's perspective in an intercultural encounter e. The motivation of a health care professional to "want to" engage in cultural competence

ANS: 1d, 2a, 3e, 4c, 5b.

A nurse desires to communicate with a young woman who is Serbian and who has limited experience with being in a hospital. The nurse has 10 years of experience caring for Serbian women. The patient was admitted for a serious pregnancy complication. Apply the LEARN model and match the nurse's behaviors with each step of the model. 1.___ 2.__ 3.__ 4.__ 5.__ a. The nurse notes that she has learned that fathers can visit mothers at any time in both Serbia and the United States. b. The nurse shares her perception of the woman's experiences as a patient. c. The nurse asks the patient how she can maintain bed rest when she returns home. d. The nurse attends to the patient and listens to her story about hospitals in Serbia. e. The nurse involves the patient in a discussion of the treatment

ANS: 1d, 2b, 3a, 4e, 5c

A nurse sees an AP perform the following interventions for a patient receiving continuous enteral feedings. Which action would require immediate attention by the nurse? 1. Fastening tube to the gown with new tape 2. Placing patient supine while giving a bath 3. Monitoring the patient's weight as ordered 4. Ambulating patient with enteral feedings still infusing

ANS: 2

A patient has just been diagnosed with a malignant brain tumor. She is alone; her family will not be arriving from out of town for an hour. What intervention does the nurse implement that best provides support of the patient's spiritual well-being at this time? 1. Make a referral to a professional spiritual care adviser. 2. Sit down and talk with the patient; have her discuss her feelings and listen attentively. 3. Move the patient's Bible from her bedside cabinet drawer to the top of the over-bed table. 4. Ask the patient whether she would like to learn more about the implications of having this type of tumor.

ANS: 2

The nurse is inserting a urinary catheter for a female patient, and after the catheter has been inserted 3 inches, no urine is returned. What should the nurse do next? 1. Remove the catheter and start all over with a new kit and catheter. 2. Leave the catheter there and start over with a new catheter. 3. Pull the catheter back and reinsert at a different angle. 4. Ask the patient to bear down and insert the catheter farther.

ANS: 2

What should the nurse teach family caregivers when a patient has fecal incontinence because of cognitive impairment? 1. Cleanse the skin with antibacterial soap and apply talcum powder to the buttocks. 2. Initiate a bowel or habit training program to promote continence. 3. Help the patient go to the toilet once every hour. 4. Use sanitary pads in the patient's underwear.

ANS: 2

Which nursing intervention decreases the risk for CAUTI? 1. Cleansing the urinary meatus 3 to 4 times daily with antiseptic solution 2. Hanging the urinary drainage bag below the level of the bladder 3. Emptying the urinary drainage bag daily 4. Irrigating the urinary catheter with sterile water

ANS: 2

The nurse is caring for a patient with pneumonia, who has severe malnutrition. The patient's condition places her at risk for which of the following life-threatening complications during hospitalization? (Select all that apply.) 1. Heart disease 2. Sepsis 3. Hemorrhage 4. Skin breakdown 5. Diarrhea

ANS: 2, 3, 4

A 44-year-old male patient has just been told that his wife and child were killed in an auto accident while coming to visit him in the hospital. After analyzing the assessment findings, the nurse recognizes that which statements made by the patient support a nursing diagnosis of Spiritual Distress related to loss of family members? (Select all that apply.) 1. "I need to call my sister for support." 2. "I have nothing to live for now." 3. "Why would my God do this to me?" 4. "I need to pray for a miracle." 5. "I want to be more involved in my church."

ANS: 2,3

Which nursing interventions should a nurse implement when removing an indwelling urinary catheter in an adult patient? (Select all that apply.) 1. Attach a 3-mL syringe to the inflation port. 2. Allow the balloon to drain into the syringe by gravity. 3. Initiate a voiding record/bladder diary. 4. Pull the catheter quickly. 5. Clamp the catheter before removal.

ANS: 2,3

Which of the following nursing activities apply to an MDRPI? (Select all that apply.) 1. Assess skin under devices every 2 hours. 2. Cushion at risk areas (e.g., ears, nose with foam or protective dressing). 3. Choose correct size of device. 4. Observe for erythema or irritation that conforms to pattern or shape of device. 5. Observe under casts and splints.

ANS: 2,3,4,5

A nurse is preparing to teach an older adult who has chronic arthritis how to practice meditation. Which of the following teaching strategies are appropriate? (Select all that apply.) 1. Encourage family members to participate in the exercise. 2. Have the patient identify a quiet room in the home that has minimal interruptions. 3. Suggest the use of a quiet fan running in the room. 4. Explain that it is best to meditate about 5 minutes 4 times a day. 5. Show the patient how to sit comfortably with the limitation of his arthritis and focus on a prayer.

ANS: 2,3,5

Which of the following is an indication for a binder to be placed around a surgical patient with a new abdominal wound? (Select all that apply.) 1. Collection of wound drainage 2. Provision of support to abdominal tissues when coughing or walking 3. Reduction of abdominal swelling 4. Reduction of stress on the abdominal incision 5. Stimulation of peristalsis (return of bowel function) from direct pressure

ANS: 2,4

A patient has an order for application of compression stockings. Place the following steps for application of the compression stockings in the correct order: 1. Place patient's toes into foot of stocking up to the heel; keep smooth. 2. Use tape measure to measure patient's leg for proper stocking size. 3. Slide stocking up over patient's calf until sock is completely extended. 4. Turn elastic stocking inside out, keeping hand inside holding heel. Take other hand and pull stocking inside out until reaching the heel. 5. Slide remaining portion of stocking over patient's foot, covering toes. Be sure foot fits into toe and heel of stocking.

ANS: 2,4,1,5,3

Which nursing actions does the nurse take when placing a bedpan under a patient who is immobilized? (Select all that apply.) 1. Lift the patient's hips off the bed and slide the bedpan under the patient. 2. After positioning the patient on the bedpan, elevate the head of the bed to a 45-degree angle. 3. Adjust the head of the bed so that it is lower than the feet and use gentle but firm pressure to push the bedpan under the patient. 4. Have the patient stand beside the bed, and then have the patient sit on the bedpan on the edge of the bed. 5. Make sure the patient has a nurse call system in reach to notify the nurse when ready to have the bedpan removed.

ANS: 2,5

A nurse is caring for a patient who is Muslim and has diabetes mellitus. Which of the following items does the nurse need to remove from the meal tray when it is delivered to the patient? 1. Small container of vanilla ice cream 2. A dozen red grapes 3. Bacon and eggs 4. Garden salad with ranch dressing

ANS: 3

A nurse is teaching a patient to obtain a specimen for fecal occult blood testing using fecal immunochemical testing (FIT) at home. How does the nurse instruct the patient to collect the specimen? 1. Get three fecal smears from one bowel movement. 2. Obtain one fecal smear from an early-morning bowel movement. 3. Collect one fecal smear from three separate bowel movements. 4. Get three fecal smears when you see blood in your bowel movement.

ANS: 3

A nurse used spiritual rituals as an intervention in a patient's care. Which of the following questions is most appropriate to evaluate the efficacy of the intervention? 1. Do you feel the need to forgive your wife over your loss? 2. What can I do to help you feel more at peace? 3. Did either prayer or meditation prove helpful to you? 4. Should we plan on having your family try to visit you more often in the hospital?

ANS: 3

A nurse working in a large occupational health clinic knows that many of the workers at her company are marginalized and at risk for poor health outcomes. Which of the following individuals are most likely to be marginalized? 1. Wives of the employees 2. The head supervisors of the company 3. Workers who have a high school education 4. Workers employed for less than a year at the company

ANS: 3

A patient comes to an urgent care clinic with reports of pain in the right lower calf and ankle after participating in a 5K run. Which of the following assessment questions will determine the effects exercise has had on this patient? 1. Tell me specifically when your pain began. 2. Describe for me the pain you are having. 3. In what way has your daily activity changed since you noticed your pain? 4. How long have you been having the pain?

ANS: 3

A patient is receiving an enteral feeding at 65 mL/h. The GRV in 4 hours was 125 mL. What is the priority nursing intervention? 1. Assess bowel sounds. 2. Raise the head of the bed to at least 45 degrees. 3. Continue the feedings; this is normal gastric residual for this feeding. 4. Hold the feeding until you talk to the primary care provider.

ANS: 3

What is a critical step when inserting an indwelling catheter into a male patient? 1. Slowly inflate the catheter balloon with sterile saline. 2. Secure the catheter drainage tubing to the bedsheets. 3. Advance the catheter to the bifurcation of the drainage and balloon ports. 4. Advance the catheter until urine flows, then insert ¼ inch more.

ANS: 3

Which instruction should the nurse give the assistive personnel (AP) concerning a patient who has had an indwelling urinary catheter removed that day? 1. Limit oral fluid intake to avoid possible UI. 2. Expect patient complaints of suprapubic fullness and discomfort. 3. Report the time and amount of first voiding. 4. Instruct patient to stay in bed and use a urinal or bedpan.

ANS: 3

The nurse is preparing to perform a blood glucose monitoring test on a patient. Place the steps for performing the procedure in the correct sequence. 1. Press button on meter to confirm match codes. 2. Bringing meter to test strip, allow blood drop to wick onto test strip. 3. Instruct patient to perform hand hygiene with soap and water. 4. Clean patient finger with antiseptic swab. 5. Interpret results and document. 6. Check code on test strip vial. 7. Holding lancet to finger, press release button on machine. 8. Perform hand hygiene and put on clean gloves.

ANS: 3, 6, 1, 8, 4, 7, 2, 5

An older-adult patient is admitted following a hip fracture and surgical repair. Before ambulating the patient postoperatively on the evening of surgery, which of the following would be most important to assess? (Select all that apply.) 1. Patient's usual exercise pattern at home 2. Time and date of the patient's last bowel movement 3. Preadmission activity tolerance 4. Baseline heart rate and blood pressure 5. Patient's home living situation

ANS: 3,4

A nursing student is developing a plan of care for a 74-year-old-female patient who has spiritual distress over losing a spouse. As the nurse develops appropriate interventions, which characteristics of older adults should be considered? (Select all that apply.) 1. Older adults do not routinely use complementary medicine to cope with illness. 2. Older adults dislike discussing the afterlife and what might have happened to people who have passed on. 3. Older adults achieve spiritual resilience through frequent expressions of gratitude. 4. Have the patient determine whether her husband left a legacy behind. 5. Offer the patient her choice of rituals or participation in exercise.

ANS: 3,4,5

A nurse is instructing a patient who has decreased leg strength on the left side on how to use a cane. Which actions indicate proper cane use by the patient? (Select all that apply.) 1. The patient keeps the cane on the left side of the body. 2. The patient slightly leans to one side while walking. 3. The patient keeps two points of support on the floor at all times. 4. After the patient places the cane forward, the patient then moves the right leg forward to the cane. 5. The patient places the cane forward 15 to 25 cm (6 to 10 inches) with each step.

ANS: 3,5

Which of the following indicates that additional assistance is needed to transfer a patient from the bed to the stretcher? (Select all that apply.) 1. The patient is 167.6 cm (5 feet 6 inches) and weighs 54.5 kg (120 lb). 2. The patient speaks and understands English. 3. The patient is returning to the unit from the recovery room after a procedure requiring conscious sedation. 4. The patient has a history of being able to stand independently. 5. The patient received analgesia for pain 30 minutes ago.

ANS: 3,5

Place the following steps in the correct order to show how to transfer a patient with partial weight bearing and sufficient upper body strength to a chair. 1. Spread your feet apart. Flex hips and knees, aligning knees with patient's knees. 2. Instruct patient to use armrests on chair for support and ease into chair. 3. Apply gait belt. 4. Flex hips and knees while lowering patient into chair. 5. Pivot on foot farthest from chair. 6. Hold the gait belt with both hands and fingers pointing up. 7. Maintain stability of patient's weaker leg with your knee if needed. 8. Rock patient up to standing position on count of three while straightening hips and legs.

ANS: 3,6,1,8,7,5,2,4

1. When repositioning an immobile patient, the nurse notices redness over the hip bone. What is indicated when a reddened area blanches on fingertip touch? 1. A local skin infection requiring antibiotics 2. Sensitive skin that requires special bed linen 3. A Stage 3 pressure injury needing the appropriate dressing 4. Blanching hyperemia, indicating the attempt by the body to overcome the ischemic episode

ANS: 4

A mother is concerned about her child's flulike symptoms. You learn from the health assessment that the mother practices the use of "hot" and "cold" foods to treat ailments. Which of the following foods do you expect the mother to use to treat her child? 1. Chicken 2. Yogurt 3. Fresh fruits 4. Eggs

ANS: 4

A nurse is caring for a patient who was in an auto accident and has entered rehab after a 6-day hospitalization. The patient had multiple internal injuries and has nursing diagnoses of Hopelessness and Impaired Mobility at time of discharge. The nurse's assessment revealed the patient asking nurses to let him stay in bed and the patient having limited involvement in hygiene and a loss of appetite. The patient has a cast on his nondominant left hand and has reduced movement in the right lower leg, which is splinted. The health care provider has ordered the patient to ambulate 3 times a day. Which of the following is a priority for the rehab nurse? 1. Providing assistance with meals 2. Teaching patient exercises to strengthen right leg 3. Making preferred hygiene products available to the patient to use 4. Setting times to discuss relationship of hopelessness to injuries

ANS: 4

A patient is receiving both PN and EN. When would the nurse collaborate with the health care provider and request a discontinuation of PN? 1. When 25% of the patient's nutritional needs are met by the tube feedings 2. When bowel sounds return 3. When the central line has been in for 10 days 4. When 75% of the patient's nutritional needs are met by the tube feedings

ANS: 4

The nurse is caring for a patient with dysphagia and is feeding her a pureed chicken diet. Suddenly the patient begins to choke. What is the priority nursing intervention? 1. Suction her mouth and throat. 2. Turn her on her side. 3. Put on oxygen at 2 L nasal cannula. 4. Stop feeding her.

ANS: 4

The nurse is planning spiritual care interventions for an 8-year-old patient. Based on the patient's age, the nurse must consider which growth and development factor when planning the spiritual care? 1. This age-group's concept of spirituality is provided by parents, and love and affection promote spirituality. 2. This age-group begins to ask about God or a Supreme Being and is influenced by spiritual and religious stories. 3. This age-group is fascinated with magic and mystery and often believes illness is related to bad behavior. 4. This age-group has a clear concept of a higher spiritual being and is interested in learning about spirituality.

ANS: 4

The patient states, "I have diarrhea and cramping every time I have ice cream. I am sure this is because the food is cold." Based on this assessment data, which health problem does the nurse suspect? 1. A food allergy 2. Irritable bowel syndrome 3. Increased peristalsis 4. Lactose intolerance

ANS: 4

Which statement made by the parents of a 2-month-old infant requires further education by the nurse? 1. "I'll continue to use formula for the baby until he is at least a year old." 2. "I'll make sure that I purchase iron-fortified formula." 3. "I'll start feeding the baby cereal at 4 months." 4. "I'm going to alternate formula with whole milk, starting next month."

ANS: 4

Place the steps when performing wound irrigation of a large open wound in the correct sequence. 1. Use slow, continuous pressure to irrigate wound. 2. Attach 19-gauge angiocatheter to syringe. 3. Fill syringe with irrigation fluid. 4. Place biohazard bag near bed. 5. Position angiocatheter over wound.

ANS: 4,3,2,5,1

Place the following steps for insertion of an indwelling catheter in a female patient in appropriate order. 1. Insert and advance catheter. 2. Lubricate catheter. 3. Inflate catheter balloon. 4. Cleanse urethral meatus with antiseptic solution. 5. Drape patient with the sterile square and fenestrated drapes. 6. When urine appears, advance another 2.5 to 5 cm. 7. Prepare sterile field and supplies. 8. Gently pull catheter until resistance is felt. 9. Attach drainage tubing.

ANS: 5,7,2,4,1,6,3,8,9

Place the steps for an ileostomy pouch change in the correct order. 1. Close the end of the pouch. 2. Measure the stoma. 3. Cut the hole in the wafer to fit around the stoma and not leave skin exposed to the effluent. 4. Press the pouch in place over the stoma. 5. Remove the old pouch. 6. Trace the correct measurement onto the back of the wafer. 7. Assess the stoma and the skin around it. 8. Cleanse and dry the peristomal skin.

ANS: 5,8,7,2,6,3,4,1

Place the following steps in the correct order for repositioning a patient in the 30-degree lateral side-lying position using two nurses. 1. Using knee and hip for leverage, nurse rolls patient onto side. 2. Nurse facing patient's back places hands under patient's dependent shoulder and brings shoulder blade forward. 3. Place pillows under semiflexed upper leg level at hip from groin to foot. 4. Flex patient's knee that will not be next to mattress, after being turned. Keep foot on mattress. Nurse places one hand on patient's upper bent leg. 5. Place hands under dependent hip and bring hip slightly forward so angle from hip to mattress is approximately 30 degrees. 6. Position patient on side of bed in opposite direction toward which patient is to be turned, then move upper and lower trunk. 7. Lower head of bed completely or as low as patient can tolerate. One nurse on each side of bed.

ANS: 7,6,4,1,2,5,3

A 55-year-old patient is preparing to start an exercise program. The health care provider wants 60% of maximum target heart rate. Calculate the heart rate that the nurse will add to the care plan as the target heart rate. Record answer as a whole number. _____ maximum heart rate

ANS: 99 Teach patients to calculate their maximum heart rate by subtracting their current age in years from 220 and then obtaining their target heart rate by taking 60% to 90% of the maximum, depending on their health care provider's recommendation. 220 55 = 165 0.6 = 99.

A nurse is caring for a patient with limited English-speaking skills. What intervention should be implemented to best assist in educating the patient about their disease process? a. Request a trained medical interpreter. b. Provide information in graphic form when possible. c. Use handouts prepared in the patient's native language. d. Ask that a family member be present during educational teaching.

ANS: A The National Culturally and Linguistically Appropriate Services (CLAS) Standards include standards for communication and language assistance. The standards apply when you are caring for patients who have limited English proficiency and/or other communication needs. All United States health care organizations must provide language assistance resources (e.g., trained medical interpreters, qualified translators, telecommunication devices for the deaf) for individuals who have limited English proficiency and/or other communication needs, at no cost to them, to facilitate timely access to all health care and services. None of the other options provides the best form of communication since they all are subject to misinterpretation and nursing evaluation.

A nurse is providing discharge teaching for a patient who is going home with a guaiac test. Which statement by the patient indicates the need for further education? a. ―If I get a blue color that means the test is negative.‖ b. ―I should not get any urine on the stool I am testing.‖ c. ―If I eat red meat before my test, it could give me false results.‖ d. ―I should check with my doctor to stop taking aspirin before the test.‖

ANS: A A blue color indicates a positive guaiac, or presence of fecal occult blood; the patient needs more teaching to correct this misconception. Proper patient education is important for viable results. Be sure specimen is free of toilet paper and not contaminated with urine. The patient needs to avoid certain foods, like red meat, to rule out a false positive. While the health care provider should be consulted before asking a patient to stop any medication, if there are no contraindications, the patient should be instructed to stop taking aspirin, ibuprofen, naproxen, or other nonsteroidal anti-inflammatory drugs for 7 days because these could cause a false-positive test result.

A nurse is caring for a hospitalized patient with a urinary catheter. Which nursing action best prevents the patient from acquiring an infection? a. Maintaining a closed urinary drainage system b. Inserting the catheter using strict clean technique c. Disconnecting and replacing the catheter drainage bag once per shift d. Fully inflating the catheter's balloon according to the manufacturer's recommendation

ANS: A A key intervention to prevent infection is maintaining a closed urinary drainage system. A catheter should be inserted in the hospital setting using sterile technique. Inflating the balloon fully prevents dislodgment and trauma, not infection. Disconnecting the drainage bag from the catheter creates a break in the system and an open portal of entry and increases risk of infection

A nurse is supervising the logrolling of a patient. To which patient is the nurse most likely providing care? a. A patient with neck surgery b. A patient with hypostatic pneumonia c. A patient with a total knee replacement d. A patient with a stage IV pressure ulcer

ANS: A A nurse supervises and aids personnel when there is a health care provider's order to logroll a patient. Patients who have suffered from spinal cord injury or are recovering from neck, back, or spinal surgery often need to keep the spinal column in straight alignment to prevent further injury. Hypostatic pneumonia, total knee replacement, and stage IV ulcers do not have to be logrolled.

A nurse is providing care to a group of patients. Which patient will the nurse see first? a. A patient with a hip replacement on prolonged bed rest reporting chest pain and dyspnea b. A bedridden patient who has a reddened area on the buttocks who needs to be turned c. A patient on bed rest who has renal calculi and needs to go to the bathroom d. A patient after knee surgery who needs range of motion exercises

ANS: A A patient on prolonged bed rest will be prone to deep vein thrombosis, which can lead to an embolus. An embolus can travel through the circulatory system to the lungs and impair circulation and oxygenation, resulting in tachycardia and shortness of breath. Venous emboli that travel to the lungs are sometimes life threatening. While the patient with a reddened area needs to be turned, a patient with renal calculi needing the restroom, and a patient needing range of motion, these are not as life threatening as the chest pain and dyspnea

The nurse is caring for a group of patients. Which patient will the nurse see first? a. A patient saying that God has left and there is no reason for living b. A patient refusing treatment on the Sabbath c. A patient having a folk healer in the room d. A patient praying to Allah

ANS: A A patient saying that God has left and there is no reason for living must be seen first for safety reasons. It must be determined by the nurse if the patient is planning suicide or is just angry and frustrated. A patient refusing treatment on the Sabbath is within that patient's right and doesn't need to be seen first. A patient with a folk healer is within the patient's right and does not need to be seen first. A patient praying to Allah is within the patient's right and does not need to be seen first.

A patient presents to the emergency department with a laceration of the right forearm caused by a fall. After determining that the patient is stable, what is the next best step for the nurse to take? a. Inspect the wound for bleeding. b. Irrigate the wound to remove foreign bodies. c. Measure and document the size of the wound. d. Determine when the patient last had a tetanus antitoxin injection.

ANS: A After determining that a patient's condition is stable, inspect the wound for bleeding. An abrasion will have limited bleeding, a laceration can bleed more profusely, and a puncture wound bleeds in relation to the size and depth of the wound. Address any bleeding issues. Inspect the wound for foreign bodies; traumatic wounds are dirty and may need to be addressed. Determine the size of the wound. A large open wound may expose bone or tissue and be protected, or the wound may need suturing. When the wound is caused by a dirty penetrating object, determine the need for a tetanus vaccination.

The nurse is caring for a patient who has been diagnosed with a terminal illness. The patient states, ―I just don't feel like going to work. I have no energy, and I can't eat or sleep.‖ The patient shows no interest in taking part in the care by saying, ―What's the use?‖ Which response by the nurse is best? a. It sounds like you have lost hope. b. It sounds like you have lost energy. c. It sounds like you have lost your appetite. d. It sounds like you have lost the ability to sleep.

ANS: A All of the patient's description are describing a loss of hope. While losses of energy, appetite, and sleep are indicated, they only address a part of patient's problems. A loss of hope encompasses the holistic view of the patient.

Which action indicates the nurse is meeting a primary goal of culturally competent care for marginalized patients? a. Provides care to transgender patients. b. Provides care to restore relationships. c. Provides care to patients that is individualized. d. Provides care to surgical patients.

ANS: A Although cultural competence and patient-centered care both aim to improve health care quality, their focus is slightly different. The primary aim of cultural competence care is to reduce health disparities and increase health equity and fairness by concentrating on people of color and other marginalized groups, like transgender patients. Patient-centered care, rather than cultural competence care, provides individualized care and restores an emphasis on personal relationships; it aims to elevate quality for all patients

The patient has been diagnosed with a spinal cord injury and needs to be repositioned using the logrolling technique. Which technique will the nurse use for logrolling? a. Use at least three people. b. Have the patient reach for the opposite side rail when turning. c. Move the top part of the patient's torso and then the bottom part. d. Do not use pillows after turning.

ANS: A At least three to four people are needed to perform this skill safely. Have the patient cross the arms on the chest to prevent injury to the arms. Move the patient as one unit in a smooth, continuous motion on the count of three. Gently lean the patient as a unit back toward pillows for support.

A small-bore feeding tube is placed. Which technique will the nurse use to best verify tube placement? a. X-ray b. pH testing c. Auscultation d. Aspiration of contents

ANS: A At present, the most reliable method for verification of placement of small-bore feeding tubes is x-ray examination. Aspiration of contents and pH testing are not infallible. The nurse would need a more precise indicator to help differentiate the source of tube feeding aspirate. Auscultation is no longer considered a reliable method for verification of tube placement because a tube inadvertently placed in the lungs, pharynx, or esophagus transmits sound similar to that of air entering the stomach.

A patient requests the nurse's help to the bedside commode and becomes frustrated when unable to void in front of the nurse. How should the nurse interpret the patient's inability to void? a. The patient can be anxious, making it difficult for abdominal and perineal muscles to relax enough to void. b. The patient does not recognize the physiological signals that indicate a need to void. c. The patient is lonely and calling the nurse in under false pretenses is a way to get attention. d. The patient is not drinking enough fluids to produce adequate urine output.

ANS: A Attempting to void in the presence of another can cause anxiety and tension in the muscles that make voiding difficult. Anxiety can impact bladder emptying due to inadequate relaxation of the pelvic floor muscles and urinary sphincter. The nurse should give the patient privacy and adequate time if appropriate. No evidence suggests that an underlying physiological (does not recognize signals or not drinking enough fluids) or psychological (lonely) condition exists.

A nurse is providing care to a patient with an indwelling catheter. Which practice indicates the nurse is following guidelines for avoiding catheter-associated urinary tract infection (CAUTI)? a. Drapes the urinary drainage tubing with no dependent loops. b. Washes the drainage tube toward the meatus with soap and water. c. Places the urinary drainage bag gently on the floor below the patient. d. Allows the spigot to touch the receptacle when emptying the drainage bag.

ANS: A Avoid dependent loops in urinary drainage tubing. Prevent the urinary drainage bag from touching or dragging on the floor. When emptying the urinary drainage bag, use a separate measuring receptacle for each patient. Do not let the drainage spigot touch the receptacle. Using a clean washcloth, soap, and water, with your dominant hand wipe in a circular motion along the length of the catheter for about 10 cm (4 inches), starting at the meatus and moving away.

The nurse is caring for a patient who has a wound drain with a collection device. The nurse notices that the collection device has a sudden decrease in drainage. Which action will the nurse take initially? a. Call the health care provider; a blockage is present in the tubing. b. Chart the results on the intake and output flow sheet. c. Do nothing, as long as the evacuator is compressed. d. Remove the drain; a drain is no longer needed.

ANS: A Because a drainage system needs to be patent, look for drainage flow through the tubing, as well as around the tubing. A sudden decrease in drainage through the tubing may indicate a blocked drain, and you will need to notify the health care provider. The health care provider, not the nurse, determines the need for drain removal and removes drains. Charting the results on the intake and output flow sheet does not take care of the problem. The evacuator may be compressed even when a blockage is present.

A nurse is providing spiritual care to patients. Which action is essential for the nurse to take? a. Knowing one's own personal beliefs b. Learning about other religions c. Visit churches, temples, mosques, or synagogues d. Travel to other areas that do not have the same beliefs

ANS: A Because each person has a unique spirituality, you need to know your own beliefs, so you are able to care for each patient without bias. While learning about religions, visiting other religious areas of worship, and traveling

The nurse is caring for a patient on the medical-surgical unit with a wound that has a drain and a dressing that needs changing. Which action should the nurse take first? a. Provide analgesic medications as ordered. b. Avoid accidentally removing the drain. c. Don sterile gloves. d. Gather supplies.

ANS: A Because removal of dressings is painful, if often helps to give an analgesic at least 30 minutes before exposing a wound and changing the dressing. The next sequence of events includes gathering supplies for the dressing change, donning gloves, and avoiding the accidental removal of the drain during the procedure.

The nurse is providing teaching to an immobilized patient with impaired skin integrity about diet. Which diet will the nurse recommend? a. High protein, high calorie b. High carbohydrate, low fat c. High vitamin A, high vitamin E d. Fluid restricted, bland

ANS: A Because the body needs protein to repair injured tissue and rebuild depleted protein stores, give the immobilized patient a high-protein, high-calorie diet. A high-carbohydrate, low-fat diet is not beneficial for an immobilized patient. Vitamins B and C are needed rather than A and E. Fluid restriction can be detrimental to the immobilized patient; this can lead to dehydration. A bland diet is not necessary for immobilized patients.

A nurse is beginning to use patient-centered care and cultural competence to improve nursing care. Which step should the nurse take first in achieving cultural awareness? a. Assessing own biases and attitude b. Learning about the world view of others c. Understanding organizational forces d. Developing cultural skills

ANS: A Becoming more aware of your biases and attitudes about human behavior is the first step in providing patient-centered care, leading to culturally competent care. It is helpful to think about cultural competence as a lifelong process of learning about others and also about yourself. Learning about the world view, developing cultural skills, and understanding organizational forces are not the first steps.

A patient has damage to the cerebellum. Which disorder is most important for the nurse to assess? a. Impaired balance b. Hemiplegia c. Muscle sprain d. Lower extremity paralysis

ANS: A Damage to the cerebellum causes problems with balance, and motor impairment is directly related to the amount of destruction of the motor strip. A stroke can lead to hemiplegia. Direct trauma to the musculoskeletal system results in bruises, contusions, sprains, and fractures. A complete transection of the spinal cord can lead to lower extremity paralysis.

The patient is being admitted to the neurological unit with a diagnosis of stroke. When will the nurse begin discharge planning? a. At the time of admission b. The day before the patient is to be discharged c. When outpatient therapy will no longer be needed d. As soon as the patient's discharge destination is known

ANS: A Discharge planning begins when a patient enters the health care system. In anticipation of the patient's discharge from an institution, the nurse makes appropriate referrals or consults a case manager or a discharge planner to ensure that the patient's needs are met at home. Referrals to home care or outpatient therapy are often needed. Planning the day before discharge, when outpatient therapy is no longer needed, and as soon as the discharge destination is known is too late.

A nurse is teaching a health promotion class about isotonic exercises. Which types of exercises will the nurse give as examples? a. Swimming, jogging, and bicycling b. Tightening or tensing of muscles without moving body parts c. Quadriceps set exercises and contraction of the gluteal muscles d. Push-ups, hip lifting, pushing feet against a footboard on the bed

ANS: A Examples of isotonic exercises are walking, swimming, dance aerobics, jogging, bicycling, and moving arms and legs with light resistance. Isometric exercises involve tightening or tensing of muscles without moving body parts. Examples include quadriceps set exercises and contraction of the gluteal muscles. Examples of resistive isometric exercises are push-ups and hip lifting, as well as placing a footboard on the foot of the bed for patients to push against with their feet.

A patient reports severe flank pain. The urinalysis reveals presence of calcium phosphate crystals. The nurse will anticipate an order for which diagnostic test? a. Intravenous pyelogram b. Mid-stream urinalysis c. Bladder scan d. Cystoscopy

ANS: A Flank pain and calcium phosphate crystals are associated with renal calculi. An intravenous pyelogram allows the provider to observe pathological problems such as obstruction of the ureter. A mid-stream urinalysis is performed for a routine urinalysis or if an infection is suspected, a urinalysis was already performed, a mid-stream would not be obtained again. A cystoscopy is used to detect bladder tumors and obstruction of the bladder outlet and urethra. A bladder scan measures the amount of urine in the bladder.

A nurse is caring for a patient with a continent urinary reservoir. Which action will the nurse teach the patient? a. Catheterizing the pouch b. Preforming Kegel exercises c. Changing the collection pouch d. To avoid using the Valsalva technique

ANS: A In a continent urinary reservoir, the ileocecal valve creates a one-way valve in the pouch through which a catheter is inserted through the stoma to empty the urine from the pouch. Patients must be willing and able to catheterize the pouch 4 to 6 times a day for the rest of their lives. The second type of continent urinary diversion is called an orthotopic neobladder, which uses an ileal pouch to replace the bladder. Anatomically, the pouch is in the same position as the bladder was before removal, allowing a patient to void through the urethra using a Valsalva technique. In a ureterostomy or ileal conduit, the patient has no sensation or control over the continuous flow of urine through the ileal conduit, requiring the effluent (drainage) to be collected in a pouch. Kegel exercises are ineffective for a patient with a continent urinary reservoir.

A nurse delegates a position change to a nursing assistive personnel. The nurse instructs the assistive personnel (AP) to place the patient in the lateral position. Which finding by the nurse indicates a correct outcome? a. Patient is lying on side. b. Patient is lying on back. c. Patient is lying semi-prone. d. Patient is lying on abdomen.

ANS: A In the side-lying (or lateral) position, the patient rests on the side with the major portion of body weight on the dependent hip and shoulder. Patients in the supine position rest on their backs. Sims' position is semi-prone. The patient in the prone position lies face or chest down on the abdomen.

The patient weighs 450 lbs. (204.5 kg) and reports shortness of breath with any exertion. The health care provider has recommended beginning an exercise program. The patient states that she can hardly get out of bed and just cannot do anything around the house. Which nursing diagnosis will the nurse add to the care plan? a. Activity intolerance related to excessive weight b. Impaired physical mobility related to bed rest c. Imbalanced nutrition: less than body requirements d. Impaired gas exchange related to shortness of breath

ANS: A In this case, activity intolerance is related to the patient's excessive weight. The patient is not on bed rest although claims that it is difficult to get out of bed, making this diagnosis inappropriate. Shortness of breath is a symptom, not a cause, of Impaired gas exchange, making this nursing diagnosis ineffective. The patient certainly has an imbalance of nutrition, but it is more than body requirements (obesity).

A nurse is preparing to lavage a patient in the emergency department for an overdose. Which tube should the nurse obtain? a. Ewald b. Dobhoff c. Miller-Abbott d. Sengstaken-Blakemore

ANS: A Lavage is irrigation of the stomach in cases of active bleeding, poisoning, or gastric dilation. The types of tubes include Levin, Ewald, and Salem sump. Sengstaken-Blakemore is used for compression by internal application of pressure by means of inflated balloon to prevent internal esophageal or GI hemorrhage. Dobhoff is used for enteral feeding. Miller-Abbott is used for gastric decompression.

The nurse is describing the My Plate program to a patient. Which statement from the patient indicates successful learning? a. ―I can use this to make healthy lifestyle food choices.‖ b. ―I can use this to count specific calories of food.‖ c. ―I can use this for my baby girl.‖ d. ―I can use this when I am sick.‖

ANS: A MyPlate serves as a basic guide for making food choices for a healthy lifestyle. The MyPlate program was developed by the U.S. Department of Agriculture to replace the MyFoodPyramid program. It helps balance calories but does not provide specific calories of food. These guidelines are for Americans over the age of 2 years. These guidelines are provided for health, not sickness.

The nurse is caring for an older-adult patient who has been diagnosed with a stroke. Which intervention will the nurse add to the care plan? a. Encourage the patient to perform as many self-care activities as possible. b. Provide a complete bed bath to promote patient comfort. c. Coordinate with occupational therapy for gait training. d. Place the patient on bed rest to prevent fatigue.

ANS: A Nurses should encourage the older-adult patient to perform as many self-care activities as possible, thereby maintaining the highest level of mobility. Sometimes nurses inadvertently contribute to a patient's immobility by providing unnecessary help with activities such as bathing and transferring. Placing the patient on bed rest without sufficient ambulation leads to loss of mobility and functional decline, resulting in weakness, fatigue, and increased risk for falls. After a stroke or brain attack, a patient likely receives gait training from a physical therapist; speech rehabilitation from a speech therapist; and help from an occupational therapist for ADLs such as dressing, bathing and toileting, or household chores.

The nurse is completing an assessment of the patient's skin's integrity. Which assessment is the priority? a. Pressure points b. Breath sounds c. Bowel sounds d. Pulse points

ANS: A Observe pressure points such as bony prominences. The nurse continually assesses the skin for signs of ulcer development. Assessment for tissue pressure damage includes visual and tactile inspection of the skin. Assessment of pulses, breath sounds, and bowel sounds is part of a head-to-toe assessment and could influence the function of the body and ultimately skin integrity; however, this assessment is not a specific part or priority of a skin assessment.

A patient is experiencing oliguria. Which action should the nurse perform first? a. Assess for bladder distention. b. Request an order for diuretics. c. Increase the patient's intravenous fluid rate. d. Encourage the patient to drink caffeinated beverages.

ANS: A Oliguria is diminished urinary output in relation to fluid intake. The nurse first should gather all assessment data to determine the potential cause of oliguria. It could be that the patient does not have adequate intake, or it could be that the bladder sphincter is not functioning and the patient is retaining water. Increasing fluids is effective if the patient does not have adequate intake or if dehydration occurs. Caffeine can work as a diuretic but is not helpful if an underlying pathology is present. An order for diuretics can be obtained if the patient was retaining water, but this should not be the first action.

The patient has just started on enteral feedings and is now reporting abdominal cramping. Which action will the nurse take next? a. Slow the rate of tube feeding. b. Instill cold formula to ―numb‖ the stomach. c. Change the tube feeding to a high-fat formula. d. Consult with the health care provider about prokinetic medication.

ANS: A One possible cause of abdominal cramping is a rapid increase in rate or volume. Lowering the rate of delivery may increase tolerance. Another possible cause of abdominal cramping is the use of cold formula. The nurse should warm the formula to room temperature. High-fat formulas are also a cause of abdominal cramping. Consult with the health care provider regarding prokinetic medication for increasing gastric motility for delayed gastric emptying.

The nurse is caring for a patient with a chronic illness who is having conflicts with beliefs. Which health care team member will the nurse ask to see this patient? a. The clergy b. A psychiatrist c. A social worker d. An occupational therapist

ANS: A Other important resources to patients are spiritual advisors and members of the clergy. Spiritual care helps people identify meaning and purpose in life, look beyond the present, and maintain personal relationships, as well as a relationship with a higher being or life force. A psychiatrist is for emotional health. A social worker focuses on social, financial, and community resources. An occupational therapist provides care with vocational issues and functioning within physical limitations.

A nurse participating in a research project associated with pressure ulcers will assess for what predisposing factor that tends to increase the risk for pressure ulcer development? a. Decreased level of consciousness b. Adequate dietary intake c. Shortness of breath d. Muscular pain

ANS: A Patients who are confused or disoriented or who have changing levels of consciousness are unable to protect themselves. The patient may feel the pressure but may not understand what to do to relieve the discomfort or to communicate that he or she is feeling discomfort. Impaired sensory perception, impaired mobility, shear, friction, and moisture are other predisposing factors. Shortness of breath, muscular pain, and an adequate dietary intake are not included among the predisposing factors.

The nurse is caring for a patient who needs to be placed in the prone position. Which action will the nurse take? a. Place pillow under the patient's lower legs. b. Turn head toward one side with large, soft pillow. c. Position legs flat against bed. d. Raise head of bed to 45 degrees.

ANS: A Placing a pillow under the lower leg permits dorsiflexion of the ankles and some knee flexion, which promote relaxation. Head is turned toward one side with a small pillow to reduce flexion or hyperextension of cervical vertebrae. Legs should be supported with pillows to elevate toes and prevent footdrop. Forty-five degrees is the position for Fowler's position; prone is on the stomach.

A patient asks about treatment for stress urinary incontinence. Which is the nurse's best response? a. Perform pelvic floor exercises. b. Avoid voiding frequently. c. Drink cranberry juice. d. Wear an adult diaper.

ANS: A Poor muscle tone leads to an inability to control urine flow. The nurse should recommend pelvic muscle strengthening exercises such as Kegel exercises; this solution best addresses the patient's problem. Evidence has shown that patients with urgency, stress, and mixed urinary incontinence can eventually achieve continence when treated with pelvic floor muscle training. Drinking cranberry juice is a preventative measure for urinary tract infection. The nurse should not encourage the patient to reduce voiding; residual urine in the bladder increases the risk of infection. Wearing an adult diaper could be considered if attempts to correct the root of the problem fail.

A patient has fallen several times in the past week when attempting to get to the bathroom. The patient gets up 3 or 4 times a night to urinate. Which recommendation by the nurse is most appropriate in correcting this urinary problem? a. Limit fluid and caffeine intake before bed. b. Leave the bathroom light on to illuminate a pathway. c. Practice Kegel exercises to strengthen bladder muscles. d. Clear the path to the bathroom of all obstacles before bedtime.

ANS: A Reducing fluids, especially caffeine and alcohol, before bedtime can reduce nocturia. To prevent nocturia, suggest that the patient avoid drinking fluids 2 hours before bedtime. Clearing a path to the bathroom, illuminating the path, or shortening the distance to the bathroom may reduce falls but will not correct the urination problem. Kegel exercises are useful if a patient is experiencing stress incontinence.

The nurse is preparing to move a patient to a wheelchair. Which action indicates the nurse is following recommendations for safe patient handling? a. Mentally reviews the transfer steps before beginning. b. Uses own strength to transfer the patient. c. Focuses solely on body mechanics. d. Bases decisions on intuition.

ANS: A Safe patient handling includes mentally reviewing the transfer steps before beginning the procedure to ensure both the patient's and your safety. Use the patient's strength when lifting, transferring, or moving when possible. Body mechanics alone do not protect the nurse from injury to the musculoskeletal system when moving, lifting, or transferring patients. After completing the assessment, nurses use an algorithm to guide decisions about safe patient handling.

A nurse notices that a patient has a structural curvature of the spine associated with vertebral rotation. Which condition will the nurse most likely find documented in the patient's medical record? a. Scoliosis b. Arthritis c. Osteomalacia d. Osteogenesis

ANS: A Scoliosis is a structural curvature of the spine associated with vertebral rotation. Osteogenesis imperfecta is an inherited disorder that makes bones porous, short, bowed, and deformed. Osteomalacia is an uncommon metabolic disease characterized by inadequate and delayed mineralization, resulting in compact and spongy bone. Arthritis is an inflammatory joint disease characterized by inflammation or destruction of the synovial membrane and articular cartilage and by systemic signs of inflammation

The patient is on parenteral nutrition is lethargic while reporting thirst and headache and has had increased urination. Which problem does the nurse prepare to address? a. Hyperglycemia b. Hypoglycemia c. Hypercapnia d. Hypocapnia

ANS: A Signs and symptoms of hyperglycemia are thirst, headache, lethargy, and increased urination. Hypocapnia is not associated with parenteral nutrition. Hypercapnia increases oxygen consumption and increases CO2 levels. Ventilator-dependent patients are at greatest risk for this. Hypoglycemia is characterized by diaphoresis, shakiness, confusion, and loss of consciousness.

When planning care for an adolescent who plays sports, which modification should the nurse include in the care plan? a. Increasing carbohydrates to 55% to 60% of total intake b. Providing vitamin and mineral supplements c. Decreasing protein intake to 0.75 g/kg/day d. Limiting water before and after exercise

ANS: A Sports and regular moderate to intense exercise necessitate dietary modification to meet increased energy needs for adolescents. Carbohydrates, both simple and complex, are the main source of energy, providing 55% to 60% of total daily kilocalories. Protein needs increase to 1 to 1.5 g/kg/day. Fat needs do not increase. Adequate hydration is very important. Adolescents need to ingest water before and after exercise to prevent dehydration, especially in hot, humid environments. Vitamin and mineral supplements are not required, but intake of iron-rich foods is required to prevent anemia.

As prescribed, the nurse leaves the pressure ulcer open to air and does not apply a dressing. Which stage of ulcer did the nurse appropriately treat? a. A Stage I b. A Stage II c. A Stage III d. A Stage IV

ANS: A Stage I intact pressure ulcers that resolve slowly without epidermal loss over 7 to 14 days do not require a dressing. A composite film, hydrocolloid, or hydrogel can be utilized on a clean Stage II. A hydrocolloid, hydrogel covered with foam, calcium alginate, and gauze can be utilized with a clean Stage III. Hydrogel covered with foam, calcium alginate, and gauze can be utilized with a clean Stage IV. An unstageable wound covered with eschar should utilize a dressing of adherent film or gauze with an ordered solution of enzymes.

A patient is using laxatives 3 times daily to lose weight. After stopping laxative use, the patient has difficulty with constipation and wonders if laxatives should be taken again. Which information will the nurse share with the patient? a. Long-term laxative use causes the bowel to become less responsive to stimuli, and constipation may occur. b. Laxatives can cause trauma to the intestinal lining and scarring may result, leading to decreased peristalsis. c. Long-term use of emollient laxatives is effective for treatment of chronic constipation and may be useful in certain situations. d. Laxatives cause the body to become malnourished, so when the patient begins eating again, the body absorbs all of the food, and no waste products are produced.

ANS: A Teach patients about the potential harmful effects of overuse of laxatives, such as impaired bowel motility and decreased response to sensory stimulus. Make sure the patient understands that laxatives are not to be used long term for maintenance of bowel function. Increasing fluid and fiber intake can help with this problem. Laxatives do not cause scarring. Even if malnourished, the body will produce waste if any substance is consumed.

A nurse is teaching a nutrition class about the different daily values. When teaching about the referenced daily intakes (RDIs), which information should the nurse include? a. Have values for protein, vitamins, and minerals. b. Are based on percentages of fat, cholesterol, and fiber. c. Have replaced recommended daily allowances (RDAs). d. Are used to develop diets for chronic illnesses requiring 1800 cal/day.

ANS: A The RDIs are the first set, comprising protein, vitamins, and minerals based on the RDA. The daily reference values (DRVs) make up the second set and consist of nutrients such as total fat, saturated fat, cholesterol, carbohydrates, fiber, sodium, and potassium. Combined, both sets make up the daily values used on food labels. Daily values did not replace RDAs but provided a separate, more understandable format for the public. Daily values are based on percentages of a diet consisting of 2000 kcal/day for adults and children 4 years or older.

A patient is diagnosed with a bowel obstruction. Which type of tube is the best for the nurse to obtain for gastric decompression? a. Salem sump b. Small bore c. Levin d. 8 Fr

ANS: A The Salem sump tube is preferable for stomach decompression. The Salem sump tube has two lumina: one for removal of gastric contents and one to provide an air vent. When the main lumen of the sump tube is connected to suction, the air vent permits free, continuous drainage of secretions. While the Levin tube can be used for decompression, it is only a single-lumen tube with holes near the tip. Large-bore tubes, 12 Fr and above, are usually used for gastric decompression or removal of gastric secretions. Fine- or small-bore tubes are frequently used for medication administration and enteral feedings.

The nurse is starting an exercise program in a local community as a health promotion project. Which information will the nurse include in the teaching session? a. A cool-down period lasts about 5 to 10 minutes. b. The purpose of weight training is to bulk up muscles. c. Resistance training is appropriate for warm-up and cool-down periods. d. Aerobic exercise should be done 3 to 5 times per week for about 20 minutes.

ANS: A The cool-down period follows the exercise routine and usually lasts about 5 to 10 minutes. The purposes of weight training from a health perspective are to develop tone and strength and to simulate and maintain healthy bone. Stretching and flexibility exercises are ideal for warm-up and cool-down periods. The recommended frequency of aerobic exercise is 3 to 5 times per week or every other day for approximately 30 minutes.

A nurse is providing care to a culturally diverse population. Which action indicates the nurse is successful in the role of providing culturally congruent care? a. Provides care that fits the patient's valued life patterns and set of meanings. b. Provides care that is based on meanings generated by predetermined criteria. c. Provides care that makes the nurse the leader in determining what is needed. d. Provides care that is the same as the values of the professional health care system.

ANS: A The goal of transcultural nursing is to provide culturally congruent care, or care that fits the person's life patterns, values, and system of meaning. Patterns and meanings are generated from people themselves, rather than from predetermined criteria. Discovering patients' cultural values, beliefs, and practices as they relate to nursing and health care requires you to assume the role of learner (not become the leader) and to partner with your patients and their families to determine what is needed to provide meaningful and beneficial nursing care. Culturally congruent care is sometimes different from the values and meanings of the professional health care system.

A nurse is teaching a patient about the large intestine in elimination. In which order will the nurse list the structures, starting with the first portion? a. Cecum, ascending, transverse, descending, sigmoid, and rectum b. Ascending, transverse, descending, sigmoid, rectum, and cecum c. Cecum, sigmoid, ascending, transverse, descending, and rectum d. Ascending, transverse, descending, rectum, sigmoid, and cecum

ANS: A The large intestine is divided into the cecum, ascending colon, transverse colon, descending colon, sigmoid colon, and rectum. The large intestine is the primary organ of bowel elimination

A nurse is developing an individualized plan of care for a patient. Which action is important for the nurse to take? a. Establish goals that are measurable and realistic. b. Set goals that are a little beyond the capabilities of the patient. c. Use the nurse's own judgment and not be swayed by family desires. d. Explain that without taking alignment risks, there can be no progress.

ANS: A The nurse must develop an individualized plan of care for each nursing diagnosis and must set goals that are individualized, realistic, and measurable. The nurse should set realistic expectations for care and should include the patient and family when possible. The goals focus on preventing problems or risks to body alignment and mobility.

A nurse is working in a facility that follows a comprehensive safe patient-handling program. Which finding will alert the nurse to intervene? a. Mechanical lifts are in a locked closet. b. Algorithms for patient handling are available. c. Ergonomic assessment protocols are being followed. d. A no-lift policy is in place with adherence by all staff.

ANS: A The nurse will follow up when lifts are not kept in convenient locations. Comprehensive safe patient-handling programs include the following elements: an ergonomics assessment protocol for health care environments, patient assessment criteria, algorithms for patient handling and movement, special equipment kept in convenient locations to help transfer patients, back injury resource nurses, an ―after-action review‖ that allows the health care team to apply knowledge about moving patients safely in different settings, and a no-lift policy.

The nurse is devising a plan of care for a patient with the nursing diagnosis of Constipation related to opioid use. Which outcome will the nurse evaluate as successful for the patient to establish normal defecation? a. The patient reports eliminating a soft, formed stool. b. The patient has quit taking opioid pain medication. c. The patient's lower left quadrant is tender to the touch. d. The nurse hears bowel sounds in all four quadrants.

ANS: A The nurse's goal is for the patient to take opioid medication and to have normal bowel elimination. Normal stools are soft and formed. Ceasing pain medication is not a desired outcome for the patient. Tenderness in the left lower quadrant indicates constipation and does not indicate success. Bowel sounds indicate that the bowels are moving; however, they are not an indication of defecation.

Upon entering the room of a patient with a healing Stage III pressure ulcer, the nurse notices an odor and observes a purulent discharge, along with increased redness at the wound site. What action should the nurse give priority to? a. Completing a head-to-toe assessment, including current treatment, vital signs, and laboratory results b. Notifying the health care provider by utilizing Situation, Background, Assessment, and Recommendation (SBAR) c. Consulting the wound care nurse about the change in status and the potential for infection d. Conferring with the charge nurse about the change in status and the potential for infection

ANS: A The patient is showing signs and symptoms associated with infection in the wound. The nurse should complete the assessment: gather all data such as current treatment modalities, medications, vital signs including temperature, and laboratory results such as the most recent complete blood count or white cell count. The nurse can then notify the primary care provider and receive treatment orders for the patient. It is important to notify the charge nurse and consult the wound nurse on the patient's status and on any new orders.

Which nursing observation will indicate the patient is at risk for pressure ulcer formation? a. Fecal incontinence b. Ate two thirds of breakfast c. A raised red rash on the right shin d. Capillary refill is less than 2 seconds

ANS: A The presence and duration of moisture on the skin increase the risk of ulcer formation by making it susceptible to injury. Moisture can originate from wound drainage, excessive perspiration, and fecal or urinary incontinence. Bacteria and enzymes in the stool can enhance the opportunity for skin breakdown because the skin is moistened and softened, causing maceration. Eating a balanced diet is important for nutrition but eating just two thirds of the meal does not indicate that the individual is at risk. A raised red rash on the leg again is a concern and can affect the integrity of the skin, but it is located on the shin, which is not a high-risk area for skin breakdown. Pressure can influence capillary refill, leading to skin breakdown, but this capillary response is within normal limits

A nurse is planning care for a group of patients. Which task will the nurse assign to the nursing assistive personnel (AP)? a. Obtaining a midstream urine specimen b. Interpreting a bladder scan result c. Inserting a straight catheter d. Irrigating a catheter

ANS: A The skill of collecting midstream (clean-voided) urine specimens can be delegated to nursing assistive personnel. The nurse must first determine the timing and frequency of the bladder scan measurement and interprets the measurements obtained. Inserting a straight or an indwelling catheter cannot be delegated. Catheter irrigation or instillation cannot be delegated to nursing assistive personnel.

The nurse is planning care for a group of stable patients receiving enteral nutrition. Which task will the nurse assign to the nursing assistive personnel? a. Measuring capillary blood glucose level b. Measuring nasoenteric tube for insertion c. Measuring pH in gastrointestinal aspirate d. Measuring the patient's risk for aspiration

ANS: A The skill of measuring blood glucose level after skin puncture (capillary puncture) can be delegated to nursing assistive personnel when the patient's condition is stable. The other skills cannot be delegated. A nurse must measure a nasoenteric tube for insertion, pH in gastrointestinal aspirate, and patient's risk for aspiration.

In caring for the patient's spiritual needs, the nurse asks 20 questions to assess the patient's relationship with God and a sense of life purpose and satisfaction. Which method is the nurse using? a. The spiritual well-being scale b. The FICA assessment tool c. Belief tool d. Hope scale

ANS: A The spiritual well-being scale (SWB) has 20 questions that assess a patient's relationship with God and his or her sense of life purpose and life satisfaction. The FICA assessment tool evaluates spirituality and is closely correlated to quality of life. This does not describe belief or hope.

A nurse is assessing body alignment. What is the nurse monitoring? a. The relationship of one body part to another while in different positions b. The coordinated efforts of the musculoskeletal and nervous systems c. The force that occurs in a direction to oppose movement d. The inability to move about freely

ANS: A The terms body alignment and posture are similar and refer to the positioning of the joints, tendons, ligaments, and muscles while standing, sitting, and lying. Body alignment means that the individual's center of gravity is stable. Body mechanics is a term used to describe the coordinated efforts of the musculoskeletal and nervous systems. Friction is a force that occurs in a direction to oppose movement. Immobility is the inability to move about freely.

The patient is an 80-year-old male who is visiting the clinic today for a routine physical examination. The patient's skin turgor is fair, but the patient reports fatigue and weakness. The skin is warm and dry, pulse rate is 116 beats/min, and urinary sodium level is slightly elevated. Which instruction should the nurse provide? a. Drink more water to prevent further dehydration. b. Drink more calorie-dense fluids to increase caloric intake. c. Drink more milk and dairy products to decrease the risk of osteoporosis. d. Drink more grapefruit juice to enhance vitamin C intake and medication absorption.

ANS: A Thirst sensation diminishes, leading to inadequate fluid intake or dehydration; the patient should be encouraged to drink more water/fluids. Symptoms of dehydration in older adults include confusion, weakness, hot dry skin, furrowed tongue, and high urinary sodium. Milk continues to be an important food for older woman and men, who need adequate calcium to protect against osteoporosis; the patient's problem is dehydration, not osteoporosis. Caution older adults to avoid grapefruit and grapefruit juice because these will decrease absorption of many drugs. The patient needs fluids, not calories; drinking calorie-dense fluids is unnecessary.

A nurse observes a patient rising from a chair slowly by pushing on the chair arms. Which type of tension and contraction did the nurse observe? a. Eccentric tension and isotonic contraction b. Eccentric tension and isometric contraction c. Concentric tension and isotonic contraction d. Concentric tension and isometric contraction

ANS: A This movement causes eccentric tension and isotonic contraction. Eccentric tension helps control the speed and direction of movement. For example, when using an overhead trapeze, the patient slowly lowers himself to the bed. The lowering is controlled when the antagonistic muscles lengthen. By pushing on the chair arms and rising eccentric tension and isotonic contraction occurred. In concentric tension, increased muscle contraction causes muscle shortening, resulting in movement such as when a patient uses an overhead trapeze to pull up in bed. Concentric and eccentric muscle actions are necessary for active movement and therefore are referred to as dynamic or isotonic contraction. Isometric contraction (static contraction) causes an increase in muscle tension or muscle work but no shortening or active movement of the muscle (e.g., instructing the patient to tighten and relax a muscle group, as in quadriceps set exercises or pelvic floor exercises).

A nurse is assessing the patient's meaning of illness. Which area of focus by the nurse is priority? a. On the way a patient reacts to disease b. On the malfunctioning of biological processes c. On the malfunctioning of psychological processes d. On the way a patient reacts to family/social interactions

ANS: A To provide culturally congruent care, you need to understand the difference between disease and illness. Illness is the way that individuals and families react to disease, whereas disease is a malfunctioning of biological or psychological processes. The way a patient interacts to family/social interactions is communication processes and family dynamics.

To reduce patient discomfort during a closed intermittent catheter irrigation, what should the nurse do? a. Use room temperature irrigation solution. b. Administer the solution as quickly as possible. c. Allow the solution to sit in the bladder for at least 1 hour. d. Raise the bag of the irrigation solution at least 12 inches above the bladder.

ANS: A To reduce discomfort use room temperature solution. Using cold solutions and instilling solutions too quickly can cause discomfort. During an irrigation, the solution does not sit in the bladder; it is allowed to drain. A container is not raised about the bladder 12 inches when performing a closed intermittent catheter irrigation.

The nurse is caring for a patient at risk for skin impairment. Which initial action should the nurse take to decrease this risk? a. After cleansing thoroughly dry the skin. b. Request a therapeutic bed and mattress. c. Pad the bed with absorbent pads. d. Use products that retain moisture.

ANS: A Use cleansers with nonionic surfactants that are gentle to the skin. After you clean the skin, make sure that it is completely dry. Absorbent pads and garments are controversial and should be considered only when other alternatives have been exhausted. Depending on the needs of the patient, a specialty bed may be needed, but again, this does not provide the initial defense for skin breakdown. Use only products that wick moisture away from the patient's skin.

The nurse is preparing to apply an external catheter. Which action will the nurse take? a. Allow 1 to 2 inches of space between the tip of the penis and the end of the catheter. b. Spiral wrap the penile shaft using adhesive tape to secure the catheter. c. Twist the catheter before applying drainage tubing to the end of the catheter. d. Shave the pubic area before applying the catheter.

ANS: A When applying an external catheter, allow 2.5 to 5 cm (1 to 2 inches) of space between the tip of the penis and the end of the catheter. Spiral wrap the penile shaft with supplied elastic adhesive. The strip should not overlap. The elastic strip should be snug but not tight. NOTE: Never use adhesive tape. Connect drainage tubing to the end of the condom catheter. Be sure the condom is not twisted. Connect the catheter to a large-volume drainage bag or leg. Clip hair at the base of the penile shaft, as necessary. Do not shave the pubic area.

A patient at risk for skin impairment is able to sit up in a chair. How long should the nurse schedule the patient to sit in the chair? a. 2 hours or less at any one time b. For a total of least than 3 hours daily c. No longer than 30 minutes out of every hour d. Until the patient expresses being uncomfortable

ANS: A When patients are able to sit up in a chair, make sure to limit the amount of time to 2 hours or less at any given time. The chair sitting time should be individualized. In the sitting position, pressure on the ischial tuberosities is greater than in a supine position. Utilize foam, gel, or an air cushion to distribute weight. Sitting for longer than 2 hours can increase the chance of ischemia.

A nurse is assessing the body alignment of a standing patient. Which finding will the nurse report as normal? a. When observed laterally, the spinal curves align in a reversed ―S‖ pattern. b. When observed posteriorly, the hips and shoulders form an ―S‖ pattern. c. The arms should be crossed over the chest or in the lap. d. The feet should be close together with toes pointed out.

ANS: A When the patient is observed laterally, the head is erect, and the spinal curves are aligned in a reversed ―S‖ pattern. When observed posteriorly, the shoulders and hips are straight and parallel. The arms hang comfortably at the sides. The feet are slightly apart to achieve a base of support, and the toes are pointed forward.

A nurse is assessing the health care disparities among population groups. Which area is the nurse monitoring? a. Accessibility of health care services b. Outcomes of health conditions c. Prevalence of complications d. Incidence of diseases

ANS: A While health disparities are the differences among populations in the incidence, prevalence, and outcomes of health conditions, diseases and related complications, health care disparities are differences among populations in the availability, accessibility, and quality of health care services (e.g., screening, diagnostic, treatment, management, and rehabilitation) aimed at prevention, treatment, and management of diseases and their complications.

The nurse is caring for a patient with inner ear problems. Which goal is the priority? a. Maintain balance. b. Maintain proprioception. c. Maintain muscle strength. d. Maintain body alignment

ANS: A Within the inner ear are the semicircular canals, three fluid-filled structures that help maintain balance. Proprioception is the awareness of the position of the body and its parts, and proprioceptors are located on nerve endings, not the inner ear. Muscle strength is maintained with activity and exercise. Although body alignment is important, it is not maintained by the inner ear.

The nurse is providing home care for a patient diagnosed with acquired immunodeficiency syndrome (AIDS). Which dietary intervention will the nurse add to the care plan? a. Provide small, frequent nutrient-dense meals for maximizing kilocalories. b. Prepare hot meals because they are more easily tolerated by the patient. c. Avoid salty foods and limit liquids to preserve electrolytes. d. Encourage intake of fatty foods to increase caloric intake.

ANS: A Small, frequent, nutrient-dense meals that limit fatty foods and overly sweet foods are easier to tolerate. Restorative care of malnutrition resulting from AIDS focuses on maximizing kilocalories and nutrients. Patients benefit from eating cold foods and drier or saltier foods with fluid in between.

A nurse is evaluating a patient's spiritual care. Which areas will the nurse include in the evaluation process? (Select all that apply.) a. Review the patient's view of the purpose in life. b. Ask whether the patient's expectations were met. c. Discuss with family and friends the patient's connectedness. d. Review the patient's self-perception regarding spiritual health. e. Impress on the patient that spiritual health is permanent once obtained.

ANS: A, B, C, D In evaluating care include a review of the patient's self-perception regarding spiritual health, the patient's view of his or her purpose in life, discussion with the family and friends about connectedness, and determining whether the patient's expectations were met. Attainment of spiritual health is a lifelong goal; it is not permanent once obtained.

A nurse is teaching a health class about colorectal cancer. Which information should the nurse include in the teaching session? (Select all that apply.) a. A risk factor is smoking. b. A risk factor is high intake of animal fats or red meat. c. A warning sign is rectal bleeding. d. A warning sign is a sense of incomplete evacuation. e. Screening with a colonoscopy is every 5 years, starting at age 50. f. Screening with flexible sigmoidoscopy is every 10 years, starting at age 50.

ANS: A, B, C, D Risk factors for colorectal cancer are a diet high in animal fats or red meat and low intake of fruits and vegetables; smoking and heavy alcohol consumption are also risk factors. Warning signs are change in bowel habits, rectal bleeding, a sensation of incomplete evacuation, and unexplained abdominal or back pain. A flexible sigmoidoscopy is every 5 years, starting at age 50, while a colonoscopy is every 10 years, starting at age 50.

The nurse is caring for a patient with a wound healing by full-thickness repair. Which phases will the nurse monitor for in this patient? (Select all that apply.) a. Hemostasis b. Maturation c. Inflammatory d. Proliferative e. Reproduction f. Reestablishment of epidermal layers

ANS: A, B, C, D The four phases involved in the healing process of a full-thickness wound are hemostasis, inflammatory, proliferative, and maturation. Three components are involved in the healing process of a partial-thickness wound: inflammatory response, epithelial proliferation (reproduction) and migration, and reestablishment of the epidermal layers.

The nurse is completing a skin assessment on a medical-surgical patient. Which nursing assessment questions should be included in a skin integrity assessment? (Select all that apply.) a. ―Can you easily change your position?‖ b. ―Do you have sensitivity to heat or cold?‖ c. ―How often do you need to use the toilet?‖ d. ―What medications do you take?‖ e. ―Is movement painful?‖ f. ―Have you ever fallen?‖

ANS: A, B, C, E Changing positions is important for decreasing the pressure associated with long periods of time in the same position. If the patient is able to feel heat or cold and is mobile, she can protect herself by withdrawing from the source. Knowing toileting habits and any potential for incontinence is important because urine and feces in contact with the skin for long periods can increase skin breakdown. Knowing whether the patient has problems with painful movement will alert the nurse to any potential for decreased movement and increased risk for skin breakdown. Medications and falling are safety risk questions.

A nurse is assessing activity tolerance of a patient. Which areas will the nurse assess? (Select all that apply.) a. Skeletal abnormalities b. Emotional factors c. Pregnancy status d. Race e. Age

ANS: A, B, C, E Physiological, emotional, and developmental factors (age) influence the patient's activity tolerance. Factors influencing activity tolerance include physiological factors such as skeletal abnormalities, emotional factors such as anxiety/depression, developmental factors such as age and gender, and pregnancy status. Race is not a factor because people of all races are faced with similar factors that affect their activity tolerance.

A nurse is using Campinha-Bacote's model of cultural competency. Which areas will the nurse focus on to become competent? (Select all that apply.) a. Cultural skills b. Cultural desire c. Cultural transition d. Cultural knowledge e. Cultural encounters

ANS: A, B, D, E Campinha-Bacote's model of cultural competency has five interrelated components: cultural awareness, cultural knowledge, cultural skills, cultural encounters, and cultural desire. Cultural transition is not a component of this model.

A nurse is preparing a bowel training program for a patient. Which actions will the nurse take? (Select all that apply.) a. Record times when the patient is incontinent. b. Help the patient to the toilet at the designated time. c. Lean backward on the hips while sitting on the toilet. d. Maintain normal exercise within the patient's physical ability. e. Apply pressure with hands over the abdomen, and strain while pushing. f. Choose a time based on the patient's pattern to initiate defecation-control measures.

ANS: A, B, D, F A successful program includes the following: Assessing the normal elimination pattern and recording times when the patient is incontinent. Choosing a time based on the patient's pattern to initiate defecation-control measures. Maintaining normal exercise within the patient's physical ability. Helping the patient to the toilet at the designated time. Offering a hot drink (hot tea) or fruit juice (prune juice) (or whatever fluids normally stimulate peristalsis for the patient) before the defecation time. Instructing the patient to lean forward at the hips while sitting on the toilet, apply manual pressure with the hands over the abdomen, and bear down but do not strain to stimulate colon emptying.

A nurse is teaching a health class about the nutritional requirements throughout the life span. Which information should the nurse include in the teaching session? (Select all that apply.) a. Infants triple weight at 1 year. b. Toddlers become picky eaters. c. School-age children need to avoid hot dogs and grapes. d. Breastfeeding women need an additional 750 kcal/day. e. Older adults have altered food flavor from a decrease in taste cells.

ANS: A, B, E An infant usually doubles birth weight at 4 to 5 months and triples it at 1 year. Toddlers exhibit strong food preferences and become picky eaters. Older adults often experience a decrease in taste cells that alters food flavor and may decrease intake. Toddlers need to avoid hot dogs and grapes, not school-age children. The lactating woman needs 500 kcal/day above the usual allowance because the production of milk increases energy requirements.

A nurse writes the following outcomes for a patient who has chronic obstructive pulmonary disease to improve activity level: Diastolic blood pressure will remain below 70 mm Hg with systolic below 130 mm Hg. Resting heart rate will range between 65 and 75. The last goal is that the patient will exercise 3 times a week. Which evaluative findings indicate successful goal achievement? (Select all that apply.) a. Resting heart rate 70 b. Blood pressure 126/64 c. Blood pressure 140/90 d. Reports doing stretching and flexibility exercises 2 times this week e. Reports doing resistive training 1 time and aerobics 2 times this week

ANS: A, B, E Compare actual outcomes with expected outcomes to determine the patient's health status and progression. Heart rate of 70 is between 65 and 75. Blood pressure 126/64 meets the goal. Did resistive training 1 time and aerobics 2 times equals exercising 3 times a week. Did stretching and flexibility exercises 2 times is below the 3 times a week. Blood pressure 140/90 is too high and does not meet the goal.

The nurse is caring for a patient who has experienced a recent stroke and is paralyzed on the left side. The patient has no respiratory or cardiac issues but cannot walk. The patient cannot button a shirt and cannot feed self-due to being left-handed and becomes frustrated very easily. The patient has been eating very little and has lost 2 lb. The patient asks the nurse, ―How can I go home like this? I'm not getting better.‖ Which health care team members will the nurse need to consult? (Select all that apply.) a. Dietitian b. Physical therapist c. Respiratory therapist d. Cardiac rehabilitation therapist e. Occupational therapist f. Psychologist

ANS: A, B, E, F Physical therapists are a resource for planning ROM or strengthening exercises, and occupational therapists are a resource for planning ADLs that patients need to modify or relearn. Because of the loss of 2 lb and eating very little, a dietitian will also be helpful. Referral to a mental health advanced practice nurse, a licensed social worker, or a physiologist to assist with coping or other psychosocial issues is also wise. Because the patient exhibits good cardiac and respiratory function, respiratory therapy and cardiac rehabilitation probably are not needed at this time.

The nurse is using different toileting schedules. Which principles will the nurse keep in mind when planning care? (Select all that apply.) a. Habit training uses a bladder diary. b. Timed voiding is based upon the patient's urge to void. c. Prompted voiding includes asking patients if they are wet or dry. d. Elevation of feet in patients with edema can decrease nighttime voiding. e. Bladder retraining teaches patients to follow the urge to void as quickly as possible.

ANS: A, C Habit training is a toileting schedule based upon the patient's usual voiding pattern. Using a bladder diary, the usual times a patient voids are identified. It is at these times that the patient is then toileted. Prompted voiding is a program of toileting designed for patients with mild or moderately cognitive impairment. Patients are toileted based upon their usual voiding pattern. Caregivers ask the patient if they are wet or dry, give positive feedback for dryness, prompt the patient to toilet, and reward the patient for desired behavior. Timed voiding or scheduled toileting is toileting based upon a fixed schedule, not the patient's urge to void. The schedule maybe set by a time interval, every 2 to 3 hours or at times of day such as before and after meals. In bladder retraining, patients are taught to inhibit the urge to void by taking slow and deep breaths to relax, perform 5 to 6 quick strong pelvic muscle exercises (flicks) in quick succession followed by distracting attention from bladder sensations. When the urge to void becomes less severe or subsides, only then should the patient start the trip to the bathroom. Encourage patients with edema to elevate the feet for a minimum of a few hours in the afternoon to help diminish nighttime voiding frequency; while this is helpful, it is not a toileting schedule.

A nurse is attempting to establish a respectful relationship with a newly admitted patient from another county. Which actions should the nurse take? (Select all that apply.) a. Engage in face-to-face interactions. b. Help the patient overcome barriers. c. Consciously attempt to suspend judgment. d. Stress that they will be working together to address problems. e. Know limitations in addressing medical issues across cultures.

ANS: A, C Ongoing development of cultural competence will present as your ability to interact effectively with people from different cultures, identifying the need to be respectful and responsive to the health belief practices or linguistic needs of our diverse population. Cultural awareness is the process of conducting a self-examination of one's own biases toward other cultures and the in-depth exploration of one's cultural and professional background. Cultural encounter is a process that encourages health care professionals to directly engage in face-to-face cultural interactions and other types of encounters with patients from culturally diverse backgrounds. The remaining options are not specific to a culturally competent plan of nursing care.

The nurse is caring for a patient with impaired physical mobility. Which potential complications will the nurse monitor for in this patient? (Select all that apply.) a. Foot drop b. Somnolence c. Hypostatic pneumonia d. Impaired skin integrity e. Increased socialization

ANS: A, C, D Immobility leads to complications such as hypostatic pneumonia. Other possible complications include footdrop and impaired skin integrity. Interruptions in the sleep-wake cycle and social isolation are more common complications than somnolence or increased socialization.

When assessing patient with nutritional needs, which patients will require follow-up from the nurse? (Select all that apply.) a. A patient with infection taking tetracycline with milk b. A patient with irritable bowel syndrome increasing fiber c. A patient with diverticulitis following a high-fiber diet daily d. A patient with an enteral feeding and 500 mL of gastric residual e. A patient with dysphagia being referred to a speech-language pathologist

ANS: A, C, D The nurse should follow up with the tetracycline, diverticulitis, and enteral feeding. Tetracycline has decreased drug absorption with milk and antacids and has decreased nutrient absorption of calcium from binding. Nutritional treatment for diverticulitis includes a moderate- or low-residue diet until the infection subsides. Afterward, prescribing a high-fiber diet for chronic diverticula problems ensues. A patient with a gastric residual volume of 500 mL needs to have the feeding withheld and reassessed for tolerance to feedings. All the rest are normal and expected and do not require follow-up. Patients manage irritable bowel syndrome by increasing fiber, reducing fat, avoiding large meals, and avoiding lactose or sorbitol-containing foods for susceptible individuals. Initiate consultation with a speech-language pathologist for swallowing exercises and techniques to improve swallowing and reduce risk of aspiration for a patient with dysphagia.

The patient is asking the nurse about the best way to stay healthy. The nurse explains to the patient which teaching points? (Select all that apply.) a. Increase physical activity. b. Keep total fat intake to 10% or less. c. Maintain body weight in a healthy range. d. Choose and prepare foods with little salt. e. Increase intake of meat and other high-protein foods.

ANS: A, C, D Recommendations include maintaining body weight in a healthy range; increasing physical activity and decreasing sedentary activities; increasing intake of fruits, vegetables, whole grain products, and fat-free or low-fat milk; eating moderate amount of lean meats, poultry, and eggs; keeping fat intake between 20% and 35% of total calories, with most fats coming from polyunsaturated or monounsaturated fatty acids (most meats contain saturated fatty acids); and choosing prepared foods with little salt while at the same time eating potassium-rich foods.

The nurse is caring for a patient who will have a large abdominal bandage secured with an abdominal binder. Which actions will the nurse take before applying the bandage and binder? (Select all that apply.) a. Cover exposed wounds. b. Mark the sites of all abrasions. c. Assess the condition of current dressings. d. Inspect the skin for abrasions and edema. e. Cleanse the area with hydrogen peroxide. f. Assess the skin at underlying areas for circulatory impairment.

ANS: A, C, D, F Before applying a bandage or a binder, the nurse has several responsibilities. The nurse would need to inspect the skin for abrasions, edema, and discoloration or exposed wound edges. The nurse also is responsible for covering exposed wounds or open abrasions with a dressing and assessing the condition of underlying dressings and changing if soiled, as well as assessing the skin of underlying areas that will be distal to the bandage. This checks for signs of circulatory impairment, so that a comparison can be made after bandages are applied. Marking the sites of all abrasions is not necessary. Although it is important for the skin to be clean, and even though it may need to be cleaned with a noncytotoxic cleanser, cleansing with hydrogen peroxide can interfere with wound healing.

1. The nurse is caring for a patient with a surgical incision that eviscerates. Which actions will the nurse take? (Select all that apply.) a. Place moist sterile gauze over the site. b. Gently place the organs back. c. Contact the surgical team. d. Offer a glass of water. e. Monitor for shock.

ANS: A, C, E The presence of an evisceration (protrusion of visceral organs through a wound opening) is a surgical emergency. Immediately place damp sterile gauze over the site, contact the surgical team, do not allow the patient anything by mouth (NPO), observe for signs and symptoms of shock, and prepare the patient for emergency surgery.

A nurse is working in a facility that uses no-lift policies. Which benefits will the nurse observe in the facility? (Select all that apply.) a. Reduced number of work-related injuries b. Increased musculoskeletal accidents c. Reduced safety of patients d. Improved health of nurses e. Increased indirect costs

ANS: A, D Implementing evidence-based interventions and programs (e.g., lift teams) reduces the number of work-related injuries, which improves the health of the nurse and reduces indirect costs to the health care facility (e.g., workers' compensation and replacing injured workers). Knowing the movements and functions of muscles in maintaining posture and movement and implementing evidence-based knowledge about safe patient handling are essential to protecting the safety of both the patient and the nurse.

Upon assessment a nurse discovers that a patient has erythema. Which actions will the nurse take? (Select all that apply.) a. Consult a dietitian. b. Increase fiber in the diet. c. Place on chest physiotherapy. d. Increase frequency of turning. e. Place on pressure-relieving mattress.

ANS: A, D, E If skin shows areas of erythema and breakdown, increase the frequency of turning and repositioning; place the turning schedule above the patient's bed; implement other activities per agency skin care policy or protocol (e.g., assess more frequently, consult dietitian, place patient on pressure-relieving mattress). Increased fiber will help constipation. Chest physiotherapy is for respiratory complications.

Spiritual distress has been identified in a patient who has been diagnosed with a chronic illness. Which interventions will the nurse add to the care plan? (Select all that apply.) a. Being supportive of a patient's wish to pray b. Encouraging time with the support group c. Arranging for a clergy member to visit the patient d. Developing activities to heal body, mind, and spirit e. Teaching relaxation, guided imagery, and meditation

ANS: A, D, E Interventions that are appropriate for spiritual distress include: (1) helping the patient develop/identify activities to heal body, mind, and spirit; (2) supporting the patient's desire to pray; (3) encourage attendance with an appropriate support group; and (4) teaching relaxation, guided imagery, and medication. Clergy consults should be made only with the patient's permission.

The nurse is obtaining a 24-hour urine specimen collection from the patient. Which actions should the nurse take? (Select all that apply.) a. Keeping the urine collection container on ice when indicated b. Withholding all patient medications for the day c. Irrigating the sample as needed with sterile solution d. Testing the urine sample with a reagent strip by dipping it in the urine e. Asking the patient to void and discarding that urine to start the collection

ANS: A, E When obtaining a 24-hour urine specimen, it is important to keep the urine in cool conditions, depending upon the test. The patient should be asked to void and to discard the urine before the procedure begins. Medications do not need to be held unless indicated by the provider. If properly educated about the collection procedure, the patient can maintain autonomy and perform the procedure alone, taking care to maintain the integrity of the solution. A 24-hour urine specimen is not tested with a reagent strip

The nurse is caring for a patient who is immobile and needs to be turned every 2 hours. Which device will the nurse use to help maintain foot function? a. Hand rolls b. A foot boot c. A trapeze bar d. A trochanter roll

ANS: B A foot boot prevents foot drop by maintaining a foot in dorsiflexion. A trochanter roll prevents external rotation of the hips when the patient is in a supine position. This is especially useful in patients who have lost the ability to move the lower extremities. Hand rolls maintain the thumb in slight adduction and in opposition to the fingers. The trapeze bar is a triangular device that hangs down from a securely fastened overhead bar that is attached to the bedframe. It allows the patient to pull with the upper extremities to raise the trunk off the bed, to assist in transfer from bed to wheelchair, or to perform upper arm exercises.

The nurse is concerned about pulmonary aspiration when providing care to the patient with an intermittent tube feeding. Which action is the priority? a. Observe the color of gastric contents. b. Verify tube placement before feeding. c. Add blue food coloring to the enteral formula. d. Run the formula over 12 hours to decrease overload.

ANS: B A major cause of pulmonary aspiration is regurgitation of formula. The nurse needs to first verify tube placement and elevate the head of the bed 30 to 45 degrees during feedings and for 2 hours afterward. While observing the color of gastric contents is a component, it is not the priority component; pH is the primary component. The addition of blue food coloring to enteral formula to assist with detection of aspirate is no longer used. Do not hang formula longer than 4 to 8 hours. Formula becomes a medium for bacterial growth after that length of time.

The nurse is teaching a patient how to sit with crutches. In which order will the nurse present the instructions starting with the first step? 1. Place both crutches in one hand. 2. Grasp arm of chair with free hand. 3. Completely lower self into chair. 4. Transfer weight to crutches and unaffected leg. a. 4, 1, 2, 3 b. 1, 4, 2, 3 c. 1, 2, 4, 3 d. 4, 2, 1, 3

ANS: B A patient is sitting in a chair with crutches. Both crutches are held in one hand. The patient then transfers weight to the crutches and the unaffected leg. Next, the patient grasps the arm of the chair with the free hand and begins to lower self into chair. Finally, the patient completely lowers self into chair.

The nurse is caring for an Islamic patient who wants to pray. Which intervention is most appropriate? a. Postponing their bath until after they are finished praying b. Offering to turn on their music in preparation for prayer c. Ask their roommate to leave while they pray d. Set aside time for their praying 3 times a day

ANS: B A patient who is a follower of Islam will want to say daily prayers (Salah) five times a day. Patients pray in private or participate in group prayer with family, friends, or clergy. Muslims must be clean before they pray, so offer hygiene measures according to Islam practices. Some pray while listening to music.

The nurse anticipates a suprapubic catheter for which patient? a. A patient with recent prostatectomy b. A patient with a urethral stricture c. A patient with an appendectomy d. A patient with menopause

ANS: B A patient with a urethral stricture is most likely to have a suprapubic catheter. Suprapubic catheters are placed when there is blockage of the urethra (e.g., enlarged prostate, urethral stricture, after urological surgery). A patient with a recent prostatectomy indicates the enlarged prostate was removed and would not need a suprapubic catheter; however, continuous bladder irrigation may be needed. Appendectomies and menopause do not require a suprapubic catheter.

The nurse caring for a patient in the burn unit should expect what type of wound healing when planning care for this patient? a. Partial-thickness repair b. Secondary intention c. Tertiary intention d. Primary intention

ANS: B A wound involving loss of tissue such as a burn or a pressure ulcer or laceration heals by secondary intention. The wound is left open until it becomes filled with scar tissue. It takes longer for a wound to heal by secondary intention; thus the chance of infection is greater. A clean surgical incision is an example of a wound with little loss of tissue that heals by primary intention. The skin edges are approximated or closed, and the risk for infection is low. Partial-thickness repair is done on partial-thickness wounds that are shallow, involving loss of the epidermis and maybe partial loss of the dermis. These wounds heal by regeneration because the epidermis regenerates. Tertiary intention is seen when a wound is left open for several days, and then the wound edges are approximated. Wound closure is delayed until the risk of infection is resolved.

A nurse is assessing a patient's ethnic history. Which question should the nurse ask? a. What language do you speak at home? b. How different is your life here from back home? c. Which caregivers do you seek when you are sick? d. How different is what we do from what your family does when you are sick?

ANS: B An ethnohistory question is the following: How different is your life here from back home? Caring beliefs and practice questions include the following: Which caregivers do you seek when you are sick and How different is what we do from what your family does when you are sick? The language and communication is the following: What language do you speak at home?

The patient is having a difficult time dealing with an AIDS diagnosis. The patient states, ―It's not fair. I'm totally isolated from God and my family because of this. Even my father hates me for this. He won't even speak to me.‖ What should the nurse do? a. Tell the patient to move on and focus on getting better. b. Use therapeutic communication to establish trust and caring. c. Assure the patient that the father will accept this situation soon. d. Point out that the patient has no control and that he or she must face the consequences.

ANS: B Application of therapeutic communication principles and caring helps you establish therapeutic trust with patients. The nurse should not offer false hope (father will accept the situation soon). The nurse should help the patient maintain feelings of control, not no control. The nurse should encourage renewing relationships if possible and establishing connections with self, significant others, and God.

The nurse is assessing body alignment for a patient who is immobilized. Which patient position will the nurse use? a. Supine position b. Lateral position c. Lateral position with positioning supports d. Supine position with no pillow under the patient's head

ANS: B Assess body alignment for a patient who is immobilized or bedridden with the patient in the lateral position, not supine. Remove all positioning supports from the bed except for the pillow under the head and support the body with an adequate mattress.

The patient is brought to the emergency department with possible injury to the left shoulder. Which area will the nurse assess to best determine joint mobility? a. The patient's gait b. The patient's range of motion c. The patient's ethnic influences d. The patient's fine-motor coordination

ANS: B Assessing range of motion is one assessment technique used to determine the degree of joint mobility and injury to a joint. Gait is the manner or style of walking. It has little bearing on the shoulder damage. Assessing fine-motor coordination would be beneficial in helping to assess the patient's ability to perform tasks such as feeding and dressing but would not help in evaluating the shoulder. Ethnic influences would not have a direct bearing on the amount of mobility in the joint.

The patient is admitted to a skilled care unit for rehabilitation after the surgical procedure of fixation of a fractured left hip. The patient's nursing diagnosis is Impaired physical mobility related to musculoskeletal impairment from surgery and pain with movement. The patient is able to use a walker but needs assistance ambulating and transferring from the bed to the chair. Which nursing intervention is most appropriate for this patient? a. Obtain assistance and physically transfer the patient to the chair. b. Assist with ambulation and measure how far the patient walks. c. Give pain medication after ambulation so the patient will have a clear mind. d. Bring the patient to the cafeteria for group instruction on ambulation.

ANS: B Assist with walking and measure how far the patient walks to quantify progress. The nurse should allow the patient to do as much for self as possible. Therefore, the nurse should observe the patient transferring from the bed to the chair using the walker and should provide assistance as needed. The patient should be encouraged to use adequate pain medication to decrease the effects of pain and to increase mobility. The patient should be instructed on safe transfer and ambulation techniques in an environment with few distractions, not in the cafeteria.

The nurse is assessing the patient for respiratory complications of immobility. Which action will the nurse take when assessing the respiratory system? a. Inspect chest wall movements primarily during the expiratory cycle. b. Auscultate the entire lung region to assess lung sounds. c. Focus auscultation on the upper lung fields. d. Assess the patient at least every 4 hours.

ANS: B Auscultate the entire lung region to identify diminished breath sounds, crackles, or wheezes. Perform a respiratory assessment at least every 2 hours for patients with restricted activity. Inspect chest wall movements during the full inspiratory-expiratory cycle. Focus auscultation on the dependent lung fields because pulmonary secretions tend to collect in these lower regions.

The nurse is emptying an ileostomy pouch for a patient. Which assessment finding will the nurse report immediately? a. Liquid consistency of stool b. Presence of blood in the stool c. Malodorous stool d. Continuous output from the stoma

ANS: B Blood in the stool indicates a problem, and the health care provider should be notified. All other options are expected findings for an ileostomy. The stool should be liquid, there should be an odor, and the output should be continuous.

The nurse is evaluating the body alignment of a patient in the sitting position. Which observation by the nurse will indicate a normal finding? a. The edge of the seat is in contact with the popliteal space. b. Both feet are supported on the floor with ankles flexed. c. The body weight is directly on the buttocks only. d. The arms hang comfortably at the sides.

ANS: B Both feet are supported on the floor, and the ankles are comfortably flexed. Body weight is evenly distributed on the buttocks and thighs. A 1- to 2-inch space is maintained between the edge of the seat and the popliteal space on the posterior surface of the knee to ensure that no pressure is placed on the popliteal artery or nerve. The patient's forearms are supported on the armrest, in the lap, or on a table in front of the chair.

A nurse is caring for a patient who has had diarrhea for the past week. Which additional assessment finding will the nurse expect? a. Distended abdomen b. Increased skin dryness c. Increased energy levels d. Elevated blood pressure

ANS: B Chronic diarrhea can result in dehydration. Patients with chronic diarrhea are dehydrated with dark colored urine and dry skin. Diarrhea also causes loss of electrolytes, nutrients, and fluid, which decreases energy levels. A distended abdomen could indicate constipation.

A nurse is providing care to a patient from a different culture. Which action by the nurse indicates cultural competence? a. Communicates effectively in a multicultural context. b. Effectively provides for multifaceted healthcare needs. c. Visits a foreign country. d. Speaks a different language.

ANS: B Cultural competence refers to a developmental process that evolves over time that impacts ability to effectively function in the multifaceted context. Communicates effectively and speaking a different language indicates linguistic competence. Visiting a foreign country does not indicate cultural competence.

A nurse is assessing culturally diverse population groups for the risk of suicide. Which assessment question will provide the most culturally relevant information? a. ―Is suicide common in your culture?‖ b. ―How is suicide viewed in your culture?‖ c. ―Has anyone here every considered suicide?‖ d. ―Do you know anyone who as committed suicide?‖

ANS: B Culturally congruent care or transcultural care emphasizes the need to provide cares based on the individual's cultural beliefs, practices, and values; therefore, effective communication is a critical skill in culturally competent care and helps you engage a patient and family in respectful, patient-centered dialogue. Asking how the act of suicide is viewed provides information on the cultural values, beliefs, and practices of a culture. None of the other options provide that insight.

The nurse caring for a patient of Hispanic descent who speaks no English, is working with an interpreter. Which action should the nurse take? a. Use long sentences when talking. b. Look at the patient when talking. c. Use breaks in sentences when talking. d. Look at only nonverbal behaviors when talking.

ANS: B Direct your questions to the patient. Look at the patient, instead of looking at the interpreter. Pace your speech by using short sentences, but do not break your sentences. Observe the patient's nonverbal and verbal behaviors.

The nurse, upon reviewing the history, discovers the patient has dysuria. Which assessment finding is consistent with dysuria? a. Blood in the urine b. Burning upon urination c. Immediate, strong desire to void d. Awakes from sleep due to urge to void

ANS: B Dysuria is burning or pain with urination. Hematuria is blood in the urine. Urgency is an immediate and strong desire to void that is not easily deferred. Nocturia is awakening form sleep due to urge to void.

The nurse is preparing to check the gastric aspirate for pH. Which equipment will the nurse obtain? a. 10-mL Luer-Lok syringe b. ENFit syringe c. Sterile gloves d. Double gloves

ANS: B ENFit syringe is needed for testing of gastric aspirate for pH; these syringes are better than a Luer-Lok syringe. Clean gloves are needed, not sterile or double

A nurse is caring for a patient diagnosed with osteoporosis and lactose intolerance. What intervention will the nurse implement? a. Encourage dairy products. b. Monitor intake of calcium. c. Increase intake of caffeinated drinks. d. Try to do as much as possible for the patient.

ANS: B Encourage patients at risk to be screened for osteoporosis and assess their diets for calcium and vitamin D intake. Patients who have lactose intolerance need dietary teaching about alternative sources of calcium. Caffeine should be decreased. The goal of the patient with osteoporosis is to maintain independence with ADLs. Assistive ambulatory devices, adaptive clothing, and safety bars help the patient maintain independence.

The nurse caring for a terminally ill patient sits down and lightly touches the patient's hand. Which technique is the nurse using? a. ―Doing for‖ b. Establishing presence c. Offering transcendence d. Providing health promotion

ANS: B Establishing presence by sitting with a patient to attentively listen to his or her feelings and situation, talking with the patient, crying with the patient, and simply offering time are powerful spiritual care approaches. Benner explains that presence involves ―being with‖ a patient versus ―doing for‖ a patient. Transcendence is the belief that a force outside of and greater than the person exists beyond the material world. In settings where health promotion activities occur, patients often need information, counseling, and guidance to make the necessary choices to remain healthy.

The patient appears anxious as the nurse is preparing to change their wound dressing. Which action should the nurse take? a. Distract the patient with the television. b. Offer to explain what they should expect. c. Suggest that the patient ―Close your eyes.‖ d. Wait until family is visiting to support the patient

ANS: B Explaining the procedure educates the patient regarding the dressing change and involves him in the care, thereby allowing the patient some control in decreasing anxiety. Telling the patient to close the eyes and turning on the television are distractions that do not usually decrease a patient's anxiety. If the family is a support system, their presence may be helpful but it isn't practical to postpone the treatment until they are present.

Which patient comment should the nurse identify as a demonstration of faith? a. I go to church every Sunday. b. I believe there is life after death. c. I have something to look forward to each day. d. I get a feeling of awe when looking at the sunset.

ANS: B Faith allows people to have firm beliefs despite lack of physical evidence (life after death). Religion refers to the system of organized beliefs and worship that a person practices to outwardly express spirituality (go to church). When a person has the attitude of something to live for and look forward to, hope is present (look forward to each day). Self-transcendence is the belief that there is a force outside of and greater than the person (awe when looking at a sunset)

The nurse is caring for a patient who has a Stage IV pressure ulcer with grafted surgical sites. Which specialty bed will the nurse use for this patient? a. Low-air-loss b. Air-fluidized c. Lateral rotation d. Standard mattress

ANS: B For a patient with newly flapped or grafted surgical sites, the air-fluidized bed will be the best choice; this uses air and fluid support to provide pressure redistribution via a fluid-like medium created by forcing air through beads as characterized by immersion and envelopment. A low-air-loss bed is utilized for prevention or treatment of skin breakdown by preventing buildup of moisture and skin breakdown through the use of airflow. A standard mattress is utilized for an individual who does not have actual or potential altered or impaired skin integrity. Lateral rotation is used for treatment and prevention of pulmonary, venous stasis and urinary complications associated with mobility.

A nurse is evaluating care of an immobilized patient. Which action will the nurse take? a. Focus on whether the interdisciplinary team is satisfied with the care. b. Compare the patient's actual outcomes with the outcomes in the care plan. c. Involve primarily the patient's family and health care team to determine goal achievement. d. Use objective data solely in determining whether interventions have been successful.

ANS: B From your perspective as the nurse, you are to evaluate outcomes and response to nursing care and compare the patient's actual outcomes with the outcomes selected during planning. Ask if the patient's expectations (subjective data) of care are being met and use objective data to determine the success of interventions. Just as it was important to include the patient during the assessment and planning phase of the care plan, it is essential to have the patient's evaluation of the plan of care, not just the patient's family and health care team.

A patient has a decreased gag reflex, left-sided weakness, and drooling. Which action will the nurse take when feeding this patient? a. Position in semi-Fowler's. b. Flex head with chin down. c. Place food on left side. d. Offer fruit juice.

ANS: B Have the patient flex the head slightly to a chin-down position to help prevent aspiration. If the patient has unilateral weakness, teach him or her and the caregiver to place food in the stronger side of the mouth. Provide a 30-minute rest period before eating and position the patient in an upright, seated position in a chair or raise the head of the bed to 90 degrees. Thin liquids such as water and fruit juice are difficult to control in the mouth and are more easily aspirated.

The nurse is teaching a patient how to use a cane. Which information will the nurse include in the teaching session? a. Place the cane at the top of the hip bone. b. Place the cane on the stronger side of the body. c. Place the cane in front of the body and then move the good leg. d. Place the cane 10 to 15 inches in front of the body when walking.

ANS: B Have the patient keep the cane on the stronger side of the body. A person's cane length is equal to the distance between the greater trochanter and the floor. The cane should be moved first and then the weaker leg. For maximum support when walking, the patient places the cane forward 15 to 25 cm (6 to 10 inches), keeping body weight on both legs. The weaker leg is then moved forward to the cane, so body weight is divided between the cane and the stronger leg.

Before being administered a cleansing enema an 80-year-old patient says ―I don't think I will be able to hold the enema.‖ Which is the next priority nursing action? a. Rolling the patient into right-lying Sims' position b. Positioning the patient in the dorsal recumbent position on a bedpan c. Inserting a rectal plug to contain the enema solution after administering d. Assisting the patient to the bedside commode and administering the enema

ANS: B If you suspect the patient of having poor sphincter control, position on bedpan in a comfortable dorsal recumbent position. Patients with poor sphincter control are unable to retain all of the enema solution. Administering an enema with the patient sitting on the toilet is unsafe because it is impossible to safely guide the tubing into the rectum, and it will be difficult for the patient to retain the fluid as he or she is in the position used for emptying the bowel. Rolling the patient into right-lying Sims' position will not help the patient retain the enema. Use of a rectal plug to contain the solution is inappropriate and unsafe.

A nurse is assessing pressure points in a patient placed in the Sims' position. Which areas will the nurse observe? a. Chin, elbow, hips b. Ileum, clavicle, humerus c. Shoulder, anterior iliac spine, ankles d. Occipital region of the head, coccyx, heels

ANS: B In the Sims' position, pressure points include the ileum, humerus, clavicle, knees, and ankles. The lateral position pressure points include the ear, shoulder, anterior iliac spine, and ankles. The prone position pressure points include the chin, elbows, female breasts, hips, knees, and toes. Supine position pressure points include the occipital region of the head, vertebrae, coccyx, elbows, and heels.

A patient diagnosed with diabetes mellitus is starting an exercise program. Which types of exercises will the nurse suggest? a. Low intensity b. Low to moderate intensity c. Moderate to high intensity d. High intensity

ANS: B Instruct patients diagnosed with diabetes mellitus to perform low- to moderate-intensity exercises, carry a concentrated form of carbohydrates (sugar packets or hard candy), and wear a medical alert bracelet. Low intensity is not beneficial. Moderate to high and high intensity are not recommended for a beginner exercise program.

A nurse makes a connection with the patient when providing spiritual care. Which type of connectedness did the nurse experience? a. Intrapersonal b. Interpersonal c. Transpersonal d. Multipersonal

ANS: B Interpersonal means connected with others and the environment. Intrapersonal means connected within oneself. Transpersonal means connected with God or an unseen higher power. There is no such term as multipersonal for connectedness.

The nurse is developing a plan of care for a patient diagnosed with activity intolerance. Which strategy will the nurse use to provide the best chance of maintaining patient compliance? a. Performing 20 minutes of aerobic exercise 7 days a week with 10-minute warm-up and cool-down periods b. Instructing the patient to use an exercise log to record day, time, duration, and responses to exercise activity c. Stressing the harm of not exercising by getting the patient to take responsibility for current health status d. Arranging for the patient to join a gym that takes self-pay rather than insurance

ANS: B Keeping a log may increase adherence to an exercise prescription. Recommended frequency of aerobic exercise is 3 to 5 times per week or every other day for approximately 30 minutes. Focusing on the harm of not exercising is usually counterproductive. Instead, the nurse should instruct the patient about the physiological benefits of a regular exercise program. Developing a plan of exercise that the patient may perform at home may improve compliance.

An adolescent tells the nurse that a health professional said the fibrous tissue that connects bone and cartilage was strained in a sporting accident. On which structure will the nurse focus an assessment? a. Tendon b. Ligament c. Synergistic muscle d. Antagonistic muscle

ANS: B Ligaments are white, shiny, and flexible bands of fibrous tissue that bind joints and connect bones and cartilage. Tendons are strong, flexible, and inelastic as they serve to connect muscle to bone. Muscles attach bone to bone. Synergistic muscles contract to accomplish the same movement. Antagonistic muscles cause movement at the joint.

A nurse is teaching patients about health care information. Which patient will the nurse assess closely for health literacy? a. A patient 35 years old b. A patient with a chronic illness c. A patient with a college degree d. A patient with a high-school diploma

ANS: B Many people in the United States experience challenges in using health care information. Patients who are especially vulnerable are the elderly (age 65+), immigrants, persons with low incomes, persons who do not have a high-school diploma or GED, and persons with chronic mental and/or physical health conditions. A 35-year-old patient and patients with high-school and college education are not identified in the vulnerable populations.

The nurse is assessing an immobile patient for deep vein thromboses (DVTs). Which action will the nurse take? a. Remove elastic stockings every 4 hours. b. Measure the calf circumference of both legs. c. Lightly rub the lower leg for redness and tenderness. d. Dorsiflex the foot while assessing for patient discomfort.

ANS: B Measure bilateral calf circumference and record it daily as an assessment for DVT. Unilateral increases in calf circumference are an early indication of thrombosis. Homan's sign, or calf pain on dorsiflexion of the foot, is no longer a reliable indicator in assessing for DVT, and it is present in other conditions. Remove the patient's elastic stockings and/or sequential compression devices (SCDs) every 8 hours, and observe the calves for redness, warmth, and tenderness. Instruct the family, patient, and all health care personnel not to massage the area because of the danger of dislodging the thrombus.

The nurse is preparing to test a patient for postvoid residual with a bladder scan. Which action will the nurse take? a. Measure bladder before the patient voids. b. Measure bladder within 15 minutes after the patient voids. c. Measure bladder with head of bed raised to 60 degrees. d. Measure bladder with head of bed raised to 90 degrees.

ANS: B Measurement should be within 5 to 15 minutes of voiding. It is a postvoid so the measurement is after the patient voids and the urine volume is recorded. Patient is supine with head slightly elevated.

A nurse is caring for a patient with a wound. Which assessment data will be most relevant with regard to wound healing? a. Muscular strength assessment b. Pulse oximetry assessment c. Sensation assessment d. Sleep assessment

ANS: B Oxygen fuels the cellular functions essential to the healing process; the ability to perfuse tissues with adequate amounts of oxygenated blood is critical in wound healing. Pulse oximetry measures the oxygen saturation of blood. Assessment of muscular strength and sensation, although useful for fitness and mobility testing, does not provide any data with regard to wound healing. Sleep, although important for rest and for integration of learning and restoration of cognitive function, does not provide any data with regard to wound healing.

The nurse is caring for a patient who has experienced a stroke causing total paralysis of the right side. To help maintain joint function and minimize the disability from contractures, passive range of motion (ROM) will be initiated. When should the nurse begin this therapy? a. After the acute phase of the disease has passed b. As soon as the ability to move is lost c. Once the patient enters the rehab unit d. When the patient requests it

ANS: B Passive ROM exercises should begin as soon as the patient's ability to move the extremity or joint is lost. The nurse should not wait for the acute phase to end. It may be some time before the patient enters the rehab unit or the patient requests it, and contractures could form by then.

Which statement by the patient about an upcoming contrast computed tomography (CT) scan indicates a need for further teaching? a. ―I will follow the food and drink restrictions as directed before the test is scheduled.‖ b. ―I will be anesthetized so that I lie perfectly still during the procedure.‖ c. ―I will complete my bowel prep program the night before the scan.‖ d. ―I will be drinking a lot of fluid after the test is over.‖

ANS: B Patients are not put under anesthesia for a CT scan; instead, the nurse should educate patients about the need to lie perfectly still and about possible methods of overcoming feelings of claustrophobia. The other options are correct and require no further teaching. Patients need to be assessed for an allergy to shellfish if receiving contrast for the CT scan. Bowel cleansing is often performed before CT scan. Another area to address is food and fluid restriction up to 4 hours prior to the test. After the procedure, encourage fluids to promote dye excretion.

A patient is receiving opioids for pain. Which bowel assessment is a priority? a. C. difficile b. Constipation c. Hemorrhoids d. Diarrhea

ANS: B Patients receiving opiates for pain after surgery often require a stool softener or laxative to prevent constipation. Clostridium difficile occurs from antibiotics, not opioids. Hemorrhoids are caused by conditions other than opioids. Diarrhea does not occur as frequently as constipation.

The patient is admitted with facial trauma, including a broken nose, and has a history of esophageal reflux and of aspiration pneumonia. With which tube will the nurse most likely administer the feeding? a. Nasogastric tube b. Jejunostomy tube c. Nasointestinal tube d. Percutaneous endoscopic gastrostomy (PEG) tube

ANS: B Patients with gastroparesis or esophageal reflux or with a history of aspiration pneumonia may require placement of tubes beyond the stomach into the intestine. The jejunostomy tube is the only tube in the list that is beyond the stomach and is not contraindicated by facial trauma. The nasogastric tube and the PEG tube are placed in the stomach, and placement could lead to aspiration. The nasointestinal tube and the nasogastric tube may be contraindicated by facial trauma and the broken nose.

A nurse is caring for an immigrant with low income. Which information should the nurse consider when planning care for this patient? a. There is a decreased frequency of morbidity. b. There is an increased incidence of disease. c. There is an increased level of health. d. There is a decreased mortality rate.

ANS: B Populations with health disparities (immigrant with low income) have a significantly increased incidence of disease or increased morbidity and mortality when compared with the general population. Although Americans' health overall has improved during the past few decades, the health of members of marginalized groups has actually declined.

The nurse caring for an unconscious patient who was involved in an automobile accident 2 weeks ago will give priority to which element when planning care to decrease the development of a decubitus ulcer? a. Resistance b. Pressure c. Weight d. Stress

ANS: B Pressure is the main element that causes pressure ulcers. Three pressure-related factors contribute to pressure ulcer development: pressure intensity, pressure duration, and tissue tolerance. When the intensity of the pressure exerted on the capillary exceeds 15 to 32 mm Hg, this occludes the vessel, causing ischemic injury to the tissues it normally feeds. High pressure over a short time and low pressure over a long-time cause skin breakdown. Resistance, stress, and weight are not the priority causes of pressure ulcers.

The nurse caring for a patient with a healing Stage III pressure ulcer notes that the wound is clean and granulating. Which health care provider's order will the nurse question? a. Use a low-air-loss therapy unit. b. Irrigate with Dakin's solution. c. Apply a hydrogel dressing. d. Consult a dietitian.

ANS: B Pressure ulcers should be with noncytotoxic cleansers such as normal saline, which will not kill fibroblasts and healing tissue. Cytotoxic cleansers such as Dakin's solution, acetic acid, povidone-iodine, and hydrogen peroxide can hinder the healing process and should not be utilized on clean, granulating wounds. Consulting a dietitian for the nutritional needs of the patient, utilizing a low-air-loss therapy unit to decrease pressure, and applying hydrogel dressings to provide a moist environment for healing are all orders that would be appropriate.

The nurse is collaborating with the dietitian about a patient with a Stage III pressure ulcer. Which nutrient will the nurse expect to be increased after collaboration with the dietitian? a. Fat b. Protein c. Vitamin E d. Carbohydrate

ANS: B Protein needs are especially increased in supporting the activity of wound healing. The physiological processes of wound healing depend on the availability of protein, vitamins (especially A and C), and the trace minerals of zinc and copper. Wound healing does not require increased amounts of fats or carbohydrates. Vitamin E will not be increased for wound healing.

Which health care team member will the nurse consult when a patient has received a nursing diagnosis of Impaired skin integrity? a. Respiratory therapist b. Registered dietitian c. Case manager d. Chaplain

ANS: B Refer patients with pressure ulcers to the dietitian for early intervention for nutritional problems. Adequate calories, protein, vitamins, and minerals promote wound healing for the impaired skin integrity. The nurse is the coordinator of care and collaborating with the dietitian would result in planning the best meals for the patient. The respiratory therapist can be consulted when a patient has issues with the respiratory system. Case management can be consulted when the patient has a discharge need. A chaplain can be consulted when the patient has a spiritual need.

A nurse reviews an immobilized patient's laboratory results and discovers hypercalcemia. Which condition will the nurse monitor for most closely in this patient? a. Hypostatic pneumonia b. Renal stones c. Pressure ulcers d. Thrombus formation

ANS: B Renal calculi are calcium stones that lodge in the renal pelvis or pass through the ureters. Immobilized patients are at risk for calculi because they frequently have hypercalcemia. Hypercalcemia does not lead to hypostatic pneumonia, pressure ulcers, or thrombus formation. Immobility is one cause of hypostatic pneumonia, which is inflammation of the lung from stasis or pooling of secretions. A pressure ulcer is an impairment of the skin that results from prolonged ischemia (decreased blood supply) within tissues. A thrombus is an accumulation of platelets, fibrin, clotting factors, and cellular elements of the blood attached to the interior wall of a vein or artery, which sometimes occludes the lumen of the vessel.

The patient is prescribed phenazopyridine. When assessing the urine, what will the nurse expect? a. Red color b. Orange color c. Dark amber color d. Intense yellow color

ANS: B Some drugs change the color of urine (e.g., phenazopyridine—orange, riboflavin—intense yellow). Eating beets, rhubarb, and blackberries causes red urine. Dark amber urine is the result of high concentrations of bilirubin in patients with liver disease.

1. A co-worker asks the nurse to explain spirituality. What is the nurse's best response? a. It has a minor effect on health. b. It is awareness of one's inner self. c. It is not as essential as physical needs. d. It refers to fire or giving of life to a person.

ANS: B Spirituality is often defined as an awareness of one's inner self and a sense of connection to a higher being, to nature, or to some purpose greater than oneself. Spirituality is an important factor that helps individuals achieve the balance needed to maintain health and well-being and to cope with illness. Florence Nightingale believed that spirituality was a force that provided energy needed to promote a healthy hospital environment and that caring for a person's spiritual needs was just as essential as caring for his or her physical needs. The word spirituality comes from the Latin word spiritus, which refers to breath or wind. The spirit gives life to a person

The nurse is caring for a patient with a Stage IV pressure ulcer. Which type of healing will the nurse consider when planning care for this patient? a. Partial thickness wound repair b. Full thickness wound repair c. Primary intention d. Tertiary intention

ANS: B Stage IV pressure ulcers are full-thickness wounds that extend into the dermis and heal by scar formation because the deeper structures do not regenerate, hence the need for full-thickness repair. The full-thickness repair has four phases: hemostasis, inflammatory, proliferative, and maturation. A wound heals by primary intention when wounds such as surgical wounds have little tissue loss; the skin edges are approximated or closed, and the risk for infection is low. Partial-thickness repairs are done on partial-thickness wounds that are shallow, involving loss of the epidermis and maybe partial loss of the dermis. These wounds heal by regeneration because the epidermis regenerates. Tertiary intention is seen when a wound is left open for several days, and then the wound edges are approximated. Wound closure is delayed until risk of infection is resolved.

Which patient is most at risk for increased peristalsis? a. A 5-year-old child who ignores the urge to defecate owing to embarrassment b. A 21-year-old female with three final examinations on the same day c. A 40-year-old female with major depressive disorder d. An 80-year-old male in an assisted-living environment

ANS: B Stress can stimulate digestion and increase peristalsis, resulting in diarrhea; three finals on the same day is stressful. Ignoring the urge to defecate, depression, and age-related changes of the older adult (80-year-old man) are causes of constipation, which is from slowed peristalsis.

The nurse is caring for a patient in the emergency department with an injured elbow. Which type of joint will the nurse assess? a. Fibrous b. Synovial c. Synergistic d. Cartilaginous

ANS: B Synovial joints, or true joints, such as the hinge type at the elbow, are freely movable and the most mobile, numerous, and anatomically complex body joints. Fibrous joints fit closely together and are fixed, permitting little, if any, movement such as the syndesmosis between the tibia and the fibula. Synergistic is a type of muscle, not joint. Cartilaginous joints have little movement but are elastic and use cartilage to unite separate bony surfaces such as the synchondrosis that attaches the ribs to the costal cartilage.

The nurse documents the following assessment data: right heel with reddened area that does not blanch. Which nursing diagnosis will the nurse assign to this patient? a. Imbalanced nutrition: less than body requirements b. Impaired peripheral tissue perfusion c. Risk for infection d. Acute pain

ANS: B The area on the heel has experienced a decreased supply of blood and oxygen (tissue perfusion), which has resulted in tissue damage. The most appropriate nursing diagnosis with this information is Impaired peripheral tissue perfusion. Risk for infection, Acute pain, and Imbalanced nutrition do not support the data in the question.

A nurse is pouching an ostomy on a patient with an ileostomy. Which action by the nurse is most appropriate? a. Changing the skin barrier portion of the ostomy pouch daily b. Emptying the pouch at least once every 7 days. c. Thoroughly cleansing the skin around the stoma with soap and water to remove excess stool and adhesive d. Measuring the correct size for the barrier device while leaving a 1/2-inch space around the stoma

ANS: B The barrier device should be changed every 3 to 7 days unless it is leaking or is no longer effective. Peristomal skin should be gently cleansed; vigorous rubbing can cause further irritation or skin breakdown. Avoid soap. It leaves a residue on skin, which may irritate the skin. The pouch opening should fit around the stoma and cover the peristomal skin to prevent contact with the effluent. Excess space, like 1/2 inch, allows fecal matter to have prolonged exposure to skin, resulting in skin breakdown.

A nurse is teaching about the energy needed at rest to maintain life-sustaining activities for a specific period of time. What form of energy is the nurse discussing? a. Resting energy expenditure (REE) b. Basal metabolic rate (BMR) c. Nutrient density d. Nutrients

ANS: B The basal metabolic rate (BMR) is the energy needed at rest to maintain life-sustaining activities for a specific period of time. The resting energy expenditure (REE), or resting metabolic rate, is the amount of energy an individual needs to consume over a 24-hour period for the body to maintain all of its internal working activities while at rest. Nutrients are the elements necessary for body processes and function. Nutrient density is the proportion of essential nutrients to the number of kilocalories. High-nutrient density foods provide a large number of nutrients in relation to kilocalories.

Which nursing intervention is most effective in promoting normal defecation for a patient who has muscle weakness in the legs? a. Administer a soapsuds enema every 2 hours. b. Use a mobility device to place the patient on a bedside commode. c. Give the patient a pillow to brace against the abdomen while bearing down. d. Elevate the head of the bed 20 degrees 60 minutes after breakfast while on bedpan.

ANS: B The best way to promote normal defecation is to assist the patient into a posture that is as normal as possible for defecation. Using a mobility device promotes nurse and patient safety. Elevating the head of the bed is appropriate but is not the most effective; closer to 30 to 45 degrees is the proper position for the patient on a bedpan, and the patient is not on bed rest so a bedside commode is the best choice. Giving the patient a pillow may reduce discomfort, but this is not the best way to promote defecation. A soapsuds enema is indicated for a patient who needs assistance to stimulate peristalsis. It promotes non-natural defecation.

In providing diabetic teaching for a patient diagnosed with type 1 diabetes mellitus, which instructions will the nurse provide to the patient? a. Insulin is the only consideration that must be taken into account. b. Saturated fat should be limited to less than 7% of total calories. c. Nonnutritive sweeteners can be used without restriction. d. Cholesterol intake should be greater than 200 mg/day.

ANS: B The diabetic patient should limit saturated fat to less than 7% of total calories and cholesterol intake to less than 200 mg/day. Type 1 diabetes requires both insulin and dietary restrictions for optimal control. Nonnutritive sweeteners can be eaten as long as the recommended daily intake levels are followed.

Which patient will cause the nurse to select a nursing diagnosis of Impaired physical mobility for a care plan? a. A patient who is completely immobile b. A patient who is not completely immobile c. A patient at risk for single-system involvement d. A patient who is at risk for multisystem problems

ANS: B The diagnosis of Impaired physical mobility applies to the patient who has some limitation but is not completely immobile. The diagnosis of Risk for disuse syndrome applies to the patient who is immobile and at risk for multisystem problems because of inactivity. Beyond these diagnoses, the list of potential diagnoses is extensive because immobility affects multiple body systems.

A nurse is teaching a patient about the urinary system. In which order will the nurse present the structures, following the flow of urine? a. Kidney, urethra, bladder, ureters b. Kidney, ureters, bladder, urethra c. Bladder, kidney, ureters, urethra d. Bladder, kidney, urethra, ureters

ANS: B The flow of urine follows these structures in this order: kidney, ureters, bladder, and urethra.

A nurse is providing care to a group of patients. Which patient will the nurse see first? a. A patient who is dribbling urine and has a diagnosis of urge incontinence b. A patient with reflex incontinence with elevated blood pressure and pulse rate c. A patient with an indwelling catheter that has stool on the catheter tubing d. A patient who has just voided and needs a postvoid residual test

ANS: B The nurse should see the patient with reflex incontinence first. Patients with reflex incontinence are at risk for developing autonomic dysreflexia, a life-threatening condition that causes severe elevation of blood pressure and pulse rate and diaphoresis. This is a medical emergency requiring immediate intervention; notify the health care provider immediately. A patient with urge incontinence will dribble, and this is expected. While a patient with a catheter and stool on the tubing does need to be cleaned, it is not life threatening. The nurse has 10 minutes before checking on the patient who has a postvoid residual test.

A nurse is caring for a group of patients. Which patient will the nurse see first? a. Patient receiving total parenteral nutrition of 2-in-1 for 50 hours b. Patient receiving total parenteral nutrition infusing with same tubing for 26 hours c. Patient receiving continuous enteral feeding with same feeding bag for 12 hours d. Patient receiving continuous enteral feeding with same tubing for 24 hours

ANS: B The nurse should see the patient with total parenteral nutrition that has the same tubing for 26 hours. To prevent infection, change the TPN infusion tubing every 24 hours. Change the administration system every 72 hours when infusing a 2-in-1 solution and every 24 hours for a 3-in-1 solution. Change bag and use a new administration set every 24 hours for a continuous enteral feeding. While the patient with the continuous enteral feeding has the same tubing for 24 hours, it has not extended the time like the total parenteral nutrition has.

When caring for a group of patients, which task can the nurse delegate to the nursing assistive personnel (AP)? a. Assessing a surgical patient for risk of pressure ulcers b. Applying a gauze bandage to secure a nonsterile dressing c. Treating a pressure ulcer on the buttocks of a medical patient d. Implementing negative pressure wound therapy on a stable patient

ANS: B The skill of applying bandages to secure nonsterile dressings can be delegated to AP. Assessing pressure ulcer risk, treating a pressure ulcer, and implementing negative pressure wound therapy cannot be delegated to an AP.

A patient requires repositioning every 2 hours. Which task can the nurse delegate to the nursing assistive personnel? a. Determining the level of comfort b. Changing the patient's position c. Identifying immobility hazards d. Assessing circulation

ANS: B The skill of moving and positioning patients in bed can be delegated to nursing assistive personnel (NAP). The nurse is responsible for assessing the patient's level of comfort and for any hazards of immobility and assessing circulation.

The nurse performing a moist-to-dry dressing has prepared the supplies, solution, and removed the old dressing. In which order will the nurse implement the following steps, starting with the first one? 1. Apply sterile gloves. 2. Cover and secure topper dressing. 3. Assess wound and surrounding skin. 4. Moisten gauze with prescribed solution. 5. Gently wring out excess solution and unfold. 6. Loosely pack until all wound surfaces are in contact with gauze. a. 4, 3, 1, 5, 6, 2 b. 1, 3, 4, 5, 6, 2 c. 4, 1, 3, 5, 6, 2 d. 1, 4, 3, 5, 6, 2

ANS: B The steps for a moist-to-dry dressing are as follows: (1) apply sterile gloves, (2) assess appearance of surrounding skin, (3) moisten gauze with prescribed solution, (4) gently wring out excess solution and unfold; apply gauze as single layer directly onto wound surface, (5) if wound is deep, gently pack dressing into wound base by hand until all wound surfaces are in contact with gauze, and (6) cover with sterile dry gauze and secure topper dressing.

The patient is eager to begin an exercise program with a 2-mile jog. The nurse instructs the patient to warm up. The patient does not want to waste time with a ―warm-up.‖ Which information will the nurse share with the patient? a. The warm-up in this case can be done after the 2-mile jog. b. The warm-up prepares the body and decreases the potential for injury. c. The warm-up allows the body to readjust gradually to baseline functioning. d. The warm-up should be performed with high intensity to prepare for the coming challenge

ANS: B The warm-up activity prepares the body for activity and decreases the potential for injury and should not be omitted. It usually lasts about 5 to 10 minutes and may include stretching, calisthenics, and/or aerobic activity performed at a lower intensity. The warm-up is before the exercise, while the cool-down period is after the exercise. The cool-down, not the warm-up, allows the body to readjust gradually to baseline functioning and provides an opportunity to combine movement such as stretching with relaxation-enhancing mind-body awareness. The warm-up should not be a high-intensity workout

A nurse is performing a cultural assessment using the LEARN mnemonic for communication. Which area will the nurse assess for the ―L‖? a. Look b. Listen c. Liken d. Leave

ANS: B The ―L‖ in Learn stands for Listen: Listen to the patient's perception of the problem. Be nonjudgmental and use encouraging comments such as, ―Tell me more‖ or ―I understand what you are saying.‖

The nurse admitting an older patient notes a shallow open reddish, pink ulcer without slough on the right heel of the patient. How will the nurse stage this pressure ulcer? a. Stage I b. Stage II c. Stage III d. Stage IV

ANS: B This would be a Stage II pressure ulcer because it presents as partial-thickness skin loss involving epidermis and dermis. The ulcer presents clinically as an abrasion, blister, or shallow crater. Stage I is intact skin with non-blanchable redness over a bony prominence. With a Stage III pressure ulcer, subcutaneous fat may be visible, but bone, tendon, and muscles are not exposed. Stage IV involves full-thickness tissue loss with exposed bone, tendon, or muscle.

A nurse is using core measures to reduce health disparities. Which group should the nurse focus on to cause the most improvement in core measures? a. Caucasians b. Poor people c. Alaska Natives d. American Indians

ANS: B To improve results, the nurse should focus on the highest disparity. Poor people consistently received inferior care compared to high-income people. American Indians and Alaska Natives received worse care than Caucasians.

The patient has been in bed for several days and needs to be ambulated. Which action will the nurse take first? a. Maintain a narrow base of support. b. Dangle the patient at the bedside. c. Encourage isometric exercises. d. Suggest a high-calcium diet.

ANS: B To prevent injury, nurses implement interventions that reduce or eliminate the effects of orthostatic hypotension. Mobilize the patient as soon as the physical condition allows, even if this only involves dangling at the bedside or moving to a chair. A wide base of support increases balance. Isometric exercises (i.e., activities that involve muscle tension without muscle shortening) have no beneficial effect on preventing orthostatic hypotension, but they improve activity tolerance. A high-calcium diet can help with osteoporosis but can be detrimental in an immobile patient.

A female patient is having difficulty voiding in a bedpan but states that her bladder feels full. To stimulate micturition, which nursing intervention should the nurse try first? a. Exiting the room and informing the patient that the nurse will return in 30 minutes to check on the patient's progress b. Utilizing the power of suggestion by turning on the faucet and letting the water run c. Obtaining an order for a Foley catheter d. Administering diuretic medication

ANS: B To stimulate micturition, the nurse should attempt noninvasive procedures first. Running warm water or stroking the inner aspect of the upper thigh promotes sensory perception that leads to urination. A patient should not be left alone on a bedpan for 30 minutes because this could cause skin breakdown. Catheterization places the patient at increased risk of infection and should not be the first intervention attempted. Diuretics are useful if the patient is not producing urine, but they do not stimulate micturition.

A patient develops a foodborne disease from Escherichia coli. When taking a health history, which food item will the nurse most likely find the patient ingested? a. Improperly home-canned food b. Undercooked ground beef c. Soft cheese d. Custard

ANS: B Undercooked ground beef is the usual food source for E. coli. Botulism is associated with improperly home-canned foods. Soft cheese is the usual food source for listeriosis. Custards are associated with salmonellosis and Staphylococcus.

An 86-year-old patient is experiencing uncontrollable leakage of urine with a strong desire to void and even leaks on the way to the toilet. Which priority nursing diagnosis will the nurse include in the patient's plan of care? a. Functional urinary incontinence b. Urge urinary incontinence c. Impaired skin integrity d. Urinary retention

ANS: B Urge urinary incontinence is the leakage of urine associated with a strong urge to void. Patients leak urine on the way to or at the toilet and rush or hurry to the toilet. Urinary retention is the inability to empty the bladder. Functional urinary incontinence is incontinence due to causes outside the urinary tract, such as mobility or cognitive impairments. While Impaired skin integrity can occur, it is not the priority at this time, and there is no data to support this diagnosis.

While receiving a shift report on a female patient, the nurse is informed that the patient has been experiencing urinary incontinence. Upon assessment, which finding will the nurse expect? a. An indwelling Foley catheter b. Reddened irritated skin on buttocks c. Tiny blood clots in the patient's urine d. Foul-smelling discharge indicative of infection

ANS: B Urinary incontinence is uncontrolled urinary elimination; if the urine has prolonged contact with the skin, skin breakdown can occur. An indwelling Foley catheter is a solution for urine retention. Blood clots and foul-smelling discharge are often signs of infection.

A male patient in stable condition is in the intensive care unit (ICU) and is asking to see his spouse and two daughters. What should the nurse be prepared to plan for? a. 5 to 10 minutes for the family visit b. Complying with the patient's request c. Arranging for the two daughters to visit, then the wife d. Asking the family to decide which two persons will visit

ANS: B Use of support systems is important in any health care setting. Allowing the family to visit is appropriate since the patient is in stable condition. When patients depend on family and friends for support, encourage them to visit the patient. As long as no interference with active patient care is involved, there is no reason to limit visitation. Limiting the visit is not necessary since the patient is stable. Breaking the family apart is not needed; the patient is stable and can see all three at once.

The nurse is preparing to transfer an uncooperative patient who does not have upper body strength. Which piece of equipment will be best for the nurses to obtain? a. Drawsheet b. Full body sling c. Overhead trapeze d. Friction-reducing slide sheet

ANS: B Using a mechanical lift and full body sling to transfer an uncooperative patient who can bear partial weight or a patient who cannot bear weight and is either uncooperative or does not have upper body strength to move from bed to chair prevents musculoskeletal injuries to health care workers. The nurse should not attempt to move the patient with a drawsheet. The patient does not have upper body strength, so an overhead trapeze is not appropriate. A friction-reducing slide sheet that minimizes shearing forces is not as effective as a full body sling.

The nurse has attempted to administer a tap water enema for a patient with fecal impaction with no success. The fecal mass is too large for the patient to pass voluntarily. Which is the next priority nursing action? a. Preparing the patient for a second tap water enema b. Obtaining an order for digital removal of stool c. Positioning the patient on the left side d. Inserting a rectal tube

ANS: B When enemas are not successful, digital removal of the stool may be necessary to break up pieces of the stool or to stimulate the anus to defecate. Tap water enemas should not be repeated because of risk of fluid imbalance. Positioning the patient on the left side does not promote defecation. A rectal tube is indicated for a patient with liquid stool incontinence or flatus but would not be applicable or effective for this patient.

The nurse is preparing to lift and reposition a patient. Which action will the nurse take first? a. Position a drawsheet under the patient. b. Assess weight to determine assistance needs. c. Delegate the task to a nursing assistive personnel. d. Attempt to manually lift the patient alone before asking for assistance

ANS: B When lifting, assess the weight you will lift, and determine the assistance you will need. The nurse has to assess before positioning a drawsheet or delegating the task. Manual lifting is the last resort, and it is used when the task at hand does not involve lifting most or all of the patient's weight; most facilities have a no-lift policy.

The nurse is caring for a patient who is immobile and is at risk for skin impairment. The plan of care includes turning the patient. Which is the best method for repositioning the patient? a. Place the patient in a 30-degree supine position. b. Utilize a transfer device to lift the patient. c. Elevate the head of the bed 45 degrees. d. Slide the patient into the new position.

ANS: B When repositioning the patient, obtain assistance and utilize a transfer device to lift rather than drag the patient. Sliding the patient into the new position will increase friction. The patient should be placed in a 30-degree lateral position, not a supine position. The head of the bed should be elevated less than 30 degrees to prevent pressure ulcer development from shearing forces.

Which goal is most appropriate for a patient who has had a total hip replacement? a. The patient will ambulate briskly on the treadmill by the time of discharge. b. The patient will walk 100 feet using a walker by the time of discharge. c. The nurse will assist the patient to ambulate in the hall 2 times a day d. The patient will ambulate by the time of discharge.

ANS: B ―The patient will walk 100 feet using a walker by the time of discharge‖ is individualized, realistic, and measurable. ―Ambulating briskly on a treadmill‖ is not realistic for this patient. The option that focuses on the nurse, not the patient, is not a measurable goal; this is an intervention. ―The patient will ambulate by the time of discharge‖ is not measurable because it does not specify the distance. Even though we can see that the patient will ambulate, this does not quantify how far.

Which findings should the nurse follow up on after removal of a catheter from a patient? (Select all that apply.) a. Increasing fluid intake b. Dribbling of urine c. Voiding in small amounts d. Voiding within 6 hours of catheter removal e. Burning with the first couple of times voiding

ANS: B, C Abdominal pain and distention, a sensation of incomplete emptying, incontinence, constant dribbling of urine, and voiding in very small amounts can indicate inadequate bladder emptying requiring intervention. All the rest are normal and do not require follow-up. The patient should increase intake. The first few times a patient voids after catheter removal may be accompanied by some discomfort, but continued complaints of painful urination indicate possible infection. Patient should void 6 to 8 hours after catheter removal.

A nurse administers an antimuscarinic to a patient. A decrease in which findings indicate the patient is having therapeutic effects from this medication? (Select all that apply.) a. Dysuria b. Urgency c. Frequency d. Prostate size e. Bladder infection

ANS: B, C When newly started on an antimuscarinic, you should monitor the patient for effectiveness, watching for a decrease in symptoms such as urgency, frequency, and urgency urinary incontinence episodes. Patients with painful urination are sometimes prescribed urinary analgesics that act on the urethral and bladder mucosa (e.g., phenazopyridine). Antibiotics are used to treat bladder infections. Agents that shrink the prostate include finasteride and dutasteride.

The nurse is caring for a patient with potential skin breakdown. Which components will the nurse include in the skin assessment? (Select all that apply.) a. Vision b. Hyperemia c. Induration d. Blanching e. Temperature of skin

ANS: B, C, D, E Assessment of the skin includes both visual and tactile inspection. Assess for hyperemia and palpate for blanching or nonblanching. Early signs of skin damage include induration, bogginess (less-than-normal stiffness), and increased warmth at the injury site compared to nearby areas. Changes in temperature can indicate changes in blood flow to that area of the skin. Vision is not included in the skin assessment.

The nurse is updating the plan of care for a patient with impaired skin integrity. Which findings indicate achievement of goals and outcomes? (Select all that apply.) a. The patient's expectations are not being met. b. Skin is intact with no redness or swelling. c. Non-blanchable erythema is absent. d. No injuries to the skin and tissues are evident. e. Granulation tissue is present.

ANS: B, C, D, E Optimal outcomes are to prevent injury to skin and tissues, reduce injury to skin, reduce injury to underlying tissues, and restore skin integrity. Skin intact, non-blanchable erythema absent, no injuries, and presence of granulation tissue are all findings indicating achievement of goals and outcomes. The patient's expectations not being met indicate no progression toward goals/outcomes.

Which nursing actions will the nurse implement when collecting a urine specimen from a patient? (Select all that apply.) a. Growing urine cultures for up to 12 hours b. Labeling all specimens with date, time, and initials c. Allowing the patient adequate time and privacy to void d. Wearing gown, gloves, and mask for all specimen handling e. Transporting specimens to the laboratory in a timely manner f. Collecting the specimen from the drainage bag of an indwelling catheter

ANS: B, C, E All specimens should be labeled appropriately and processed in a timely fashion. Allow patients time and privacy to void. Urine cultures can take up to 48 to 72 hours to develop. Only gloves are necessary to handle a urine specimen. Gown and mask are not needed unless otherwise indicated. Never collect the specimen from the drainage bag of a catheter; obtain the sample from the special sampling port.

A nurse is using the Campinha-Bacote model to determine the cause of an illness. Which questions should the nurse ask? (Select all that apply.) a. How should your sickness be treated? b. What do you call your problem? c. How does this illness work inside your body? d. What do you fear most about your sickness? e. What name does it have?

ANS: B, C, E The questions for etiology include ―What do you call your problem?‖ and ―What name does it have?‖ Recommended treatment is asked by the question ―How should your sickness be treated?‖ Pathophysiology is asked by the question ―How does this illness work inside your body?‖ The course of illness is asked by the question ―What do you fear most about your sickness?‖

A nurse is preparing to move a patient who is able to assist. Which principles will the nurse consider when planning for safe patient handling? (Select all that apply.) a. Keep the body's center of gravity high. b. Face the direction of the movement. c. Keep the base of support narrow. d. Use the under-axilla technique. e. Use proper body mechanics. f. Use arms and legs.

ANS: B, E, F When a patient is able to assist, remember the following principles: The wider the base of support, the greater the stability of the nurse; the lower the center of gravity, the greater the stability of the nurse; facing the direction of movement prevents abnormal twisting of the spine. The use of assistive equipment and continued use of proper body mechanics significantly reduces the risk of musculoskeletal injuries. Use arms and legs (not back) because the leg muscles are stronger, larger muscles capable of greater work without injury. The under-axilla technique is physically stressful for nurses and uncomfortable for patients.

The nurse gives instructions to a nursing assistive personnel (NAP) regarding exercise for a patient. Which action by the NAP indicates a correct understanding of the directions? a. Determines the patient's ability to exercise. b. Teaches the patient how to do the exercises. c. Reports the patient got dizzy after exercising. d. Advises the patient to work through the pain.

ANS: C The NAP notifies the nurse if a patient reports increased fatigue, dizziness, or light-headedness when obtaining pre-exercise and/or post-exercise vital signs. The nurse first must assess the patient's ability and tolerance to exercise. The nurse also teaches patients and their families how to implement exercise programs. The NAP can prepare patients for exercise (e.g., putting on shoes and clothing, providing hygiene needs, and obtaining pre-exercise and post-exercise vital signs). The NAP can help the patient exercise.

The nurse will anticipate inserting a Coudé catheter for which patient? a. An 8-year-old male undergoing anesthesia for a tonsillectomy b. A 24-year-old female who is going into labor c. A 56-year-old male admitted for bladder irrigation d. An 86-year-old female admitted for a urinary tract infection

ANS: C A Coudé catheter has a curved tip that is used for patients with enlarged prostates. This would be indicated for a middle-aged male who needs bladder irrigation. Coudé catheters are not indicated for children or women.

. A nurse is preparing to assess a patient for orthostatic hypotension. Which piece of equipment will the nurse obtain to assess for this condition? a. Thermometer b. Elastic stockings c. Blood pressure cuff d. Sequential compression devices

ANS: C A blood pressure cuff is needed. Orthostatic hypotension is a drop of blood pressure greater than 20 mm Hg in systolic pressure or 10 mm Hg in diastolic pressure and symptoms of dizziness, light-headedness, nausea, tachycardia, pallor, or fainting when the patient changes from the supine to standing position. A thermometer is used to assess for fever. Elastic stockings and sequential compression devices are used to prevent thrombus.

The nurse administers a cathartic to a patient. Which finding helps the nurse determine that the cathartic has a therapeutic effect? a. Reports decreased diarrhea. b. Experiences pain relief. c. Has a bowel movement. d. Passes flatulence.

ANS: C A cathartic is a laxative that stimulates a bowel movement. It would be effective if the patient experiences a bowel movement. The other options are not outcomes of administration of a cathartic. An antidiarrheal will provide relief from diarrhea. Pain medications will provide pain relief. Carminative enemas provide relief from gaseous distention (flatulence).

A nurse is providing care to a group of patients. Which patient will the nurse assess first? a. A child about to receive a normal saline enema b. A teenager about to receive loperamide for diarrhea c. A dehydrated older patient about to receive a hypertonic enema d. A middle-aged patient with myocardial infarction about to receive docusate sodium

ANS: C A hypertonic enema is contradicted in a dehydrated patient since it will pull fluid out of the body; this patient should be seen assessed first. All the rest are expected. A child can receive normal saline enemas since they are isotonic. Loperamide, an antidiarrheal, is given for diarrhea. Docusate sodium is given to soften stool for patients with myocardial infarction to prevent straining.

A patient is receiving a neomycin solution enema. Which primary goal is the nurse trying to achieve? a. Prevent gaseous distention. b. Prevent constipation. c. Prevent colon infection. d. Prevent lower bowel inflammation.

ANS: C A medicated enema is a neomycin solution, that is, an antibiotic used to reduce bacteria in the colon before bowel surgery. Carminative enemas provide relief from gaseous distention. Bulk forming, emollient (wetting), and osmotic laxatives and cathartics help prevent constipation or treat constipation. An enema containing steroid medication may be used for acute inflammation in the lower colon.

The nurse is caring for a patient who had a colostomy placed yesterday. The nurse should report which assessment finding immediately? a. Stoma is protruding from the abdomen. b. Stoma is flush with the skin. c. Stoma is purple. d. Stoma is moist.

ANS: C A purple stoma may indicate strangulation/necrosis or poor circulation to the stoma and may require surgical intervention. A stoma should be reddish-pink and moist in appearance. It can be flush with the skin, or it can protrude.

The nurse is caring for a patient with a spinal cord injury and notices that the patient's hips have a tendency to rotate externally when the patient is supine. Which device will the nurse use to help prevent injury secondary to this rotation? a. Hand rolls b. A trapeze bar c. A trochanter roll d. Hand-wrist splints

ANS: C A trochanter roll prevents external rotation of the hips when the patient is in a supine position. Hand rolls maintain the thumb in slight adduction and in opposition to the fingers. Hand-wrist splints are individually molded for the patient to maintain proper alignment of the thumb and the wrist. The trapeze bar is a triangular device that hangs down from a securely fastened overhead bar that is attached to the bedframe. It allows the patient to pull with the upper extremities to raise the trunk off the bed, to assist in transfer from bed to wheelchair, or to perform upper arm exercises.

The nurse performing a fecal occult blood test should take what action? a. Test the quality control section before testing the stool specimens. b. Apply liberal amounts of stool to the guaiac paper. c. Report a positive finding to the provider. d. Don sterile disposable gloves.

ANS: C Abnormal findings such as a positive test (turns blue) should be reported to the provider. A fecal occult blood test is a clean procedure; sterile gloves are not needed. A thin specimen smear is all that is required. The quality control section should be developed after it is determined whether the sample is positive or negative.

The nurse will irrigate a patient's nasogastric (NG) tube. Which action should the nurse take? a. Instill solution into pigtail slowly. b. Check placement after instillation of solution. c. Immediately aspirate after instilling fluid. d. Prepare 60 mL of tap water into Asepto syringe.

ANS: C After instilling saline, immediately aspirate or pull back slowly on syringe to withdraw fluid. Do not introduce saline through blue ―pigtail‖ air vent of Salem sump tube. Checking placement before instillation of normal saline prevents accidental entrance of irrigating solution into lungs. Draw up 30 mL of normal saline into Asepto syringe to minimize loss of electrolytes from stomach fluids

The nurse is caring for a patient with a Stage IV pressure ulcer. Which nursing diagnosis should the nurse add to the care plan? a. Readiness for enhanced nutrition b. Impaired physical mobility c. Impaired skin integrity d. Chronic pain

ANS: C After the assessment is completed and the information that the patient has a Stage IV pressure ulcer is gathered, a diagnosis of Impaired skin integrity is selected. Readiness for enhanced nutrition would be selected for an individual with an adequate diet that could be improved. Impaired physical mobility and Chronic pain do not support the current data in the question.

The patient diagnosed with cardiovascular disease is receiving dietary instructions from the nurse. Which information from the patient indicates teaching is successful? a. Maintain a prescribed carbohydrate intake. b. Eat fish at least 5 times/week. c. Limit cholesterol to less than 300 mg/daily. d. Avoid high-fiber foods.

ANS: C American Heart Association guidelines recommend limiting cholesterol to less than 300 mg/day. Diet therapy includes eating fish at least 2 times per week and eating whole grain high-fiber foods. Maintaining a prescribed carbohydrate intake is necessary for diabetes mellitus.

The patient is having lower abdominal surgery and the nurse inserts an indwelling catheter. What is the rationale for the nurse's action? a. The patient may void uncontrollably during the procedure. b. Local trauma sometimes promotes excessive urine incontinence. c. Anesthetics can decrease bladder contractility and cause urinary retention. d. The patient will not interrupt the procedure by asking to go to the bathroom.

ANS: C Anesthetic agents and other agents given during surgery can decrease bladder contractility and/or sensation of bladder fullness, causing urinary retention. Local trauma during lower abdominal and pelvic surgery sometimes obstructs urine flow, requiring temporary use of an indwelling urinary catheter. The patient is more likely to retain urine rather than experience uncontrollable voiding.

Which patient will the nurse assess most closely for an ileus? a. A patient with a fecal impaction b. A patient with chronic cathartic abuse c. A patient with surgery for bowel disease and anesthesia d. A patient with suppression of hydrochloric acid from medication

ANS: C Any surgery that involves direct manipulation of the bowel temporarily stops peristalsis. Anesthesia can also cause cessation of peristalsis. This condition, called an ileus, usually lasts about 24 to 48 hours. Fecal impaction, cathartic abuse, and medication to suppress hydrochloric acid will have bowel sounds, but they may be hypoactive or hyperactive.

The nurse is providing nutrition education to a newly immigrated Korean patient using the five food groups. In doing so, what should be the focus of the teaching? a. Discouraging the patient's ethnic food choices b. Changing the patient's diet to a more conventional American diet c. Including racial and ethnic practices with food preferences of the patient d. Comparing the patient's ethnic preferences with American dietary choices

ANS: C As a nurse, consider the food preferences of patients from different racial and ethnic groups, vegetarians, and others when planning diets. Initiation of a balanced diet is more important than conversion to what may be considered an American diet. Ethnic food choices may be just as nutritious as American choices. Foods should be chosen for their nutritive value and should not be compared with the American diet.

A nurse is caring for a patient who just underwent an intravenous pyelography that revealed a renal calculus obstructing the left ureter. What is the nurse's first priority in caring for this patient? a. Turn the patient on the right side to alleviate pressure on the left kidney. b. Encourage the patient to increase fluid intake to flush the obstruction. c. Monitor the patient for fever, rash, and difficulty breathing. d. Administer narcotic medications to the patient for pain.

ANS: C Assess for delayed hypersensitivity to the contrast media. Intravenous pyelography is performed by administering iodine-based dye to view functionality of the urinary system. Therefore, the first nursing priority is to assess the patient for an allergic reaction that could be life threatening. The nurse should then encourage the patient to drink fluids to flush dye resulting from the procedure. Narcotics can be administered but are not the first priority. Turning the patient on the side will not affect patient safety.

The nurse needs to move a patient up in bed using a drawsheet. The nurse has another nurse helping. In which order will the nurses perform the steps, beginning with the first one? 1. Grasp the drawsheet firmly near the patient. 2. Move the patient and drawsheet to the desired position. 3. Position one nurse at each side of the bed. 4. Place the drawsheet under the patient from shoulder to thigh. 5. Place your feet apart with a forward-backward stance. 6. Flex knees and hips and on the count of three shift weight from the front to back leg. a. 1, 4, 5, 6, 3, 2 b. 4, 1, 3, 5, 6, 2 c. 3, 4, 1, 5, 6, 2 d. 5, 6, 3, 1, 4, 2

ANS: C Assisting a patient up in bed with a drawsheet (two or three nurses): (1) Place the patient supine with the head of the bed flat. A nurse stands on each side of the bed. (2) Remove the pillow from under the patient's head and shoulders and place it at the head of the bed. (3) Turn the patient side to side to place the drawsheet under the patient, extending it from shoulders to thighs. (4) Return the patient to the supine position. (5) Fanfold the drawsheet on both sides, with each nurse grasping firmly near the patient. (6) Nurses place their feet apart with a forward-backward stance. Nurses should flex knees and hips. On the count of three, nurses should shift their weight from front to back leg and move the patient and drawsheet to the desired position in the bed.

The nurse is working on an orthopedic rehabilitation unit that requires lifting and positioning of patients. Which personal injury will the nurse most likely try to prevent? a. Arm b. Hip c. Back d. Ankle

ANS: C Back injuries are often the direct result of improper lifting and bending. The most common back injury is strain on the lumbar muscle group. While arm, hip, and ankle can occur, they are not as common as back.

Before giving the patient an intermittent gastric tube feeding, what should the nurse do? a. Make sure that the tube is secured to the gown with a safety pin. b. Inject air into the stomach via the tube and auscultate. c. Have the tube feeding at room temperature. d. Check to make sure pH is at least 5.

ANS: C Be sure that the formula is at room temperature. Cold formula causes gastric cramping and discomfort because the mouth and the esophagus do not warm the liquid. Do not use safety pins. Safety pins can become unfastened and may cause harm to the patient. Auscultation is no longer considered a reliable method for verification of tube placement because a tube inadvertently placed in the lungs, pharynx, or esophagus transmits sound similar to that of air entering the stomach. Gastric fluid of patient who has fasted for at least 4 hours usually has a pH of 1 to 4, especially when the patient is not receiving gastric-acid inhibitor.

The nurse will anticipate which diagnostic examination for a patient with black tarry stools? a. Ultrasound b. Barium enema c. Endoscopy d. Anorectal manometry

ANS: C Black tarry stools are an indication of bleeding in the GI tract; endoscopy would allow visualization of the bleeding. No other option (ultrasound, barium enema, and anorectal manometry) would allow GI visualization.

The nurse is completing an assessment on a patient who has a Stage IV pressure ulcer. The wound is odorous with a drain is currently in place. Which statement by the patient indicates issues with self-concept? a. ―I am so weak and tired. I just want to feel better.‖ b. ―I been thinking I will be ready to go home early next week.‖ c. ―I really need a bath and linen change right; I feel so awful.‖ d. ―I am hoping there will be something good to eat for my dinner tonight.‖

ANS: C Body image changes can influence self-concept. The wound is odorous, and a drain is in place. The patient who is asking for a bath and change in linens and states that this is awful gives you a clue that he or she may be concerned about the smell in the room. Factors that affect the patient's perception of the wound include the presence of scars, drains, odor from drainage, and temporary or permanent prosthetic devices. The patient's stating that he or she wants to feel better, talking about going home, and caring about what is for dinner could be interpreted as positive statements that indicate progress along the health journey.

Which intervention should be included as the nurse cleanses a wound? a. Allow the solution to flow from the most contaminated to the least contaminated. b. Scrub vigorously when applying noncytotoxic solution to the skin. c. Cleanse in a direction from the least contaminated area. d. Utilize clean gauze and clean gloves to cleanse a site.

ANS: C Cleanse in a direction from the least contaminated area, such as from the wound or incision to the surrounding skin. While cleansing surgical or traumatic wounds by applying noncytotoxic solution with sterile gauze or by irrigations is correct, vigorous scrubbing is inappropriate and can cause damage to the skin. Use gentle friction when applying solutions to the skin and allow irrigation to flow from the least to the most contaminated area.

An older adult's perineal skin is dry and thin with mild excoriation. When providing hygiene care after episodes of diarrhea, what should the nurse do? a. Thoroughly scrub the skin with a washcloth and hypoallergenic soap. b. Tape an occlusive moisture barrier pad to the patient's skin. c. Apply a skin protective ointment after perineal care. d. Massage the skin with light kneading pressure.

ANS: C Cleansing with a no-rinse cleanser and application of a barrier ointment should be done after each episode of diarrhea. Tape and occlusive dressings can damage skin. Excessive pressure and massage are inappropriate and may cause skin breakdown.

A nurse is assessing the skin of an immobilized patient. What will the nurse do? a. Assess the skin every 4 hours. b. Limit the amount of fluid intake. c. Use a standardized tool such as the Braden Scale. d. Have special times for inspection so as to not interrupt routine care.

ANS: C Consistently use a standardized tool, such as the Braden Scale. This identifies patients with a high risk for impaired skin integrity. Skin assessment can be as often as every hour. Limiting fluids can lead to dehydration, increasing skin breakdown. Observe the skin often during routine care.

The nurse suspects cystitis related to a lower urinary tract infection. Which clinical manifestation does the nurse expect the patient to report? a. Dysuria b. Flank pain c. Frequency d. Fever

ANS: C Cystitis is inflammation of the bladder; associated symptoms include hematuria, foul-smelling cloudy urine, and urgency/frequency. Dysuria is a common symptom of a lower urinary tract infection (bladder). Flank pain, fever, and chills are all signs of pyelonephritis (upper urinary tract).

The nurse notes that a patient has a black pressure ulcer on the left hip. Which event will the nurse anticipate when planning care for this patient? a. Increased monitoring of the wound condition b. Documenting the wound's status daily c. Surgical debridement of the wound d. Increased drainage from wound

ANS: C Debridement is the removal of nonviable necrotic (black) tissue. Removal of necrotic tissue is necessary to rid the ulcer of a source of infection, to enable visualization of the wound bed, and to provide a clean base for healing. A wound will not move through the phases of healing if the wound is infected. When treating a pressure ulcer, it is important to monitor and reassess the wound at least every 8 hours. Management of drainage will help keep the wound clean, but that is not the next step.

A nurse is reviewing urinary laboratory results. Which finding will cause the nurse to follow up? a. Protein level of 2 mg/100 mL b. Urine output of 80 mL/hr c. Specific gravity of 1.036 d. pH of 6.4

ANS: C Dehydration, reduced renal blood flow, and increase in antidiuretic hormone secretion elevate specific gravity. Normal specific gravity is 1.005 to 1.030. An output of 30 mL/hr or less for 2 or more hours would be cause for concern; 80 mL/hr is normal. The normal pH of urine is between 4.6 and 8.0. Protein up to 8 mg/100 mL is acceptable; however, values in excess of this could indicate renal disease.

The patient is admitted with a diagnosis of chronic anxiety. Which action is most appropriate for the nurse to take when providing holistic care? a. Focusing on finding quick remedies for the anxiety b. Realizing that the patient's only goal is relief of the anxiety c. Looking at how worry influences the patient's ability to function d. Helping the patient realize that there is little hope of relief from anxiety

ANS: C Do not just look at the patient's anxiety as a problem to solve with quick remedies, but rather look at how the anxiety influences the patient's ability to function and achieve goals established in life (not just anxiety relief). Mobilizing the patient's hope is central to a healing relationship.

A nurse is caring for a Roman Catholic patient. Which action will the nurse plan time for? a. Practice the Five Pillars b. Practice Blessingway c. Avoiding meat on Fridays d. The purity rituals

ANS: C Fasting on Fridays is practiced by some Roman Catholics. Hindus practice prayer and purity rituals. Blessingway is a practice of the Navajos that attempts to remove ill health by means of stories, songs, rituals, prayers, symbols, and sand paintings. Islams must be able to practice the Five Pillars of Islam.

A guaiac test is ordered for a patient. Which type of blood is the nurse checking for in this patient's stool? a. Bright red blood b. Dark black blood c. Microscopic d. Mucoid

ANS: C Fecal occult blood tests are used to test for blood that may be present in stool but cannot be seen by the naked eye (microscopic). This is usually indicative of a gastrointestinal bleed. All other options are incorrect. Detecting bright red blood, dark black blood, and blood that contains mucus (mucoid) is not the purpose of a guaiac test.

In providing prenatal care to a pregnant patient, what does the nurse teach the expectant mother? a. Calcium intake is especially important in the first trimester. b. Protein intake needs to decrease to preserve kidney function. c. Folic acid is needed to help prevent birth defects and anemia. d. Extra vitamins and minerals should be taken as much as possible.

ANS: C Folic acid intake is particularly important for DNA synthesis and growth of red blood cells. Inadequate intake may lead to fetal neural tube defects, anencephaly, or maternal megaloblastic anemia. Protein intake throughout pregnancy needs to increase to 60 g daily. Calcium intake is especially critical in the third trimester, when fetal bones mineralize. Prenatal care usually includes vitamin and mineral supplementation to ensure daily intakes; however, pregnant women should not take additional supplements beyond prescribed amounts.

The nurse is caring for a patient who is experiencing a full thickness wound repair. Which type of tissue will the nurse expect to observe when the wound is healing? a. Eschar b. Slough c. Granulation d. Purulent drainage

ANS: C Granulation tissue is red, moist tissue composed of new blood vessels, the presence of which indicates progression toward healing. Soft yellow or white tissue is characteristic of slough—a substance that needs to be removed for the wound to heal. Black or brown necrotic tissue is called eschar, which also needs to be removed for a wound to heal. Purulent drainage is indicative of an infection and will need to be resolved for the wound to heal.

A nurse is assisting a patient in making dietary choices that promote healthy bowel elimination. Which menu option should the nurse recommend? a. Broccoli and cheese soup with potato bread b. Turkey and mashed potatoes with brown gravy c. Grape and walnut chicken salad sandwich on whole wheat bread d. Dinner salad topped with hard-boiled eggs, cheese, and fat-free dressing

ANS: C Grapes and whole wheat bread are high fiber and should be chosen. Cheese, eggs, potato bread, and mashed potatoes do not contain as much fiber as whole wheat bread. A healthy diet for the bowel should include foods high in bulk-forming fiber. Whole grains, fresh fruit, and fresh vegetables are excellent sources. Foods without much fiber and with high levels of fat can slow down peristalsis, causing constipation.

The patient has been diagnosed with Helicobacter pylori. The nurse should encourage which action initially? a. Avoidance of wheat and oats b. Milkshakes as a nutritious snack c. Completion of antibiotic therapy d. Nonsteroidal antiinflammatory drug

ANS: C Helicobacter pylori, a bacterium that causes up to 85% of peptic ulcers, is confirmed by laboratory tests or a biopsy during endoscopy. Antibiotics treat and control the bacterial infection. Avoidance of wheat and oats are required for patients with celiac disease who must follow a gluten-free diet. Encourage patients to avoid foods that increase stomach acidity and pain such as caffeine, decaffeinated coffee, frequent milk intake, citric acid juices, and certain seasonings (hot chili peppers, chili powder, black pepper). Discourage smoking, alcohol, aspirin, and nonsteroidal antiinflammatory drugs (NSAIDs).

The patient has just been started on an enteral feeding and has developed diarrhea after being on the feeding for 2 hours. What does the nurse suspect is the most likely cause of the diarrhea? a. Antibiotic therapy b. Clostridium difficile c. Formula intolerance d. Bacterial contamination

ANS: C Hyperosmolar formulas can cause diarrhea or formula intolerance. If that is the case, the solution is to lower the rate, dilute the formula, or change to an isotonic formula. Antibiotics destroy normal intestinal flora and disturb the internal ecology, allowing for C. difficile toxin buildup. However, this takes time (more than 2 hours), and no indication suggests that this patient is on antibiotics. Bacterial contamination of the feeding usually occurs when feedings are left hanging for longer than 8 hours.

Which observation by the nurse best indicates that a continuous bladder irrigation for a patient following genitourinary surgery is effective? a. Output that is smaller than the amount instilled b. Blood clots or sediment in the drainage bag c. Bright red urine turns pink in the tubing d. Bladder distention with tenderness

ANS: C If urine is bright red or has clots, increase irrigation rate until drainage appears pink, indicating successful irrigation. Expect more output than fluid instilled because of urine production. If output is smaller than the amount instilled, suspect that the tube may be clogged. The presence of blood clots indicates the patient is still bleeding, while sediment could mean an infection or bleeding. The bladder should not be distended or tender; the irrigant may not be flowing freely if these occur, or the tube may be kinked or blocked.

In general, when a patient's energy requirements are completely met by kilocalorie (kcal) intake in food, which assessment finding will the nurse observe? a. Weight increases. b. Weight decreases. c. Weight does not change. d. Weight fluctuates daily.

ANS: C In general, when energy requirements are completely met by kilocalorie (kcal) intake in food, weight does not change. When kilocalories ingested exceed a person's energy demands, the individual gains weight. If kilocalories ingested fail to meet a person's energy requirement, the individual loses weight. Fluid, not kilocalories, causes daily weight fluctuations.

A nurse is caring for a male patient experiencing urinary retention. Which action should the nurse take first? a. Limit fluid intake. b. Insert a urinary catheter. c. Assist to a standing position. d. Ask for a diuretic medication.

ANS: C In some patients just helping them to a normal position to void prompts voiding. A urinary catheter would relieve urinary retention, but it is not the first measure; other nursing interventions should be tried before catheterization. Reducing fluids would reduce the amount of urine produced but would not alleviate the urine retention and is usually not recommended unless the retention is severe. Diuretic medication would increase urine production and may worsen the discomfort caused by urine retention.

A nurse is assisting the patient to perform isometric exercises. Which action will the nurse take? a. Encourage wearing tight shoes. b. Set the pace for the exercise session. c. Stop the exercise if pain is experienced. d. Force muscles or joints to go just beyond resistance.

ANS: C Instruct the patient to stop the activity if pain, fatigue, or discomfort is experienced. Assess for pain, shortness of breath, or a change in vital signs; if present, stop the exercise. Let each patient move at his or her own pace. Assess for joint limitations, and do not force a muscle or a joint during exercise. Teach patient to wear comfortable shoes and clothing for exercise.

The nurse and the patient have the same religious affiliation. Which action will the nurse take to support the patient spiritually? a. Must use a formal assessment tool to determine patient's beliefs. b. Assume that both have the same spiritual beliefs. c. Do not impose personal values on the patient. d. Skip the spiritual belief assessment.

ANS: C It is important not to impose personal value systems on the patient. This is particularly true when the patient's values and beliefs are similar to those of the nurse because it then becomes very easy to make false assumptions. It is not a must to use a formal assessment tool when assessing a patient's beliefs. It is important to conduct the spiritual belief assessment; conducting an assessment is therapeutic because it expresses a level of caring and support.

A nurse is checking orders. Which order should the nurse question? a. A normal saline enema to be repeated every 4 hours until stool is produced b. A hypertonic solution enema for a patient with fluid volume excess c. A Kayexalate enema for a patient with severe hypokalemia d. An oil retention enema for a patient with constipation

ANS: C Kayexalate binds to and helps excrete potassium, so it would be contraindicated in patients who are hypokalemic (have low potassium). Normal saline enemas can be repeated without risk of fluid or electrolyte imbalance. Hypertonic solutions are intended for patients who cannot handle large fluid volume and are contraindicated for dehydrated patients. Oil retention enemas lubricate the feces in the rectum and colon and are used for constipation.

. A nurse is caring for an 8-year-old patient who is embarrassed about urinating in bed at night. Which intervention should the nurse suggest to reduce the frequency of this occurrence? a. ―Set your alarm clock to wake you every 2 hours, so you can get up to void.‖ b. ―Line your bedding with plastic sheets to protect your mattress.‖ c. ―Drink your nightly glass of milk earlier in the evening.‖ d. ―Empty your bladder completely before going to bed.‖

ANS: C Nightly incontinence and nocturia are often resolved by limiting fluid intake 2 hours before bedtime. Setting the alarm clock to wake does not correct the physiological problem, nor does lining the bedding with plastic sheets. Emptying the bladder may help with early nighttime urination but will not affect urine produced throughout the night from late-night fluid intake.

Which laboratory data will be important for the nurse to monitor when a patient develops a pressure ulcer? a. Vitamin E b. Potassium c. Prealbumin d. Sodium

ANS: C Normal wound healing requires proper nutrition. Serum proteins are biochemical indicators of malnutrition, and serum albumin is probably the most frequently measured of these parameters. The best measurement of nutritional status is prealbumin because it reflects not only what the patient has ingested but also what the body has absorbed, digested, and metabolized. Zinc and copper are the minerals important for wound healing, not potassium and sodium. Vitamins A and C are important for wound healing, not vitamin E.

A nurse is assessing a patient with activity intolerance for possible orthostatic hypotension. Which finding will help confirm orthostatic hypotension? a. Blood pressure sitting 120/64; blood pressure 140/70 standing b. Blood pressure sitting 126/64; blood pressure 120/58 standing c. Blood pressure sitting 130/60; blood pressure 110/60 standing d. Blood pressure sitting 140/60; blood pressure 130/54 standing

ANS: C Orthostatic hypotension results in a drop of 20 mm Hg systolic or more in blood pressure when rising from sitting position (110/60). 120 to 140 means the blood pressure increased rather than dropped. 126 to 120 is only a six points' difference. 140 to 130 is only a 10 points' difference.

Which finding will alert the nurse to a potential wound dehiscence? a. Protrusion of visceral organs through a wound opening b. Chronic drainage of fluid through the incision site c. Report by patient that something has given way d. Drainage that is odorous and purulent

ANS: C Patients often report feeling as though something has given way with dehiscence. Dehiscence occurs when an incision fails to heal properly, and the layers of skin and tissue separate. It involves abdominal surgical wounds and occurs after a sudden strain such as coughing, vomiting, or sitting up in bed. Evisceration is seen when vital organs protrude through a wound opening. When there is an increase in serosanguineous drainage from a wound in the first few days after surgery, be alert for the potential for dehiscence. Infection is characterized by drainage that is odorous and purulent.

A patient had an ileostomy surgically placed 2 days ago. Which diet will the nurse recommend to the patient to ease the transition of the new ostomy? a. Eggs over easy, whole-wheat toast, and orange juice with pulp b. Chicken fried rice with fresh pineapple and iced tea c. Turkey sandwich on whole wheat bread and iced tea d. Fish sticks with sweet corn and soda

ANS: C Patients with colostomies have no diet restrictions other than the diet discussed for normal healthy bowel function, with adequate fiber and fluid to keep the stool softly formed. Fried foods can irritate digestion. Foods high in fiber will be useful later in the recovery process but can cause food blockage if the GI tract is not accustomed to digesting with an ileostomy. Foods with indigestible fiber such as sweet corn, popcorn, raw mushrooms, fresh pineapple, and Chinese cabbage could cause this problem.

Which nutritional instruction is a priority for the nurse to advise a patient about with an ileostomy? a. Keep fiber low. b. Eat large meals. c. Increase fluid intake. d. Chew food thoroughly.

ANS: C Patients with ileostomies will digest their food completely but will lose both fluid and salt through their stoma and will need to be sure to replace this to avoid dehydration. A good reminder for patients is to encourage drinking an 8-ounce glass of fluid when they empty their pouch. This helps patients to remember that they have greater fluid needs than they did before having an ileostomy. A low-fiber diet is not necessary. Eating large meals is not advised. While chewing food thoroughly is correct, it is not the priority; liquid is the priority.

A nurse is evaluating a nursing assistive personnel's (AP) care for a patient with an indwelling catheter. Which action by the AP will cause the nurse to intervene? a. Emptying the drainage bag when half full b. Kinking the catheter tubing to obtain a urine specimen c. Placing the drainage bag on the side rail of the patient's bed d. Securing the catheter tubing to the patient's thigh

ANS: C Placing the drainage bag on the side rail of the bed could allow the bag to be raised above the level of the bladder and urine to flow back into the bladder. The urine in the drainage bag is a medium for bacteria; allowing it to reenter the bladder can cause infection. A key intervention to prevent catheter-associated urinary tract infections is prevention of urine back flow from the tubing and bag into the bladder. All the rest are correct procedures and do not require follow-up. The drainage bag should be emptied when half full; an overfull drainage bag can create tension and pulling on the catheter, resulting in trauma to the urethra and/or urinary meatus and increasing risk for urinary tract infections. Urine specimens are obtained by temporarily kinking the tubing; a prolonged kink could lead to bladder distention. Failure to secure the catheter to the patient's thigh places the patient at risk for tissue injury from catheter dislodgment.

The nurse providing care to a bedridden patient raises the height of the bed. What is the rationale for the nurse's action? a. Narrows the nurse's base of support. b. Allows the nurse to bring feet closer together. c. Prevents a shift in the nurse's base of support. d. Shifts the nurse's center of gravity farther away from the base of support.

ANS: C Raising the height of the bed when performing a procedure prevents bending too far at the waist and shifting the base of support. Balance is maintained by proper body alignment and posture through two simple techniques. First, widen the base of support by separating the feet to a comfortable distance. Second, increase balance by bringing the center of gravity closer to the base of support

The nurse is caring for a patient who is professes to being an agnostic. Which information should the nurse consider when planning care for this patient belief system? a. The patient is devoid of spirituality. b. Is certain there is no such entity as God. c. Believes there is no known ultimate reality. d. Finds no meaning through relationship with others.

ANS: C Some people do not believe in the existence of God (atheist), or they believe that there is no known ultimate reality (agnostic). Nonetheless, spirituality is important regardless of a person's religious beliefs. Agnostics discover meaning in what they do or how they live because they find no ultimate meaning for the way things are. They believe that people bring meaning to what they do.

Which assessment finding is consistent with the diagnosis of malnutrition? a. Moist lips b. Pink conjunctivae c. Spoon-shaped nails d. Not easily plucked hair

ANS: C Spoon-shaped nails, koilonychia, is an indication of poor nutrition. All the others are normal findings. Lips should be moist, conjunctivae should be pink, and hair should not be easily plucked.

Which assessment question should the nurse ask if stress incontinence is suspected? a. ―Do you think your bladder feels distended?‖ b. ―Do you empty your bladder completely when you void?‖ c. ―Do you experience urine leakage when you cough or sneeze?‖ d. ―Do your symptoms increase with consumption of alcohol or caffeine?‖

ANS: C Stress incontinence can be related to intraabdominal pressure causing urine leakage, as would happen during coughing or sneezing. Asking the patient about the fullness of the bladder would rule out retention and overflow. An inability to void completely can refer to urge incontinence. Physiological causes and medications can affect elimination, but this is not related to stress incontinence.

The nurse learns about cultural issues involved in the patient's health care belief system and enables patients and families to achieve meaningful and supportive care. Which concept is the nurse demonstrating? a. Marginalized groups b. Health care disparity c. Transcultural nursing d. Culturally congruent care

ANS: D The nurse is demonstrating culturally congruent care. Culturally congruent care, or care that fits a person's life patterns, values, and system of meaning, provides meaningful and beneficial nursing care. Marginalized groups are populations left out or excluded. Health care disparities are differences among populations in the availability, accessibility, and quality of health care services (e.g., screening, diagnostic, treatment, management, and rehabilitation) aimed at prevention, treatment, and management of diseases and their complications. Transcultural nursing is a comparative study of cultures in order to understand their similarities (culture that is universal) and the differences among them (culture that is specific to particular groups).

The nurse is caring for a patient who has experienced a laparoscopic appendectomy. For which type of healing will the nurse focus the care plan? a. Partial-thickness repair b. Secondary intention c. Tertiary intention d. Primary intention

ANS: D A clean surgical incision is an example of a wound with little loss of tissue that heals with primary intention. The skin edges are approximated or closed, and the risk for infection is low. Partial-thickness repairs are done on partial-thickness wounds that are shallow, involving loss of the epidermis and maybe partial loss of the dermis. These wounds heal by regeneration because the epidermis regenerates. Tertiary intention is seen when a wound is left open for several days, and then the wound edges are approximated. Wound closure is delayed until the risk of infection is resolved. A wound involving loss of tissue such as a burn or a pressure ulcer or laceration heals by secondary intention. The wound is left open until it becomes filled with scar tissue. It takes longer for a wound to heal by secondary intention; thus the chance of infection is greater.

A patient is experiencing a fecal impaction. Which portion of the colon will the nurse assess? a. Descending b. Transverse c. Ascending d. Rectum

ANS: D A fecal impaction is a collection of hardened feces wedged in the rectum that cannot be expelled. It results from unrelieved constipation. Feces at this point in the colon contain the least amount of moisture. Feces found in the ascending, transverse, and descending colon still consist mostly of liquid and do not form a hardened mass.

The nurse is caring for a patient who has experienced a total abdominal hysterectomy. Which nursing observation related to the incision will indicate the patient is experiencing a complication of wound healing? a. Patient reporting, ―My incision is hurting.‖ b. Approximation of the incision edges has occurred. c. Patient asks, ―Why has my incision started to itch?‖ d. The incision appears both swollen and bluish in color.

ANS: D A hematoma is a localized collection of blood underneath the tissues. It appears as swelling, change in color, sensation, or warmth or a mass that often takes on a bluish discoloration. A hematoma near a major artery or vein is dangerous because it can put pressure on the vein or artery and obstruct blood flow. Itching is not a complication. Incisions should be approximated with edges together; this is a sign of normal healing. After surgery, when nerves in the skin and tissues have been traumatized by the surgical procedure, it is expected that the patient will experience pain.

Which nursing observation will indicate the patient's wound healed by the process of secondary intention? a. Minimal loss of tissue function b. Permanent dark redness at site c. Minimal scar tissue d. Scarring that may be severe

ANS: D A wound healing by secondary intention takes longer than one healing by primary intention. The wound is left open until it becomes filled with scar tissue. If the scarring is severe, permanent loss of function often occurs. Wounds that heal by primary intention heal quickly with minimal scarring. Scar tissue contains few pigmented cells and has a lighter color than normal skin

The patient has a calculated body mass index (BMI) of 34. How will the nurse classify this finding? a. Normal weight b. Underweight c. Overweight d. Obese

ANS: D BMI greater than 30 is defined as obesity. BMI between 25 and 30 is classified as overweight. BMI from 18.5 to 24.9 is normal. BMI under 18.5 is underweight.

A nurse is reviewing results from a urine specimen. What will the nurse expect to see in a patient with a urinary tract infection? a. Casts b. Protein c. Crystals d. Bacteria

ANS: D Bacteria in the urine along with other symptoms support a diagnosis of urinary tract infection. Crystals would be seen with renal stone formation. Casts indicate renal disease. Protein indicates kidney function and damage to the glomerular membrane such as in glomerulonephritis.

When a comatose patient develops a Stage II pressure ulcer, the nurse includes the nursing diagnosis of Risk for infection to the care plan. Which is the best goal for this patient? a. The patient will state what to look for with regard to an infection. b. The patient's family will demonstrate specific care of the wound site. c. The patient's family members will wash their hands when visiting the patient. d. The patient will remain free of odorous or purulent drainage from the wound.

ANS: D Because the patient has an open wound and the skin is no longer intact to protect the tissue, the patient is at increased risk for infection. The nurse will be assessing the patient for signs and symptoms of infection, including an increase in temperature, an increase in white count, and odorous and purulent drainage from the wound. The patient is unconscious and is unable to communicate the signs and symptoms of infection. It is important for the patient's family to be able to demonstrate how to care for the wound and wash their hands, but these statements are not goals or outcomes for this nursing diagnosis.

In teaching mothers-to-be about infant nutrition, which instruction should the nurse provide? a. Supplement breast milk with corn syrup. b. Give cow's milk during the first year of life. c. Add honey to infant formulas for increased energy. d. Provide breast milk or formula for the first 4 to 6 months.

ANS: D Breast milk or formula provides sufficient nutrition for the first 4 to 6 months of life. Infants should not have regular cow's milk during the first year of life. It is too concentrated for an infant's kidneys to manage, increases the risk of milk product allergies, and is a poor source of iron and vitamins C and E. Furthermore, children under 1 year of age should never ingest honey and corn syrup products because they are potential sources of the botulism toxin, which increases the risk of infant death.

A patient expresses concerns over having black stool. The fecal occult test is negative. Which response by the nurse is most appropriate? a. ―This is probably a false negative; we should rerun the test.‖ b. ―You should schedule a colonoscopy as soon as possible.‖ c. ―Are you under a lot of stress?‖ d. ―Do you take iron supplements?‖

ANS: D Certain medications and supplements, such as iron, can alter the color of stool (black or tarry). Since the fecal occult test is negative, bleeding is not occurring. The fecal occult test takes three separate samples over a period of time and is a fairly reliable test. A colonoscopy is health prevention screening that should be done every 5 to 10 years; it is not the nurse's initial priority. Stress alters GI motility and stool consistency, not color.

The nurse is assessing a patient for nutritional status. Which action will the nurse take? a. Forego the assessment in the presence of chronic disease. b. Use the Mini Nutritional Assessment for pediatric patients. c. Choose a single objective tool that fits the patient's condition. d. Combine multiple objective measures with subjective measures

ANS: D Combine multiple objective measures with subjective measures related to nutrition to adequately screen for nutritional problems. Using a single objective measure is ineffective in predicting risk of nutritional problems. Chronic disease and increased metabolic requirements are risk factors for the development of nutritional problems; these patients may be in critical need of this assessment. The Mini Nutritional Assessment is used for screening older adults in home care programs, nursing homes, and hospitals.

The patient who has been diagnosed with cardiovascular disease and placed on a low-fat diet, asks the nurse, ―How much fat should I have? I guess the less fat, the better.‖ Which information will the nurse include in the teaching session? a. Cholesterol intake needs to be less than 300 mg/day. b. Fats have no significance in health and the incidence of disease. c. All fats come from external sources, so this can be easily controlled. d. Deficiencies occur when fat intake falls below 10% of daily nutrition.

ANS: D Deficiency occurs when fat intake falls below 10% of daily nutrition. While keeping cholesterol below 300 mg is correct according to the American Heart Association, it does not answer the patient's question about fat. Various types of fatty acids have significance for health and for the incidence of disease and are referred to in dietary guidelines. Linoleic acid and arachidonic acid are important for metabolic processes but are manufactured by the body when linoleic acid is available from the diet.

The nurse is caring for a patient receiving total parenteral nutrition (TPN). Which action will the nurse take? a. Run lipids for no longer than 24 hours. b. Take down a running bag of TPN after 36 hours. c. Clean injection port with alcohol 5 seconds before and after use. d. Wear a sterile mask when changing the central venous catheter dressing

ANS: D During central venous catheter dressing changes, always use a sterile mask and gloves, and assess insertion sites for signs and symptoms of infection. To avoid infection, change the TPN infusion tubing every 24 hours, and do not hang a single container of PN for longer than 24 hours or lipids longer than 12 hours.

A nurse is providing range of motion to the shoulder and must perform external rotation. Which action will the nurse take? a. Moves patient's arm in a full circle. b. Moves patient's arm cross the body as far as possible. c. Moves patient's arm behind body, keeping elbow straight. d. Moves patient's arm until thumb is upward and lateral to head with elbow flexed.

ANS: D External rotation: With elbow flexed, move arm until thumb is upward and lateral to head. Circumduction: Move arm in full circle (Circumduction is combination of all movements of ball-and-socket joint.) Adduction: Lower arm sideways and across body as far as possible. Hyperextension: Move arm behind body, keeping elbow straight.

A nurse is asked how many kcal/g are provided by fats. How should the nurse answer? a. 3 b. 4 c. 6 d. 9

ANS: D Fats (lipids) are the most calorie-dense nutrient, providing 9 kcal/g. Carbohydrates and protein provide 4 kcal/g.

Which clinical manifestation will the nurse expect to observe in a patient with excessive white blood cells present in the urine? a. Reduced urine specific gravity b. Increased blood pressure c. Abnormal blood sugar d. Fever with chills

ANS: D Fever and chills may be observed. The presence of white blood cells in urine indicates a urinary tract infection or inflammation. Overhydration, early renal disease, and inadequate antidiuretic hormone secretion reduce specific gravity. Increased blood pressure is associated with renal disease or damage and some medications. Abnormal blood sugars would be seen in someone with ketones in the urine or a patient with diabetes.

A nurse is developing an exercise plan for a middle-aged patient. In which order will the nurse instruct the patient to execute the plan, beginning with the first step? 1. Design the fitness program. 2. Assemble equipment. 3. Assess fitness level. 4. Monitor progress. 5. Get started. a. 5, 1, 3, 2, 4 b. 1, 2, 3, 5, 4 c. 2, 5, 3, 1, 4 d. 3, 1, 2, 5, 4

ANS: D Five steps to beginning an exercise program are Step 1: Assess fitness level; Step 2: Design the fitness program; Step 3: Assemble equipment; Step 4: Get started; and Step 5: Monitor progress.

The nurse is admitting a patient who has been diagnosed as having had a stroke. The health care provider writes orders for ―ROM as needed.‖ What should the nurse do next? a. Restrict patient's mobility as much as possible. b. Realize the patient is unable to move extremities. c. Move all the patient's extremities. d. Further assess the patient.

ANS: D Further assessment of the patient is needed to determine what the patient is able to perform. Some patients are able to move some joints actively, whereas the nurse passively moves others. With a weak patient, the nurse may have to support an extremity while the patient performs the movement. In general, exercises need to be as active as health and mobility allow

. The nurse is observing the way a patient walks. Which aspect is the nurse assessing? a. Activity tolerance b. Body alignment c. Range of motion d. Gait

ANS: D Gait describes a particular manner or style of walking. Activity tolerance is the type and amount of exercise or work that a person is able to perform. Body alignment refers to the position of the joints, tendons, ligaments, and muscles while standing, sitting, and lying. Range of motion is the maximum amount of movement available at a joint in one of the three planes of the body: sagittal, frontal, or transverse.

A nurse is performing passive range of motion (ROM) and splinting on an at-risk patient. The absence of which finding will indicate goal achievement for the nurse's action? a. Atelectasis b. Renal calculi c. Pressure ulcers d. Joint contractures

ANS: D Goal achievement for passive ROM is prevention of joint contractures. Contractures develop in joints not moved periodically through their full ROM. ROM exercises reduce the risk of contractures. Researchers noted that prompt use of splinting with prescribed ROM exercises reduced contractures and improved active range of joint motion in affected lower extremities. Deep breathing and coughing and using an incentive spirometer will help prevent atelectasis. Adequate hydration helps prevent renal calculi and urinary tract infections. Interventions aimed at prevention of pressure ulcers include positioning, skin care, and the use of therapeutic devices to relieve pressure.

The nurse is caring for a postoperative patient recovering from a medial meniscus repair of the right knee. Which action should the nurse take to assist with pain management? a. Monitor vital signs every 15 minutes. b. Check pulses in the right foot. c. Keep the leg dependent. d. Apply ice.

ANS: D Ice assists in preventing edema formation, controlling bleeding, and anesthetizing the body part. Elevation (not dependent) assists in preventing edema, which in turn can cause pain. Monitoring vital signs every 15 minutes is routine postoperative care and includes a pain assessment but in itself is not an intervention that decreases pain. Checking the pulses is important to monitor the circulation of the extremity but in itself is not a pain management intervention.

A nurse is inserting a catheter into a female patient. When the nurse inserts the catheter, no urine is obtained. The nurse suspects the catheter is not in the urethra. What should the nurse do? a. Throw the catheter way and begin again. b. Fill the balloon with the recommended sterile water. c. Remove the catheter, wipe with alcohol, and reinsert after lubrication. d. Leave the catheter in the vagina as a landmark for insertion of a new, sterile catheter

ANS: D If no urine appears, the catheter may be in the vagina. If misplaced, leave the catheter in the vagina as a landmark to indicate where not to insert, and insert another sterile catheter. The catheter should be left in place until the new, sterile catheter is inserted. The balloon should not be filled since the catheter is in the vagina. The catheter must be sterile; using alcohol will not make the catheter sterile.

A nurse implementing the principles of Intersectionality will focus on what patient characteristic? a. Values b. Illness c. Health d. Experiences

ANS: D Intersectionality is a research and policy model used to study the complexities of people's lives and experiences. Illness and health are outcomes viewed in respect to the patient's experiences. The patient's values and beliefs are formulated by their life experiences.

A nurse is caring for a patient who is experiencing some symptoms related to arthritis. The nurse is teaching the patient about this process. Which information will the nurse include in the teaching session? a. This will affect synovial fluid. b. This will affect the body systemically. c. This involves mostly non-weight-bearing joints. d. This involves an increased risk for impaired weight bearing.

ANS: D Joint degeneration, which can occur with inflammatory and noninflammatory disease, is marked by changes in articular cartilage combined with overgrowth of bone at the articular ends. Degenerative changes commonly affect weight-bearing joints. Synovial fluid is normal in noninflammatory diseases. Inflammatory joint disease (e.g., arthritis) is characterized by inflammation or destruction of the synovial membrane and articular cartilage and by systemic signs of inflammation.

A nurse is preparing a care plan for a patient who is immobile. Which psychosocial aspect will the nurse assess for? a. Loss of bone mass b. Loss of strength c. Loss of weight d. Loss of hope

ANS: D Loss of hope is a psychosocial aspect. Patients with restricted mobility may have some depression. Depression is an affective disorder characterized by exaggerated feelings of sadness, melancholy, dejection, worthlessness, emptiness, and hopelessness out of proportion to reality. All the rest are physiological aspects: bone mass, strength, and weight.

A nurse is caring for an older, immobile patient whose condition requires a supine position. Which metabolic alteration will the nurse monitor for in this patient? a. Increased appetite b. Increased diarrhea c. Increased metabolic rate d. Increased pulse rate

ANS: D Lying down increases cardiac workload and results in an increased pulse rate. In older adults, the heart rate often does not tolerate the added workload, and a form of cardiac failure may develop. Immobility disrupts normal metabolic functioning: decreasing the metabolic rate, altering the metabolism of carbohydrates, fats, and proteins; causing fluid, electrolyte, and calcium imbalances; and causing gastrointestinal disturbances such as decreased appetite and slowing of peristalsis, leading to constipation.

A patient recovering from a hip fracture is having difficulty defecating into a bedpan while lying in bed. Which action by the nurse will assist the patient in having a successful bowel movement? a. Preparing to administer a barium enema b. Withholding narcotic pain medication c. Administering laxatives to the patient d. Raising the head of the bed

ANS: D Lying in bed is an unnatural position; raising the head of the bed assists the patient into a more normal position that allows proper contraction of muscles for elimination. Laxatives would not give the patient control over bowel movements. A barium enema is a diagnostic test, not an intervention to promote defecation. Pain-relief measures should be given; however, preventative action should be taken to prevent constipation.

The nurse is caring for a surgical patient. Which intervention is most important for the nurse to complete to decrease the risk of pressure ulcers and encourage the patient's willingness and ability to increase mobility? a. Explain the risks of immobility to the patient. b. Turn the patient every 3 hours while in bed. c. Encourage the patient to sit up in the chair. d. Provide analgesic medication as ordered.

ANS: D Maintaining adequate pain control (providing analgesic medications) and patient comfort increases the patient's willingness and ability to increase mobility, which in turn reduces pressure ulcer risks. Although sitting in the chair is beneficial, it does not increase mobility or provide pain control. Explaining the risk of immobility is important for the patient because it may impact the patient's willingness but not his or her ability. Turning the patient is important for decreasing pressure ulcers but needs to be done every 2 hours and, again, does not influence the patient's ability to increase mobility.

A patient is on a full liquid diet. Which food item choice by the patient will cause the nurse to intervene? a. Custard b. Frozen yogurt c. Pureed vegetables d. Mashed potatoes and gravy

ANS: D Mashed potatoes and gravy are on a dysphagia, mechanical soft, soft and regular diets but are not components of a full liquid diet. The nurse will need to provide teaching on what is allowed on the diet. Custard, frozen yogurt, and pureed vegetables are all on a full liquid diet.

The nurse is preparing to insert a nasogastric tube. To determine the length of the tube needed to be inserted, how should the nurse measure the tube? a. From the tip of the nose to the earlobe b. From the tip of the earlobe to the xiphoid process c. From the tip of the earlobe to the nose to the xiphoid process d. From the tip of the nose to the earlobe to the xiphoid process

ANS: D Measure distance from the tip of the nose to the earlobe to the xiphoid process of the sternum. This approximates the distance from the nose to the stomach in 98% of patients. For duodenal or jejunal placement, an additional 20 to 30 cm is required.

In providing diet education for a patient on a low-fat diet, which information is important for the nurse to share? a. Polyunsaturated fats should be less than 7% of the total calories. b. Trans fat should be less than 7% of the total calories. c. Unsaturated fats are found mostly in animal sources. d. Saturated fats are found mostly in animal sources.

ANS: D Most animal fats have high proportions of saturated fatty acids, whereas vegetable fats have higher amounts of unsaturated and polyunsaturated fatty acids. Linoleic acid, an unsaturated fatty acid, is the only essential fatty acid in humans. Diet recommendations include limiting saturated and trans-fat to less than 10%.

A nurse is preparing a patient for a magnetic resonance imaging (MRI) scan. Which nursing action is most important? a. Ensuring that the patient does not eat or drink 2 hours before the examination b. Administering a colon cleansing product 6 hours before the examination c. Obtaining an order for a pain medication before the test is performed d. Removing all of the patient's metallic jewelry

ANS: D No jewelry or metal products should be in the same room as an MRI machine because of the high-power magnet used in the machine. The patient needs to be NPO 4 to 6 hours before the examination. Colon cleansing products are not necessary for MRIs. Pain medication is not needed before the examination is performed

A nurse is providing passive range of motion (ROM) for a patient with impaired mobility. Which technique will the nurse use for each movement? a. Each movement is repeated 5 times by the patient. b. Each movement is performed until the patient reports pain. c. Each movement is completed quickly and smoothly by the nurse. d. Each movement is moved just to the point of resistance by the nurse.

ANS: D Passive ROM exercises are performed by the nurse. Carry out movements slowly and smoothly, just to the point of resistance; ROM should not cause pain. Never force a joint beyond its capacity. Each movement needs to be repeated 5 times during the session. The patient moves all joints through ROM unassisted in active ROM.

The nurse is caring for a patient diagnosed with C. difficile. Which nursing actions will have the greatest impact in preventing the spread of the bacteria? a. Appropriate disposal of contaminated items in biohazard bags b. Monthly inservices about contact precautions c. Mandatory cultures on all patients d. Proper hand hygiene techniques

ANS: D Proper hand hygiene is the best way to prevent the spread of bacteria. Soap and water are mandatory. Monthly inservices place emphasis on education, not on action. Biohazard bags are appropriate but cannot be used on every item that C. difficile comes in contact with, such as a human. Mandatory cultures are expensive and unnecessary and would not prevent the spread of bacteria.

Which patient diagnosis increases the risk for developing neurogenic dysphagia? a. Benign peptic stricture b. Muscular dystrophy c. Myasthenia gravis d. Stroke

ANS: D Stroke is the only cause of dysphagia in this list that is considered neurogenic. Myasthenia gravis and muscular dystrophy are considered myogenic in origin, whereas benign peptic stricture is considered obstructive.

The nurse is teaching a health class about the My Plate program. Which guidelines will the nurse include in the teaching session? a. Balancing sodium and potassium b. Decreasing water consumption c. Increasing portion size d. Balancing calories

ANS: D The MyPlate program includes guidelines for balancing calories; decreasing portion size; increasing healthy foods; increasing water consumption; and decreasing fats, sodium, and sugars. It does not balance sodium and potassium.

The patient has a risk for skin impairment and has a 15 on the Braden Scale upon admission. The nurse has implemented interventions. Upon reassessment, which Braden score will be the best sign that the risk for skin breakdown is removed? a. 12 b. 13 c. 20 d. 23

ANS: D The best sign is a perfect score of 23. The Braden Scale is composed of six subscales: sensory perception, moisture, activity, mobility, nutrition, and friction and shear. The total score ranges from 6 to 23, and a lower total score indicates a higher risk for pressure ulcer development. The cutoff score for onset of pressure ulcer risk with the Braden Scale in the general adult population is 18.

A nurse is teaching a patient about proteins that must be obtained through the diet since they cannot be synthesized in the body. Which term used by the patient indicates teaching is successful? a. Amino acids b. Triglycerides c. Dispensable amino acids d. Indispensable amino acids

ANS: D The body does not synthesize indispensable amino acids, so these need to be provided in the diet. The simplest form of protein is the amino acid. The body synthesizes dispensable amino acids. Triglycerides are made up of three fatty acids attached to a glycerol.

The patient has the nursing diagnosis of Impaired physical mobility related to pain in the left shoulder. Which priority action will the nurse take? a. Encourage the patient to do self-care. b. Keep the patient as mobile as possible. c. Encourage the patient to perform ROM. d. Assist the patient with comfort measures.

ANS: D The diagnosis related to pain requires the nurse to assist the patient with comfort measures so that the patient is then willing and more able to move. Pain must be controlled so the patient will not be reluctant to initiate movement. The diagnosis related to reluctance to initiate movement requires interventions aimed at keeping the patient as mobile as possible and encouraging the patient to perform self-care and ROM.

The nurse is teaching a health class about the gastrointestinal tract. The nurse will explain that which portion of the digestive tract absorbs most of the nutrients? a. Ileum b. Cecum c. Stomach d. Duodenum

ANS: D The duodenum and jejunum absorb most nutrients and electrolytes in the small intestine. The ileum absorbs certain vitamins, iron, and bile salts. Food is broken down in the stomach. The cecum is the beginning of the large intestine.

A nurse is watching a nursing assistive personnel (AP) perform a postvoid bladder scan on a female with a previous hysterectomy. Which action will require the nurse to follow up? a. Palpates the patient's symphysis pubis. b. Wipes scanner head with alcohol pad. c. Applies a generous amount of gel. d. Sets the scanner to female.

ANS: D The nurse will follow up if the AP sets the scanner to female. Women who have had a hysterectomy should be designated as male. All the rest are correct and require no follow-up. The NAP should palpate the symphysis pubis, the scanner head should be cleaned with an alcohol pad, and a generous amount of gel should be applied.

Which is the best explanation for the nurse to provide when teaching the patient, the reason for the binder after an open abdominal aortic aneurysm repair? a. It reduces edema at the surgical site. b. It secures the dressing in place. c. It immobilizes the abdomen. d. It supports the abdomen.

ANS: D The patient has a large abdominal incision. This incision will need support, and an abdominal binder will support this wound, especially during movement, as well as during deep breathing and coughing. A binder can be used to immobilize a body part (e.g., an elastic bandage applied around a sprained ankle). A binder can be used to prevent edema, for example, in an extremity but in this case is not used to reduce edema at a surgical site. A binder can be used to secure dressings such as elastic webbing applied around a leg after vein stripping.

To obtain a clean-voided urine specimen from a female patient, what should the nurse teach the patient to do? a. Cleanse the urethral meatus from the area of most contamination to least. b. Initiate the first part of the urine stream directly into the collection cup. c. Drink fluids 5 minutes before collecting the urine specimen. d. Hold the labia apart while voiding into the specimen cup.

ANS: D The patient should hold the labia apart to reduce bacterial levels in the specimen. The urethral meatus should be cleansed from the area of least contamination to greatest contamination (or front to back). The initial stream flushes out microorganisms in the urethra and prevents bacterial transmission in the specimen. Drink fluids 30 minutes before giving a specimen.

The nurse is managing bowel training for a patient. To which patient is the nurse most likely providing care? a. A 25-year-old patient with diarrhea b. A 30-year-old patient with Clostridium difficile c. A 40-year-old patient with an ileostomy d. A 70-year-old patient with stool incontinence

ANS: D The patient with chronic constipation or fecal incontinence secondary to cognitive impairment may benefit from bowel training, also called habit training. An ileostomy, diarrhea, and C. difficile all relate to uncontrollable bowel movements, for which no method can be used to set up a schedule of elimination.

The nurse is caring for an older-adult patient with a diagnosis of urinary tract infection (UTI). Upon assessment the nurse finds the patient confused and agitated. How will the nurse interpret these assessment findings? a. These are normal signs of aging. b. These are early signs of dementia. c. These are purely psychological in origin. d. These are common manifestation with UTIs.

ANS: D The primary symptom of compromised older patients with an acute urinary tract infection or fever is confusion. Acute confusion in older adults is not normal; a thorough nursing assessment is the priority. With the diagnosis of urinary tract infection, these are not early signs of dementia and they are not purely psychological.

The patient has a catheter that must be irrigated. The nurse is using a needleless closed irrigation technique. In which order will the nurse perform the steps, starting with the first one? 1. Clean injection port. 2. Inject prescribed solution. 3. Twist needleless syringe into port. 4. Remove clamp and allow to drain. 5. Clamp catheter just below specimen port. 6. Draw up prescribed amount of sterile solution ordered. a. 3, 2, 6, 1, 5, 4 b. 5, 6, 1, 2, 3, 4 c. 1, 5, 6, 3, 2, 4 d. 6, 5, 1, 3, 2, 4

ANS: D The steps for irrigating with a needleless closed irrigation technique is as follows: draw up in a syringe the prescribed amount of medication or sterile solution; clamp indwelling retention catheter just below specimen port; using circular motion, clean injection port with antiseptic swab; insert tip of needleless syringe using twisting motion into irrigation port; slowly and evenly inject fluid into catheter and bladder; and withdraw syringe, remove clamp, and allow solution to drain into drainage bag.

The patient is unable to move self and needs to be pulled up in bed. What will the nurse do to make this procedure safe? a. Place the pillow under the patient's head and shoulders. b. Do by self if the bed is in the flat position. c. Place the side rails in the up position. d. Use a friction-reducing device.

ANS: D This is not a one-person task. Helping a patient move up in bed without help from other co-workers or without the aid of an assistive device (e.g., friction-reducing pad) is not recommended and is not considered safe for the patient or the nurse. Remove the pillow from under head and shoulders and place it at the head of the bed to prevent striking the patient's head against the head of the bed. When pulling a patient up in bed, the bed should be flat to gain gravity assistance, and the side rails should be down.

The patient is to receive multiple medications via the nasogastric tube. The nurse is concerned that the tube may become clogged. Which action is best for the nurse to take? a. Instill nonliquid medications without diluting. b. Irrigate the tube with 60 mL of water after all medications are given. c. Mix all medications together to decrease the number of administrations. d. Check with the pharmacy for availability of the liquid forms of medications.

ANS: D Use liquid medications when available to prevent tube occlusion. Irrigate with 30 mL of water before and after each medication per tube. Completely dissolve crushed medications in liquid if liquid medication is not available. Read pharmacological information on compatibility of drugs and formula before mixing medications.

A nurse is inserting an indwelling urinary catheter for a male patient. Which action will the nurse take? a. Hold the shaft of the penis at a 60-degree angle. b. Hold the shaft of the penis with the dominant hand. c. Cleanse the meatus 3 times with the same cotton ball from clean to dirty. d. Cleanse the meatus with circular strokes beginning at the meatus and working outward.

ANS: D Using the uncontaminated dominant hand, cleanse the meatus with cotton balls/swab sticks, using circular strokes, beginning at the meatus and working outward in a spiral motion. Repeat 3 times using a clean cotton ball/swabstick each time. With the nondominant hand (now contaminated), retract the foreskin (if uncircumcised) and gently grasp the penis at the shaft just below the glans. Hold the shaft of the penis at a right angle to the body.

Which item should the nurse use first to assist in staging an ulcer on the heel of a darkly pigmented skin patient? a. Disposable measuring tape b. Cotton-tipped applicator c. Sterile gloves d. Natural light

ANS: D When assessing a patient with darkly pigmented skin, proper lighting is essential to accurately complete the first step in assessment—inspection—and the entire assessment process. Natural light is recommended. Fluorescent light sources can produce blue tones on darkly pigmented skin and can interfere with an accurate assessment. Other items that could possibly be used during the assessment include gloves for infection control, a disposable measuring device to measure the size of the wound, and a cotton-tipped applicator to measure the depth of the wound, but these items are not the first items used.

A patient is admitted after having experienced a stroke. The outcome of this disorder is uncertain, but the patient is unable to move the right arm and leg. The nurse starts passive range-of-motion (ROM) exercises. Which finding indicates successful goal achievement? a. Heart rate decreased. b. Contractures developed. c. Muscle strength improved. d. Joint mobility maintained.

ANS: D When patients cannot participate in active ROM, maintain joint mobility, and prevent contractures by implementing passive ROM into the plan of care. Exercise and active ROM can improve muscle strength. ROM is not performed for the heart but for the joints.

A patient with a three-way indwelling urinary catheter and CBI complains of lower abdominal pain and distention after surgery. What should be the nurse's initial intervention(s)? (Select all that apply.) 1. Increase the rate of the CBI. 2. Assess the patency of the drainage system. 3. Measure urine output. 4. Assess vital signs. 5. Administer ordered pain medication.

ANS:2,3

A 46-year-old patient is admitted to the emergency department following an automobile accident. The patient has a pelvic fracture and is ordered on bed rest and placed in an immobilization device to limit further injury until the fracture can safely be repaired. Which measures are appropriate for this patient to prevent complications of bed rest? (Select all that apply.) 1. Administer IV analgesic as ordered. 2. Have patient perform incentive spirometry. 3. Support patient in active assisted ROM exercises of upper extremities. 4. Provide patient a low-calorie diet. 5. Apply SCDs to legs.

ANS:2,3,5

After abdominal surgery, the patient is on the surgical unit with an indwelling urinary catheter placed. What aspects of care for this patient can be delegated to the assistive personnel (AP)? (Select all that apply.) 1. Assessing the patient for any postoperative issues with the indwelling catheter 2. Assisting the nurse with patient positioning and maintaining privacy during catheter care 3. Teaching the patient signs and symptoms of a UTI 4. Reporting to the nurse any patient discomfort or fever 5. Reporting any abnormal color, odor, or amount of urine in the drainage bag

ANS:2,4,5

Describe enuresis

This occurs when a child who is old enough to have bladder control (3 years and older) does not have control

At what age do children start to have voluntary control of their bladder?

at 18-24 months children can voluntarily control bladder

What is the description of first bowel movement?

black, thick, sticky, tar-like, shiny. this is a mixture of bile, cells, mucus, and amniotic intestinal fluids. Usually dosnet have a smell

At what age do children start to have voluntary control of their bowel?

can start to control bowel at 2-3 years

What is the first bowel movement called?

meconium

Describe encopresis

occurs when a child who is old enough to have bowel control (4 years old and older) does not have control


Set pelajaran terkait

Chapter 13: Essential Trace & Ultratrace Minerals

View Set

Chapter 4 Homework (Respiratory System)

View Set

20th Century Art History Part Two

View Set

Inflammation & Autoimmune Disorders NCLEX

View Set